You are on page 1of 237

Mahler!

s Guide to
Financial Economics
Exam MFE

prepared by
Howard C. Mahler, FCAS
Copyright !2014 by Howard C. Mahler.
Study Aid 2014-MFE
Howard Mahler
hmahler@mac.com
www.howardmahler.com/Teaching
Mahler!s Guide to Financial Economics
Copyright !2014 by Howard C. Mahler.
Concepts in Derivatives Markets by Robert L. McDonald are demonstrated.
1

Information in bold or sections whose title is in bold are more important for passing the exam.
Larger bold type indicates it is extremely important. Information presented in italics (and sections
whose titles are in italics) should not be needed to directly answer exam questions and should be
skipped on first reading. It is provided to aid the reader!s overall understanding of the subject, and to
be useful in practical applications.
Highly Recommended problems are double underlined.
Recommended problems are underlined.
2

Solutions to the problems in each section are at the end of that section.
Section # Pages Section Name
A 1 11-29 Introduction
2 30-60 European Options
3 61-99 Properties of Premiums of European Options
B 4 100-137 Put-Call Parity
5 138-150 Bounds on Premiums of European Options
6 151-169 Options on Currency
7 170-176 Exchange Options
C 8 177-183 Futures Contracts
9 184-190 Synthetic Positions
10 191-236 American Options
11 237-261 Replicating Portfolios
12 262-283 Risk Neutral Probabilities
D 13 284-301 Utility Theory and Risk Neutral Pricing
14 302-357 Binomial Trees, Risk Neutral Probabilities
15 358-371 Binomial Trees, Valuing Options on Other Assets
16 372-388 Other Binomial Trees
E 17 389-411 Binomial Trees, Actual Probabilities
18 412-422 Jensen's Inequality
19 423-434 Normal Distribution
20 435-446 LogNormal Distribution
The Table of Contents is continued on the next page.
1
All references are to the third edition.
2
Note that problems include both some written by me and some from past exams. The latter are copyright by the
Society of Actuaries and the Casualty Actuarial Society and are reproduced here solely to aid students in studying
for exams. The solutions and comments are solely the responsibility of the author; the SOA and CAS bear no
responsibility for their accuracy. While some of the comments may seem critical of certain questions, this is intended
solely to aid you in studying and in no way is intended as a criticism of the many volunteers who work extremely long
and hard to produce quality exams.
2014-MFE, Financial Economics, HCM 11/13/13, Page 1
Section # Pages Section Name
G 21 447-451 Limited Expected Value
22 452-504 A LogNormal Model of Stock Prices
H 23 505-552 Black-Scholes Formula
24 553-561 Black-Scholes, Options on Currency
25 562-566 Black-Scholes, Options on Futures Contracts
26 567-575 Black-Scholes, Stocks Paying Discrete Dividends
27 576-600 Using Historical Data to Estimate Parameters of the Stock Price Model
I 28 601-617 Implied Volatility
29 618-629 Histograms
30 630-648 Normal Probability Plots
31 649-691 Option Greeks
J 32 692-702 Delta-Gamma Approximation
33 703-719 Option Greeks in the Binomial Model
34 720-744 Profit on Options Prior to Expiration
35 745-758 Elasticity
K 36 759-763 Volatility of an Option
37 764-769 Risk Premium of an Option
38 770-778 Sharpe Ratio of an Option
39 779-780 Market Makers
40 781-825 Delta Hedging
L 41 826-842 Gamma Hedging
42 843-844 Relationship to Insurance
43 845 Exotic Options
44 846-870 Asian Options
45 871-909 Barrier Options
M 46 910-956 Compound Options
47 957-981 Gap Options
48 982-993 Valuing European Exchange Options
49 994-1004 Forward Start Options
N 50 1005-1016 Chooser Options
51 1017-1028 Options on the Best of Two Assets
52 1029-1042 Cash-or-Nothing Options
O 53 1043-1061 Asset-or-Nothing Options
54 1062-1068 Random Walks
55 1069-1090 Standard Brownian Motion
56 1091-1110 Arithmetic Brownian Motion
57 1111-1128 Geometric Brownian Motion
58 1129-1187 Geometric Brownian Motion Model of Stock Prices
P 59 1188-1237 Ito Processes
60 1238-1285 Ito's Lemma
The Table of Contents is continued on the next page.
2014-MFE, Financial Economics, HCM 11/13/13, Page 2
Section # Pages Section Name
S 61 1286-1314 Valuing a Claim on S^a
62 1315-1337 Black-Scholes Equation
63 1338-1347 Simulation
T 64 1348-1355 Simulating Normal and LogNormal Distributions
65 1356-1370 Simulating LogNormal Stock Prices
66 1371-1379 Valuing Asian Options via Simulation
67 1380-1407 Improving Efficiency of Simulation
68 1408-1444 Bonds and Interest Rates
69 1445-1447 The Rendelman-Bartter Model
U 70 1448-1482 The Vasicek Model
71 1483-1524 The Cox-Ingersoll-Ross Model
V 72 1525-1535 The Black Model
73 1536-1553 Interest Rate Caps
74 1554-1567 Binomial Trees of Interest Rates
W 75 1568-1620 The Black-Derman-Toy Model
76 1621-1663 Important Formulas and Ideas
My practice exams are sold separately.
2014-MFE, Financial Economics, HCM 11/13/13, Page 3
Throughout I make many references to Derivatives Markets by McDonald; these are to the third
edition. One does not need the textbook in order to use my study guide; the references are to help
those who are also using the textbook.
Chapter of Third Edition Derivatives Markets Sections of Study Guide
9 3-10
10
3
11, 12, 14, 15, 27
11.1 11.3
4
10, 13, 16-17, 54
12.1-12.5
5
23-26, 28, 31, 34-38
13
6
32-33, 39-42
14 43-51
18 19-22, 27, 29-30
19.1-19.5 63-67
20.1-20.6
7
55-61
21.121.3
8
62
23.1
9
52-53
24.124.2
10
27, 28, 58
25.1-25.5
11
68-75
Appendix B.1 1
Appendix C 18
I have included in my early sections, the 9 questions from the 2007 FM Sample Exam for
Derivatives Markets, based on earlier chapters of the textbook.
Unless otherwise stated chapter appendices are not included in the required readings from this text.
3
Excluding Options on Commodities on pages 315 and 316
4
Including Appendices 11.A and 11.B.
5
Including Appendix 12.A.
6
Including Appendix 13.B.
7
Sections 20.120.3 (up to but excluding Modeling Correlated Asset Prices on pages 612-613),
20.4 (excluding Multivariate It!s Lemma on pages 616-617),
20.520.6 (up to but excluding Valuing a Claim on S
a
Q
b
on pages 621-622).
8
Sections 21.121.2 (excluding What If the Underlying Asset Is Not an Investment Asset on pages 635637) and
21.3 (excluding The Backward Equation on pages 637638, and excluding the last two paragraphs of the section
on page 639).
9
But with only those definitions in Tables 23.1 and 23.2 that are relevant to Section 23.1.
10
Up to the second paragraph on page 721, but including footnote 4 on page 721 and the top panel in Figure 24.3
on page 723.
11
Sections 25.1 25.4 (up to the first paragraph on page 773), 25.5 (excluding LIBOR Market Model on pages
781-783), Appendix 25.A (this appendix contains only a reference to the following site for download,
http://wps.aw.com/wps/media/objects/14728/15081864/appendices/McDonald-web-25-A.pdf )
2014-MFE, Financial Economics, HCM 11/13/13, Page 4
Chapter of Second Edition Derivatives Markets Sections of Study Guide
9 3-10
10
12
11, 12, 14, 15.
11.1 11.4
13
10, 13, 16-17, 27, 54
12.1-12.5
14
23-26, 28, 31, 34-38
13
15
32-33, 39-42
14 43-51
18 19-22, 27, 29-30
19.1-19.5 63-67
20.1-20.7
16
55-61
21.121.3
17
62
22.1
18
52-53
23.123.2
19
27, 28, 58
24.1-24.5
20
68-75
Appendix B.1 1
Appendix C 18
I have included in my early sections, the 9 questions from the 2007 FM Sample Exam for
Derivatives Markets, based on earlier chapters of the textbook.
Unless otherwise stated chapter appendices are not included in the required readings from this text.
12
Excluding Options on Commodities on page 334.
13
Including Appendices 11.A and 11.B.
14
Including Appendix 12.A.
15
Including Appendix 13.B.
16
Sections 20.120.6 (up to but excluding Multivariate It!s Lemma on pages 665-666) and 20.7 (up to but
excluding Valuing a Claim on S
a
Q
b
on pages 670-672 and excluding Finding the lease rate on top one-half of
page 669).
17
Sections 21.121.2 (excluding What If the Underlying Asset Is Not and Investment Asset on pages 688-690)
and 21.3 (excluding The Backward Equation on pages 691-692, and excluding the paragraph on page 692 that
begins If a probability and through the end of the section).
18
But with only those definitions in Tables 22.1 and 22.2 that are relevant to Section 22.1.
19
Up to but excluding Exponentially Weighted Moving Average on page 746 and through the end of the section.
20
Up to but excluding Forward rate agreements on pages 806-808.
2014-MFE, Financial Economics, HCM 11/13/13, Page 5
This study guide covers all of the material on the Joint Exam: CAS 3F and SOA MFE.
21

The syllabus consists of various sections of the 3rd edition of Derivatives Markets by Robert L.
McDonald, plus a short study note Some Remarks On Derivatives Markets by Elias S. W. Shiu.
22

Unless stated otherwise in a question assume:
The market is frictionless. There are no taxes, transaction costs, bid/ask spreads, or restrictions
on short sales. All securities are perfectly divisible. Trading does not affect prices. Information
is available to all investors simultaneously. Every investor acts rationally
(i.e., there is no arbitrage.)
The risk-free rate is constant
The notation is the same as used in Derivatives Markets by Robert L. McDonald.
The MFE CBT exam will provide a formula document as well as a normal
distribution calculator that will be available during the test by clicking buttons on
the item screen. Details are available on the Prometric Web Site.
http://www.prometric.com/SOA/MFE3F_calculator.htm
Similar to other exam reference buttons, the normal distribution calculator button will be available
throughout the exam in the top right corner of every item screen. Click the button to call up the
calculator and calculate cumulative normal distribution and inverse cumulative normal distribution
values. Use these values to answer the question as needed.
When using the normal distribution calculator, values should be entered with five decimal places.
Use all five decimal places from the result in subsequent calculations.
The normal distribution calculator button replaces the Normal Table.
The previous rule on rounding no longer applies.
23

You can try the normal distribution calculator button at the Prometric Web Site.
You will benefit from using it at least part of the time when you are studying.
The formula sheet contains the same information about the Normal and LogNormal distributions as
was provided in the past.
21
In 2007 the CAS and SOA gave separate exams. Starting in 2008 they gave a joint exam.
Starting in 2014 the SOA adminsters MFE alone.
22
The study note is available on the SOA webpage.
23
Unfortunately, my solutions were written up using the prior rule: On Joint Exam 3F/MFE, when using the normal
distribution, choose the nearest z-value to find the probability, or if the probability is given, choose the nearest
z-value. No interpolation should be used. For example, if the given z-value is 0.759, and you need to find
Pr(Z < 0.759) from the normal distribution table, then chose the probability for z-value = 0.76:
Pr(Z < 0.76) = 0.7764.
This should not make a significant difference.
2014-MFE, Financial Economics, HCM 11/13/13, Page 6
Changes to the Syllabus from 2007:
24

Section 12.6 of Derivatives Markets, Perpetual American Options, is no longer on the syllabus.
25

The final portion of Section 20.6, Multivariate Ito!s Lemma, is no longer on the syllabus.
26

The final portion of Section 24.5, Forward Rate Agreements, is no longer on the syllabus.
Added, the first portion of Section 20.7, Valuing a Claim on S
a
, up to but excluding the
last subsection on Valuing a Claim on S
a
Q
b
.
Changes to the Syllabus for Spring 2009:
Chapter 10 of Derivatives Markets, exclude Options on Commodities on page 334.
Exclude Section 11.5 of Derivatives Markets, on Binomial Trees, Discrete Dividends.
Add Appendices 11.A and 11.B of Derivatives Markets.
Add Appendix 13.B of Derivatives Markets.
Chapter 20 of Derivatives Markets: exclude Finding the lease rate on top one-half of page 669.
Add parts of Chapter 21: Sections 21.121.2 (excluding What If the Underlying Asset Is Not and
Investment Asset on pages 688-690) and 21.3 (excluding The Backward Equation on pages
691-692, and excluding the paragraph on page 692 that begins If a probability and
through the end of the section).
Add parts of Chapter 22: Section 22.1 (but with only those definitions in Tables 22.1 and 22.2 that
are relevant to Section 22.1.)
Add parts of Chapter 23: Sections 23.123.2 (up to but excluding Exponentially Weighted
Moving Average on page 746 and through the end of the section.)
Add Appendix B.1.
Add Appendix C.
Changes to the Syllabus for Fall 2009:
27

Add Chapter 18 of Derivatives Markets, about the LogNormal Stock Price Model.
Add Chapter 19.119.5 of Derivatives Markets, about Monte Carlo Valuation, in other words
simulation.
Changes for 2011:
Computer based testing.
3 hours and approximately 30 questions.
24
Starting in 2008 there was a joint exam, 3F/MFE.
25
In 2007 Section 12.6 was on SOA MFE, but not CAS 3.
26
In 2007 this final portion of Section 20.6 was on CAS 3, but not SOA MFE.
27
Material was moved from Exam 4/C onto Exam 3F/MFE. Exam 3F/MFE was extended from 2 hours to 2.5 hours
and will consist of approximately 25 multiple-choice questions.
2014-MFE, Financial Economics, HCM 11/13/13, Page 7
Exam Questions by Section of This Study Guide:
MFE CAS 3 MFE CAS 3 MFE/3F
Section Section Name Sample 5/07 5/07 11/07 5/09
AA 1 Introduction 25
2 European Options
3 Properties of Premiums of Euro. Options 2
B 4 Put-Call Parity 1 3, 4 1, 4 14, 16 12
B 5 Bounds on Premiums of Euro. Options
6 Options on Currency 15 9
C 7 Exchange Options
8 Futures Contracts
9 Synthetic Positions 13
C 10 American Options 26 12 13
11 Replicating Portfolios 16 17 3
D 12 Risk Neutral Probabilities 27 14
13 Utility Theory and Risk Neutral Pricing
14 Bin. Trees, Risk Neutral Probs. 4, 49 15, 17 11 18, 19, 23 1
D
E 15 Binomial Trees, Options on Other Assets 5, 46
16 Other Binomial Trees 14
17 Binomial Trees, Actual Probabilities 2 7
18 Jensen's Inequality
F 19 Normal Distribution
E 20 LogNormal Distribution
21 Limited Expected Value
22 A LogNormal Model of Stock Prices 50
23 Black-Scholes Formula 6 20, 21 3, 8 20
G 24 Black-Scholes, Options on Currency 7 21
25 Black-Scholes, Options on Futures 55
H
26 Black-Scholes, Discrete Dividends 15 19
27 Historical Data to Estimate Parameters 17, 51
28 Implied Volatility
29 Histograms
30 Normal Probability Plots
The SOA did not release its 11/07 exam MFE.
The CAS/SOA did not release the 5/08, 11/08,11/09, and subsequent exams 3F/MFE.
Continued on the next page
2014-MFE, Financial Economics, HCM 11/13/13, Page 8
MFE CAS 3 MFE CAS 3 MFE/3F
Section Section Name Sample 5/07 5/07 11/07 5/09
AI 31 Option Greeks 8, 31 17
32 Delta-Gamma Approximation 19 20
33 Option Greeks in the Binomial Model 44, 45, 69
34 Profit on Options Prior to Expiration 40 13
B 35 Elasticity 20, 41 22
J
36 Volatility of an Option 5
37 Risk Premium of an Option
38 Sharpe Ratio of an Option 29
K 39 Market Makers
C 40 Delta Hedging 9, 47, 65 32
41 Gamma Hedging 33 10 24
42 Relationship to Insurance
43 Exotic Options
L 44 Asian Options 27
D 45 Barrier Options 42 34 2
46 Compound Options 28
47 Gap Options 18 17 26
M 48 Valuing European Exchange Options
49 Forward Start Options 19, 33
E
50 Chooser Options 25
N 51 Options on the Best of Two Assets 54 6
52 Cash-or-Nothing Options 28, 53
53 Asset-or-Nothing Options 4
54 Random Walks
O 55 Standard Brownian Motion 34
56 Arithmetic Brownian Motion
57 Geometric Brownian Motion
58 Geo. Brown. Mot. Model Stock Pr. 10, 11, 32, 37 16
56, 74
P
59 Ito Processes 12, 23, 48, 61 18 10, 18
63, 66, 67, 70
60 Ito's Lemma 13, 24, 35, 43 35 12 6
Q 64, 68, 73
Continued on the next page
2014-MFE, Financial Economics, HCM 11/13/13, Page 9
MFE CAS 3 MFE CAS 3 MFE/3F
Section Section Name Sample 5/07 5/07 11/07 5/09
AR 61 Valuing a Claim on S^a 16, 62, 71, 72 11
62 Black-Scholes Equation 36 8
63 Simulation
64 Simulating Normals & LogNormals
BS 65 Simulating LogNormal Stocks 52
66 Valuing Asian Options via Sim.
T 67 Improving Efficiency of Simulation 57, 58, 59, 75
68 Bonds and Interest Rates 39
69 The Rendelman-Bartter Model
C 70 The Vasicek Model 14, 22 36 13 15
71 The Cox-Ingersoll-Ross Model 21, 38, 60
U 72 The Black Model 7
73 Interest Rate Caps 3
74 Binomial Trees of Interest Rates 5
DV 75 The Black-Derman-Toy Model 15, 29, 30, 76 37 9 14
Questions no longer on the syllabus: MFE, 5/07, Q. 16.
In August 2010, the SOA/CAS updated the file of MFE Sample Exam questions.
There are now a total of 76 sample questions.
Check the SOA webpage to see if any additional Sample Exam questions have been
added.
Valuing a Claim on S
a
was added to the syllabus in Spring 2008.
Material on simulation was moved here from Exam 4/C in Fall 2009.
2014-MFE, Financial Economics, HCM 11/13/13, Page 10
Section 1, Introduction
Earlier Chapters of Derivatives Markets by McDonald are on Exam 2/FM.
28

Some of the ideas covered in those chapters are used in the chapters on your exam.
Derivatives:
29

A derivative is an agreement between two people that has a value determined by the price of
something else.
For example, Alan gives Bob the right to buy from Alan a share of IBM stock one year from now at
a price of $120. This is an example of a stock option. The value of this option depends on the price
of IBM stock one year from now.
Options:
A call is an option to buy. For example, Bob purchased a call option on IBM stock from Alan.
A put is an option to sell.
For example, if Debra purchased a put option on IBM stock from Carol, then Debra will have the
option in the future to sell a share of IBM stock to Carol at a specified price.
Continuously Compounded Risk Free Rate:
30

If r is the continuously compounded annual risk free rate, then the present value of $1 T years in the
future is: e
-rT
.
r as used by McDonald is what an actuary would call the force of interest.
Effective Annual Rate:
31

If r is the effective annual risk free rate, then the present value of $1 T years in the future is: 1/(1+r)
T
.
An effective annual rate is what an actuary would call the rate of interest.
Effective annual rate will be used in Interest Rate Caps and the Black-Derman-Toy Model, to be
discussed in subsequent sections. Otherwise, we will use continuously compounded rates.
28
3rd edition, Chapters 13, Chapter 4 (4.14.4), Chapter 5 (5.15.4 and Appendix 5.B),
Chapter 8 (8.18.3), Chapter 15 (Sections 15.3 and 15.4 only).
29
Warren E. Buffett has said, Derivatives are financial weapons of mass destruction, carrying dangers that, while
now latent, are potentially lethal.
30
See Appendix B.1 of Derivatives Markets by McDonald.
31
See Appendix B.1 of Derivatives Markets by McDonald.
2014-MFE, Financial Economics 1 Introduction, HCM 11/13/13, Page 11
Selling Short:
If we sell a stock short, then we borrow a share of stock and sell it for the current market price. We will
give this person a share of stock at the designated time in the future. We also must pay this person
any stock dividends they would have gotten on the stock, when they would have gotten them.
Forward Contracts:
A forward contract is an agreement that sets the terms today, but the buying or selling
of the asset takes place in the future.
For example, Ed will be moving in a month, and his friend Fred agrees to buy Ed!s TV one month
from now for $200.
The purchaser of an option has bought the right to do something in the future, but has no obligation
to do anything. In contrast, in a forward contract both parties are obligated to fulfill their parts of the
contract.
Value of a Forward Contract:
F
0,T
= forward price at time T in the future.
For example, if Joe buys a forward contract to buy one share of ABC stock in two years at $120,
then F
0,2
= $120. At time 2 years, Joe pays $120 and gets one share of stock.
32

PV[F
0,T
] is the present value at time 0 of a forward contract to be executed at time T.
PV[F
0,T
] = F
0,T
e
-rT
.
Let us assume the current price of XYZ stock is S
0
.
Assume XYZ stock pays no dividends.
Charlie can buy a forward contract to buy one share of XYZ stock in exchange for paying F
0,T
at
time T. If Charlie invests F
0,T
e
-rT
at the risk free rate, then at time T he will have F
0,T
.
33
He uses that amount to fulfill his forward contract and at time T Charlie has one share of XYZ Stock.
Lucy can instead buy one share of XYZ stock now, for the current market price of S
0
, and hold onto
the share of stock until at least time T.
32
This differs from the prepaid price. Joe might instead be able to pay $110 now and get a share of stock 2 years
from now. This is an example of a prepaid futures contract.
33
Charlie could invest in a Treasury Bond.
2014-MFE, Financial Economics 1 Introduction, HCM 11/13/13, Page 12
Both Lucy and Charlie end up in the same situation, with one share of XYZ stock at time T.
Therefore, their investments must have equal present value.
S
0
= F
0,T
e
-rT
.
F
0,T
= S
0
e
rT
, in the absence of dividends.
If instead XYZ stock pays dividends, then Lucy would have collected any dividends paid from time
0 to T, while she owned the stock. Charlie would not. Thus Lucy!s position is equal to Charlie!s
position plus a receipt of dividends.
Therefore, S
0
- PV[Div] = F
0,T
e
-rT
.
F
0 , T
= S
0
e
rT
- PV[Div] e
rT
.
If the dividends are paid at discrete points in time, with amount

D
t
i
paid at time t
i
, then
F
0 , T
= S
0
e
rT
-

e
r(T - t
i
)
D
t
i
" .
Exercise: The current price of a stock is $100.
It will to pay a dividend 3 months from now, a dividend 6 months from now, a dividend 9 months
from now, and a dividend 12 months from now. Each dividend is of size $2. r = 6%.
Determine the the forward price for a share of stock one year from now
[Solution: F
0,1
= (100) e
.06
- (2)(e
.045
+ e
.03
+ e
.015
+ e
0
) = $98.00.
Comment: Both sides of the equation are valued one year from now.]
Futures Contracts:
A futures contract is similar to a forward contract except:
A futures contract is typically traded on an exchange.
A futures contract is marked to market periodically.
34

The buyer and the seller post margin.
35

34
Marked-to-market means the item is revalued to reflect current market prices.
35
A deposit which compensates the other party to a futures contract in case one of the parties does not fulfill its
obligation.
2014-MFE, Financial Economics 1 Introduction, HCM 11/13/13, Page 13
Forward Contracts Versus Futures Contracts:
36

Forward Contract Futures Contract
Type of Market Dealer or Broker (Commodities) Exchange
Liquidity Low High
Contract Form Customized Standard
Performance Guarantee Creditworthiness Mark-to-Market
Transaction Costs Bid-ask spread Fees or Commissions
Continuous Dividends:
We often assume that dividends are paid at a continuous rate #.
37
Over a short period of time dt,
stock dividends of: # S(t) dt are paid, where S(t) is the stock price at time t.
So that if one buy a share of stock at time 0, and reinvests the dividends in the stock, at time T one
would have e
T#
shares of the stock.
38

Exercise: One buys 1 million shares of a stock that pays dividends at the continuous annual rate of
2%. The dividends are reinvested in that stock.
After 3 years how many shares of the stock does one own?
[Solution: (1 million)e
(3)(.02)
= 1,061,837 shares.]
If instead of discrete dividends XYZ stock pays continuous dividends, then Lucy would have e
T#

shares of the stock at time T. If Charlie!s future contract were for e
T#
shares of the stock, then his
position would be equal to Lucy!s. F
0,T
e
T#
= S
0
e
rT
.
Therefore, in the case of dividends paid continuously: F
0 , T
= S
0
e
T(r - #)
.
Prepaid Forward Price:
The forward price is the price we would pay in the future for a forward contract. In contrast, the
prepaid forward price,

F
0,T
P
, is the price we would pay today for a forward contract.

F
0,T
P
= F
0 , T
e
-rT
.
36
Taken from Table 2.2 Financial Economics, Harry H. Panjer, editor.
37
This is a good approximation for a stock index fund.
38
# acts similarly to a force of interest.
2014-MFE, Financial Economics 1 Introduction, HCM 11/13/13, Page 14
For example, let us assume we are paying today in order to own a share stock at time 3. Then the
prepaid forward price is

F
P
0, 3
(S).
We would pay this price at time 0 in exchange for receiving the stock at time 3.
However, we would not receive any dividends the stock would pay between time 0 and 3.
Therefore, in the case of discrete dividends,

F
0,T
P
(S) = S
0
- PV[Div].
Exercise: The current price of a stock is 120. The stock will pay a dividend of 3 in 2 months.
What is the 5 month prepaid forward price of the stock? r = 6%.
[Solution: S
0
- PV[Div] = 120 - 3e
-(2/12)(6%)
= 117.03.]
In the case of continuous dividends,

F
0,T
P
(S) = S
0
e
-#T
.
Exercise: The current price of a stock is 80. The stock pays dividends at a continuous rate of 1%.
What is the 5 month prepaid forward price of the stock?
[Solution: S
0
e
-#T
= 80e
-(5/12)(1%)
= 79.67.]
If we pay S
0
e
-#T
in order to buy e
-#T
shares of stock today and reinvest the dividends we would
have one share of stock at time T. Thus S
0
e
-#T
is the price we would pay today to own one share
of stock at time T. More generally,

F
t, T
P
(S) = S
t
e
-#(T-t)
.
Continuously Compounded Returns:
Let S
t
and S
t+h
be the stock prices at times t and t+h. Then the continuously compounded return on
the stock between time t and t+h is: ln[S
t+h
/ S
t
]. On an annual basis, this return is: ln[S
t+h
/ S
t
] / h.
For example, if the stock price is $80 at time 0 and $90 at time 2 years, then the continuously
compounded return from time 0 to 2 is: ln[90/80] = 11.78%. On an annual basis, this return is:
11.78% / 2 = 5.89%.
Exercise: The stock price is $90 at time 2 years and $85 at time 2.5 years, what is the annual
continuously compounded return?
[Solution: ln[85/90] / 0.5 = -11.4%.]
2014-MFE, Financial Economics 1 Introduction, HCM 11/13/13, Page 15
One can get the future stock price from the current stock price and the continuously compounded
return. For example, if the current stock price is $100, and the continuously compounded return over
the next three years is 8% per year, then the stock three years from now is: 100 e
0.24
= $127.12.
Exercise: The current price of a stock price is $60. Over the next four years the annual continuously
compounded return are: 17%, 33%, -140%, and 6%. What is the stock price in four years?
[Solution: 60 exp[0.17] exp[0.33] exp[-1.40] exp[0.06] = 60 exp[-0.84] = $25.90.
Comment: The continuously compounded returns add; the return over the whole four years is:
17% + 33% - 140% + 6% = -84%.
When the stock price declines by a very large amount, one can have a continuously compounded
return of less than -100%.]
Volatility:
The volatility of a stock is the standard deviation of its continuously compounded returns.
39

Actuarial Present Values:
40

Let us assume that one year from now an insurer will pay either $50 with probability 70% or $100
with probability 30%. Then the expected payment in one year is: (0.7)(50) + (0.3)(100) = $65.
Assume that the continuously compound annual rate of interest is now 5%.
Then the actuarial present value of the insurer!s payment is: 65 e
-0.05
= $61.83.
41

In general in order to calculate an actuarial present value, one takes a sum of the expected
payments at each point in time each multiplied by the appropriate discount factor. The discount
factor adjusts for the difference between the time value of money at the present and at the time
when the payment is made.
Exercise: In addition to the payments one year from now, the insurer will pay two years from now
either $50 with probability 50%, $100 with probability 40%, or $200 with probability 10%.
Assume that one year from now the continuously compound annual rate of interest will be 6%.
Determine the actuarial present value of the insurer!s total payments, including those made one year
from now and two years from now.
[Solution: The expected payment in two years is: (0.5)(50) + (0.4)(100) + (0.1)(200) = $85.
Discounting back to the present: 85 exp[-0.05 - 0.06] = $76.15.
Adding in the actuarial present value of the payments made in one year, the actuarial present value
of the insurer!s total payments is: $61.83 + $76.15 = $137.98.]
39
Volatility will be discussed in subsequent sections and is usually stated on an annual basis.
40
Covered extensively on CAS Exam LC and SOA Exam MLC.
41
If instead the 5% were an effective annual rate, then the actuarial present value would be: 65/1.05 = $61.90.
2014-MFE, Financial Economics 1 Introduction, HCM 11/13/13, Page 16
Named Positions:
42

One can buy various combinations of options and stock.
The more common such positions have been given names.
Bear Spread: The sale of an option together with the purchase of an otherwise identical
option with a higher strike price. Can construct a bear spread using either puts or calls.
The owner of the Bear Spread hopes that the stock price moves down.
Box Spread: Buy a call and sell a put at one strike price, plus at another (higher) strike
price sell a call and buy a put.
43

Bull Spread: The purchase of an option together with the sale of an otherwise identical
option with a higher strike price. Can construct a bull spread using either puts or calls.
The owner of the Bull Spread hopes that the stock price moves up.
Butterfly Spread: Buying a K strike option, selling two K + $K strike options,
and buying a K + 2$K strike option.
Collar: Purchase a put and sell a call with a higher strike price.
Ratio Spread: Buying m of an option and selling n of an otherwise identical option at a
different strike.
Straddle: Purchase a call and the otherwise identical put.
Strangle: The purchase of a put and a higher strike call with the same time until
expiration.
For example, Gene Green buys a Straddle with K = 80.
He buys an 80-strike call and a similar 80-strike put.
His payoff at expiration is: Max[0, S
T
- 80]
+
+ Max[0, 80 - S
T
] = |S
T
- 80|.
The further the stock price at expiration is from 80, the larger Gene!s payoff.
Gene is hoping there is a large movement in the stock price.
44

42
See Chapter 3 of Derivatives Markets by McDonald, on the syllabus of Exam FM.
43
For European options, the box spread is equivalent ot a zero-coupon bond.
44
In other words, Gene is betting that the stock!s volatility is high.
In contrast, the seller of a straddle is betting that the stock!s volatility is low.
2014-MFE, Financial Economics 1 Introduction, HCM 11/13/13, Page 17
For example, Vanessa buys a 90 strike call and sells an otherwise identical 100 strike call.
This is an example of a Call Bull Spread. Vanessa hopes the stock price increases.

If Vanessa bought her 90 strike call from Nathan and sold her 100 strike call to Nathan, than Nathan
owns a Call Bear Spread. Nathan hopes the stock price declines.

Long and Short Positions:
Entering into a long position is buying.
Entering into a short position is selling or writing.
For example if you long one call option and long the similar put option, then you bought the call and
put, and you have purchased a straddle. If instead you short a call option and the similar put option,
then you have written (sold) a straddle.
If you short a 60-strike 3-month call and long a 80-strike 3-month call, then you have purchased a
Bear Spread.
2014-MFE, Financial Economics 1 Introduction, HCM 11/13/13, Page 18
Problems:
1.1 (1 point) A stock price is 160. Assume r = 0.08 and there are no dividends.
What is the 4-year forward price?
A. less than 200
B. at least 200 but less than 210
C. at least 210 but less than 220
D. at least 220 but less than 230
E. at least 230
1.2 (1 point) A stock has a current price of 120.
The stock pays dividends at a continuously compounded rate of 1.5%. r = 0.08.
What is the 4-year prepaid forward price?
A. 113 B. 115 C. 117 D. 119 E. 121
1.3 (1 point) A stock has a two-year forward price of 99.66.
The stock pays dividends at a continuously compounded rate of 3%. r = 7%.
What is the current price of this stock?
A. 90 B. 92 C. 94 D. 96 E. 98
1.4 (1 point) A stock has a current price of 90.
The stock pays dividends at a continuously compounded rate of 2%. r = 0.06.
What is the 5-year forward price?
A. 100 B. 105 C. 110 D. 115 E. 120
1.5 (1 point) A stock has a current price of $100.
In 3 months the stock will pay a dividend of $2. r = 0.04.
What is the 4-month prepaid forward price?
1.6 (1 point) A stock has a four-year forward price of 89.43.
The stock pays dividends at a continuously compounded rate of 0.8%. r = 5.2%.
What is the current price of this stock?
A. 65 B. 70 C. 75 D. 80 E. 85
1.7 (1 point) Options are extremely risky investments.
The variance of returns is great, yet most people are assumed to be risk-averse.
Moreover, brokerage commissions on options are high.
So why are options and other derivative securities such popular financial instruments?
2014-MFE, Financial Economics 1 Introduction, HCM 11/13/13, Page 19
1.8 (CAS5B, 11/93, Q. 31) (2 points).
a. (1.5 points) List and briefly describe three examples of derivative instruments.
b. (0.5 points) Why do firms use them?
1.9 (CAS5B, 11/91, Q. 60) (1 point)
Which of the following statements concerning corporate securities is FALSE?
A. One reason firms utilize derivative instruments is to protect themselves against the effects of
adverse changes in various external factors.
B. Firms do not issue derivative securities to raise money.
C. A futures contract is an order than you place in advance to buy or sell an asset or commodity.
D. In a futures contract, the price is fixed when you place the order and paid at the time of the order.
E. A forward contract is a tailor-made product that is not traded on an organized exchange.
1.10 (CAS5B, 5/94, Q. 11) (1 point) Which of the following are true?
1. A future is an order that you place in advance to buy or sell an asset or commodity at a price that
is agreed upon when the order is placed.
2. A forward contract is traded on an organized exchange.
3. Swaps are agreements that grant bond owners the right to exchange the bond for a
predetermined number of common shares by the exercise date.
A. 1 B. 2 C. 1, 2 D. 2, 3 E 1, 2, 3
1.11 (CAS5B, 11/95, Q. 31) (2 points)
a. (1/2 point) Briefly describe warrants and convertible bonds.
b. (3/4 points) For each, describe what the rational holder probably will do on the expiration date if
the price of stock rises significantly from the date of issuance.
c. (3/4 points) For each, describe what the holder probably will do if the price of the company's
stock falls significantly.
1.12 (CAS5B, 11/98, Q.10) (1 point) Which of the following are true regarding financial
derivatives?
1. Firms typically issue derivatives to raise money on short notice.
2. A forward contract may be traded on an organized exchange.
3. A warrant is a derivative.
A. 1 B. 3 C. 1, 3 D. 2, 3 E. 1, 2, 3
2014-MFE, Financial Economics 1 Introduction, HCM 11/13/13, Page 20
1.13 (FM Sample Exam, Q.4) Zero-coupon risk-free bonds are available with the following
maturities and yield rates (effective, annual):
Maturity (years) Yield
1 0.06
2 0.065
3 0.07
You need to buy corn for producing ethanol. You want to purchase 10,000 bushels one year from
now, 15,000 bushels two years from now, and 20,000 bushels three years from now. The current
forward prices, per bushel, are 3.89, 4.11, and 4.16 for one, two, and three years respectively.
You want to enter into a commodity swap to lock in these prices.
Which of the following sequences of payments at times one, two, and three will NOT be acceptable
to you and to the corn supplier?
A. 38,900, 61,650, 83,200
B. 39,083, 61,650, 82,039
C. 40,777, 61,166, 81,554
D. 41,892, 62,340, 78,997
E. 60,184, 60,184, 60,184
1.14 (FM Sample Exam, Q.6) The current price of one share of XYZ stock is 100. The forward
price for delivery of one share of XYZ stock in one year is 105. Which of the following statements
about the expected price of one share of XYZ stock in one year is TRUE?
A. It will be less than 100
B. It will be equal to 100
C. It will be strictly between 100 and 105
D. It will be equal to 105
E. It will be greater than 105.
1.15 (FM Sample Exam, Q.7) A non-dividend paying stock currently sells for 100.
One year from now the stock sells for 110.
The risk-free rate, compounded continuously, is 6%.
The stock is purchased in the following manner:
You pay 100 today
You take possession of the security in one year
Which of the following describes this arrangement?
A. Outright purchase
B. Fully leveraged purchase
C. Prepaid forward contract
D. Forward contract
E. This arrangement is not possible due to arbitrage opportunities
2014-MFE, Financial Economics 1 Introduction, HCM 11/13/13, Page 21
1.16 (FM Sample Exam, Q.8) You believe that the volatility of a stock is higher than indicated by
market prices for options on that stock. You want to speculate on that belief by buying or selling
at-the-money options. What should you do?
A. Buy a strangle
B. Buy a straddle
C. Sell a straddle
D. Buy a butterfly spread
E. Sell a butterfly spread
1.17 (CAS3, 11/07, Q.25) (2.5 points) On January 1, 2007, the Florida Property Company
purchases a one-year property insurance policy with a deductible of $50,000. In the event of a
hurricane, the insurance company will pay the Florida Property Company for losses in excess of the
deductible. Payment occurs on December 31, 2007. For the last three months of 2007, there is a
20% chance that a single hurricane occurs and an 80% chance that no hurricane occurs. If a hurricane
occurs, then the Florida Property Company will experience $1,000,000 in losses. The continuously
compounded risk-free rate is 5%. On October 1, 2007, what is the risk-neutral expected value of
the insurance policy to the Florida Property Company?
A. Less than $185,000
B. At least $185,000, but less than $190,000
C. At least $190,000, but less than $195,000
D. At least $195,000, but less than $200,000
E. At least $200,000
1.18 (IOA, CT8, 9/08, Q.6) (6 points) Consider an asset S paying a dividend at a constant
instantaneous rate of #, a forward contract with maturity T written on S and a constant, instantaneous
(continuously compounded) risk-free rate of r.
Derive the price at time t of the forward contract, using the no-arbitrage principle.
2014-MFE, Financial Economics 1 Introduction, HCM 11/13/13, Page 22
1.19 (CAS8, 5/09, Q.21) (2.25 points) Given the following information about a box spread
related to 1,000 shares of Company XYZ stock:
The current price of Company XYZ's stock is $100.
The strike prices of the European call options underlying the box spread are $110 and $120.
The time to maturity of the box spread is 1 year.
The continuously compounded risk-free rate is 5% per annum.
Investor A is willing to purchase the box spread from you for $9,750.
Investor B is willing to sell the box spread to you for $9,750.
Assume there are no taxes or transaction costs and you can borrow or lend at the risk-free rate.
a. (1.5 point) Explain whether you purchase or sell the box spread.
Calculate the profit you earn. Show all work.
b. (0.75 point) Assume the options underlying the box spread are American instead of
European. The investor with whom you entered into the box spread transaction in part a.
above believes the price of Company XYZ will not decrease.
Explain the investor's expected actions immediately after entering the box spread transaction
with you.
1.20 (CAS8, 5/10, Q.12) (2.5 points) An investor would like to enter into a forward contract
whereby in two years the investor exchanges a fixed amount of US Dollars for one million Euros.
Assume the current exchange rate is $1.50 per Euro and that the continuously compounded risk-free
interest rates are 2% in Europe and 1% in the United States.
The investor can borrow and invest at the risk-free rate.
a. (1 point) Determine an investment strategy which would give the investor the same cash flows as
the forward contract.
b. (1.5 points) Assume the two-year forward price for one million Euros is now $1,475,000 and that
the investor can either take a long or short position on a forward contract at this price.
Determine an investment strategy which would ensure the investor an arbitrage profit.
Calculate the present value of this profit.
2014-MFE, Financial Economics 1 Introduction, HCM 11/13/13, Page 23
Solutions to Problems:
1.1. D. (160)exp[(.08)(4)] = $220.34.
1.2. A.

F
0,T
P
(S) = S
0
e
-#T
= (120) exp[-(1.5%)(4)] = 113.01.
Comment: This prepaid forward price, is what we pay today for delivery of the stock 4 years from
now. We do not receive any of the dividend payments during those 4 years.
1.3. B. The prepaid forward price is the forward price discounted for interest: 99.66 / e
0.14
= 86.64.
(current price) e
-#T
= (current price) e
-0.06
= prepaid forward price = 86.64.
% current price = 86.64 e
0.06
= 92.00.
Alternately, forward price = S
0
exp[(r-#)T]. % 99.66 = S
0
exp[(7% - 3%)(2)]. % S
0
= 92.00.
1.4. C. F
0,T
= S
0
e
T(r - #)
= (90)exp[(.06 - .02)(5)] = 109.93.
1.5.

F
0,T
P
= S
0
- PV[Div] = 100 - 2exp[-.04/4] = $98.02.
Comment: This prepaid forward price, is what we pay today for delivery of the stock 4 months from
now. We do not receive the dividend payment 3 months from now.
1.6. C. Forward price = S
0
exp[(r-#)T]. % 89.43 = S
0
exp[(5.2% - 0.8%)(4)]. % S
0
= 75.00.
Alternately, the prepaid forward price is the forward price discounted for interest:
89.43 / e
0.208
= 72.64.
(current price) e
-#T
= (current price) e
-0.032
= prepaid forward price = 72.64.
% current price = 72.64 e
0.032
= 75.00.
1.7. Options and other derivative securities allow investors to shape their exposure to risks in
ways that would otherwise be costly or impossible to attain. Depending on their individual tastes,
risk preferences, and circumstances, some investors will use derivatives to increase their exposure
to risk while others will decrease their exposure through hedging.
2014-MFE, Financial Economics 1 Introduction, HCM 11/13/13, Page 24
1.8. a. 1. Futures and forward contracts are obligations to buy or sell commodities or
securities in the future at prices determined now. Futures are standardized contracts traded on
exchanges. Forward contracts are individualized tailor-made contracts that are not traded on
exchanges.
2. Options give the purchaser the right to buy (call option) or sell (put option) securities at fixed
prices in the future.
3. Swaps are obligations to exchange commodities or values at a number of points in the future.
For instance, currency swaps exchange interest payments in different currencies.
Interest rate swaps may exchange a fixed coupon payment for a variable coupon payment.
b. Firms use these instruments to hedge uncertainties in securities prices (e.g., by call and put
options), commodities prices (e.g., by futures contracts), in currency rates (e.g., by currency swaps),
and in interest rates (e.g., by interest rate swaps).
1.9. D. In the case of futures contract, we pay upon delivery; we do not pay cash in advance.
1.10. A. Statement #1 is true.
A futures contract is traded on an organized exchange. Thus Statement #2 is false.
Statement #3 describes convertible bonds, and thus is false. Swaps are of various types,
generally involving the exchange of interest received on fixed-income securities, such as
exchanging bond interest paid in U.S. dollars for interest in Euros, or exchanging interest paid at a
fixed rate for interest paid at a variable rate.
1.11. a. A warrant is a security that entitles the holder to buy the underlying stock of the issuing
company at a fixed strike price until the expiry date. A warrant is similar to an American call option,
but it is issued by a firm on its own stock; if the warrant is exercised, new shares of stock are issued.
Convertible bond is like conventional debt, but it gives the holder the right to exchange the bond for
a fixed number of newly issued shares in the firm.
b. If the stock price on the expiration date is significantly higher than the price on the issuance date,
then the warrant holder will exercise and the convertible bond holder will convert.
c. If stock the price on the expiration date is significantly lower than the price on the issuance date, the
warrant holder will not exercise and the convertible bond holder will not convert.
1.12. B. A warrant is an option issued by a firm with its own stock as the underlying asset.
A futures contract is traded on an organized exchange.
2014-MFE, Financial Economics 1 Introduction, HCM 11/13/13, Page 25
1.13. B. The present value of the forward prices is:
10,000(3.89)/1.06 + 15,000(4.11)/1.065
2
+ 20,000(4.16)/1.07
3
= 158,968.
Any sequence of payments with that present value is acceptable.
A: 38,900/1.06 + 61,650/1.065
2
+ 83,200/1.07
3
= 158,968.
B: 39,083/1.06 + 61,650/1.065
2
+ 82,039/1.07
3
= 158,193.
C: 40,777/1.06 + 61,166/1.065
2
+ 81,554/1.07
3
= 158,969.
D: 41,892/1.06 + 62,340/1.065
2
+ 78,997/1.07
3
= 158,968.
E: 60,184/1.06 + 60,184/1.065
2
+ 60,184/1.07
3
= 158,967.
All but B have the desired present value.
1.14. E. The forward price is a biased downwards predictor of the future price.
Therefore, the expected future price of XYZ stock is greater than its forward price of 105.
Comment: Someone who bought the stock should be compensated for time and risk.
The one year forward price is S
0
e
r
> S
0
, where r is the continuously compounded annual risk free
rate. If this were the expected future stock price, then someone who bought the stock would only be
compensated for time. In order to also be compensated for risk, the expected future stock price
must be greater than the forward price.
1.15. C. All four of answers A-D are methods of acquiring the stock.
Of these, the prepaid forward has the payment at time 0 and the delivery at time T.
Comment: See Table 5.1 in Derivatives Markets by McDonald.
Since there are no dividends, the prepaid forward price is equal to the current price of 100.
In a fully leveraged purchase, you get the stock today and pay 100e
.06
one year from now. In a
forward contract, you get the stock one year from now and pay the forward price, which since there
are no dividends is 100e
.06
. In an outright purchase you would pay 100 today and also get the
stock today.
2014-MFE, Financial Economics 1 Introduction, HCM 11/13/13, Page 26
1.16. B. Only straddles can consist of at-the-money options.
Higher volatility means that it is more likely that the future stock price will be either very high or very
low. If you buy an at-the-money put and call, then you will benefit on the call if the future stock is high
and benefit from the put if the future stock price is low. The higher the future stock price the more you
benefit from the call and the lower the future stock price the more you would benefit from the put.
Therefore, buying a straddle is correct for this speculation.
Comment: One could speculate on the belief that the volatility of a stock is lower than indicated by
market prices for options on that stock by writing (selling) an at-the-money straddle.
In a straddle, you would buy a put and a call, both with the same strike price and time until expiration.
In a strangle, you would buy a put and a higher strike call, both with the same time until expiration. In
a butterfly spread, you buy a call, sell two calls at a higher strike, and buy a fourth call at a still higher
strike; the difference between strikes is the same and all of the calls have the same date of
expiration. A butterfly spread may also be put together with puts.
1.17. B. We subtract the deductible of $50,000; 1,000,000 - 50,000 = 950,000.
(20%)($950,000)/e
.05/4
= $187,640.
Comment: This question has nothing to do with derivatives. You are merely being asked to take an
actuarial present value using a continuously compound risk-free rate (a force of interest.)
1.18. Consider two portfolios.
Portfolio A: take a long position in the forward contract at time t.
Its value at time t is 0. At time T its value is: S
T
- F
t,T
[S
T
].
Portfolio B: at time t buying a fraction exp[-#(T - t)] of the underlying asset,
and borrowing F
t,T
[S
T
] exp[-r(T - t)].
Its value at time t is: exp[-#(T - t)] S
t
- F
t,T
[S
T
] exp[-r(T - t)].
At time T its value is: S
T
- F
t,T
[S
T
].
Both portfolios have the same value at time T.
Thus, using the absence of arbitrage opportunity, both portfolios should have the same
value at any intermediate time, in particular at time t. Hence:
0 = exp[-#(T - t)] S
t
- F
t,T
[S
T
] exp[-r(T - t)]. %
F
t,T
[S
T
] = exp[(r-#)(T - t)] S
t
.
Comment: The forward price is exp[r(T - t)] times the prepaid forward price of:

F
t,T
P
[S
T
] = exp[-#(T - t)] S
t
.
2014-MFE, Financial Economics 1 Introduction, HCM 11/13/13, Page 27
1.19. (a) Box Spread: Buy a call and sell a put at one strike price, plus at another (higher) strike
price sell a call and buy a put. For European options, the box spread is equivalent to a zero-coupon
bond.
Here the buyer of the box spread, would buy a $110 strike call, sell a $110 strike put, sell a $120
strike call, and buy a 120 strike put.
The payoff on this box spread is: (S
T
- 110)
+
- (110 - S
T
)
+
- (S
T
- 120)
+
+ (120 - S
T
)
+
.
If S
T
< 110, then the payoff is: 0 - (110 - S
T
) + (120 - S
T
) = 10.
If 120 > S
T
> 110, then the payoff is: (S
T
- 110) + (120 - S
T
) = 10.
If S
T
> 120, then the payoff is: (S
T
- 110) - (S
T
- 120) = 10.
The payoff is always 10; there is no stock price risk.
Thus the appropriate premium for one box spread is: 10 exp[-0.05] = 9.512.
Therefore, for 1000 box spreads, the appropriate premium is $9512.
At $9,750, the box spreads are overpriced.
I would sell the box spreads to investor A,
earning a profit now of: 9,750 - 9512 = $238.
Alternately, I sell the box spreads to investor A and invest the money at the risk free rate of 5%
for one year, and have: (9750)(e
0.05
) = 10,250. I then pay investor A (1000)(10) = $10,000.
I make a profit of 10,250 - $10,000 = $250 in one year.
(b) Investor A owns a $110 strike put as part of the box spread.
Since Investor A believes that the price of the stock will not decrease, he expects this put to have a
payoff of $110 - $100 = $10 or less.
Therefore, Investor A should exercise this $110 strike American put right away.
When he does so, I would have to pay him: (1000)($110) = $110,000, for shares of stock that are
only worth: (1000)($100) = $100,000.
2014-MFE, Financial Economics 1 Introduction, HCM 11/13/13, Page 28
1.20. a. The investor will get 1,000,000 Euros two years from now if he now buys
Euro-denominated bonds in an amount of: 1 million / exp[(2)(2%)] = 960,789.
In order to buy these bonds requires: ($1.50)(960,789) = $1,441,184.
Thus the investor borrows $1,441,184 at the U.S. risk-free rate,
and lends 960,789 Euros at the Euro risk-free rate.
At the end of 2 years, the investor receives 1,000,000 Euros and must pay back:
exp[(2)(1%)] $1,441,184 = $1,470,961.
b. In part a we saw that the investor can borrow $1,441,184 to set up a position that gives the
same cashflows as the forward contract; this involves paying back $1,470,298 two years from now.
Thus the forward price of $1,475,000 is too high.
The investor can sell the forward contract (take a long position), lend 960,789 Euros, and borrow
$1,441,184. (He has bought a synthetic forward contract as well as sold an actual forward contract.)
In two years the investor has 1,000,000 Euros, which he delivers to satisfy the forward contract.
He receives $1,475,000 from fulfilling the forward contract.
After paying back the $1,470,298 he has a net of: $1,475,000 - $1,470,298 = $4702.
The present value of this arbitrage profit is: $4702 / exp[(2)(1%)] = $4609.
2014-MFE, Financial Economics 1 Introduction, HCM 11/13/13, Page 29
Section 2, European Options
There are various types of options. The simplest and the most important type for this exam are
European Options. The adjective European does not refer to where the option is bought.
Call Options:
ABC Stock is currently selling for $100. Dick buys from Jane an option to buy one year from today
a share of ABC Stock for $150.
45
If one year from now ABC Stock has a market price of more than
$150 dollars, then Dick should use this option to buy a share of ABC Stock from Jane at $150. Dick
could then sell this share of ABC Stock for the market price and make a profit.
This is an example of a European Call Option.
A European Call Option gives the buyer the right to buy one share of a certain stock at a
strike price (exercise price) upon expiration. A European option may only be exercised
on one specific day.
46
A call is an option to buy.
Dick has purchased a 1 year European Call Option on ABC Stock, with a strike price of $150.
Payoff on a Call Option:
The eventual value to Dick of this option, depends on the price of ABC Stock one year from now.
For example, if ABC!s market price turned out to be $180 per share one year from today, then Dick
could buy a share of ABC from Jane for the $150 strike price, and turn around and sell that share for
$180. Dick would make a profit of $30, ignoring what he originally paid Jane to buy the option.
47

If the future price of ABC is $150 or less, then Dick would not exercise his option.
48
In that case, his
option turns out to have no value to Dick.
Future Price of ABC Payoff on the Option to Dick
$120 0
$140 0
$160 $10
$180 $30
45
While for simplicity I have used in the example one share, one could buy an option for 100 shares or 1000 shares.
46
There are other exercise styles. See page 32 of Derivatives Markets by McDonald.
Others will be discussed subsequently.
47
And ignoring any transaction costs.
48
Dick has not agreed to buy a share from Jane. Dick does not have an obligation to buy, rather Dick has purchased
the right to buy a share if he wishes to.
2014-MFE, Financial Economics 2 European Options, HCM 11/13/13, Page 30
When the stock price is greater than the strike price, exercising the option makes money; this call
option is in the money. If the stock price and strike price are equal, then the option is at the
money. If the stock price is less than the strike price, then the call option is out of the money.
Let Y
+
= Max[0, Y] =

Y if Y & 0
0 if Y < 0
'
(
)
.
49

Then the eventual payoff on Dick!s call option is (S
1
- 150)
+
, where S
1
is the price of ABC Stock
one year from now. Here is a graph of the future value of Dick!s call option:
50 100 150 200 250 300
Stock Price
20
40
60
80
100
120
140
Call Payoff
In general, the future value of a European call option is: (S
T
- K)
+,
where S
T
is the price of the
stock on the expiration date of the call and K is the strike price of the call.
Put Options:
XYZ Stock is currently selling for $200. Mary buys from Rob an option to sell one year from today
a share of XYZ Stock for $250. If one year from now XYZ Stock has a market price of less than
$250, then Mary should buy a share of XYZ Stock at the market price and then use her option to
sell a share of XYZ Stock to Rob for $250, making a profit.
This is an example of a European Put Option.
A European Put Option gives the buyer the right to sell one share of a certain stock at a
strike price (exercise price) upon expiration. A put is an option to sell.
Mary has purchased a 1 year European Put Option on XYZ Stock, with a strike price of $250.
49
This very useful actuarial notation is not on the syllabus of this exam.
2014-MFE, Financial Economics 2 European Options, HCM 11/13/13, Page 31
Payoff on a Put Option:
The eventual payoff to Mary of her option, depends on the price of XYZ Stock one year from now.
For example, if XYZ market price turned out to be $220 per share one year from today, then Mary
could buy a share of XYZ for $220 and turn around and sell that share for $250 to Rob. Mary would
make a profit of $30, ignoring what she originally paid Rob to buy the option.
50

If the future price of XYZ Stock is $250 or more, then Mary would not exercise her option.
51
In this
case, her option turns out to have no value to Mary.
Future Price of XYZ Payoff on the Option to Mary
$220 $30
$240 $10
$260 0
$280 0
The eventual value of Mary!s put option is (250 - S
1
)
+
, where S
1
is the price of XYZ Stock one
year from now.
52
Here is a graph of the future value of Mary!s put option:
100 200 300 400 500
Stock Price
50
100
150
200
250
Put Payoff
In general, the future value of a European put option is: (K - S
T
)
+,
where S
T
is the price of the
stock on the expiration date and K is the strike price.
50
And ignoring any transaction costs.
51
Mary has not agreed to sell a share to Rob. Mary does not have an obligation to sell, rather Mary has purchased the
right to sell a share if she wishes to.
52
Y
+
is 0 if Y < 0, and Y is Y " 0.
2014-MFE, Financial Economics 2 European Options, HCM 11/13/13, Page 32
When the stock price is less than the strike price, exercising the put makes money; this put option is
in the money. If the stock price and strike price are equal, then the option is at the money.
If the stock price is greater than the strike price, then the put option is out of the money.
Expected Future Value of an Option:
The future payoff on a European call option is: (S
T
- K)
+,
where S
T
is the price of the stock on the
expiration date and K is the strike price. The future payoff on a European put option is: (K - S
T
)
+
.
Of course, at the time one could purchase an option, one does not know the future price of the stock.
The future price of the stock is a random variable. The expected value of the option can be obtained
by averaging using the distribution of future stock prices.
The expected future value of a European call option is: E[(S
T
- K)
+
].
The expected future value of a European put option is: E[(K - S
T
)
+
].
If one knew the distribution of S
T
, then
E[(S
T
- K)
+
] = E[S
T
- K | S
T
> K] Prob[S
T
> K] + (0)Prob[S
T
# K]
= (E[S
T
| S
T
> K] - K) Prob[ S
T
> K].
Similarly,
E[(K - S
T
)
+
] = (0)Prob[S
T
> K] + E[K - S
T
| S
T
# K] Prob[ S
T
# K]
= (K- E[S
T
| S
t
# K]) Prob[ S
T
# K].
Limited Expected Values:
53
Let X * K = Min[X, K].
Then the limited expected value is: E[X * K].
E[(S
T
- K)
+
] = E[S
T
] - E[S
T
* K].
E[(K - S
T
)
+
] = K - E[S
T
* K].
This manner of writing the expected future value can be useful if for example the distribution of future
prices is LogNormal and if one had a formula for the limited expected value of a LogNormal
Distribution.
54
53
See Mahler!s Guide to Loss Distributions or Loss Models, covering material on the syllabus of Exam C.
54
If the distribution of S
t
were LogNormal, this would lead to the Black-Scholes formula for valuing a put option.
2014-MFE, Financial Economics 2 European Options, HCM 11/13/13, Page 33
Problems:
Use the following information for the next 3 questions:
The price of the stock of the Daily Planet Media Company 1 year from now has the following
distribution:
Price Probability
60 20%
80 30%
100 30%
120 20%
2.1 (1 point) Determine the expected stock price of Daily Planet Media Company one year from
now.
2.2 (1 point) Determine the expected payoff of a 1 year European call option on one share of Daily
Planet Media Company, with a strike price of 85.
(A) Less than 7
(B) At least 7, but less than 9
(C) At least 9, but less than 11
(D) At least 11, but less than 13
(E) At least 13
2.3 (1 point) Determine the expected payoff of a 1 year European put option on one share of Daily
Planet Media Company, with a strike price of 85.
A) Less than 7
(B) At least 7, but less than 9
(C) At least 9, but less than 11
(D) At least 11, but less than 13
(E) At least 13
2.4 (2 points) Graph the future value of a European call option with a strike price of 100, as a function
of the future stock price.
2.5 (2 points) Graph the future value of a European put option with a strike price of 100, as a
function of the future stock price.
2.6 (2 points) Graph the payoff on a European call option with a strike price of 100 plus the
corresponding put, as a function of the future stock price. This position is called a straddle.
2014-MFE, Financial Economics 2 European Options, HCM 11/13/13, Page 34
2.7 (3 points) Christopher buys a $60 strike European call, sells two $70 strike European calls,
and buys an $80 strike European call.
The options are on the same stock and have the same expiration date.
This position is called a Butterfly Spread.
Graph the payoff on this portfolio as a function of the future price of the stock.
2.8 (2 points) Jason buys a $100 strike European put, and sells a $120 strike European put.
The puts are on the same stock and have the same expiration date.
This position is called a Put Bull Spread.
Graph the payoff on this portfolio as a function of the future price of the stock.
2.9 (3 points) Melissa buys a $90 strike European call, sells a $90 strike European put,
sells a $130 strike European call, and buys a $130 strike European put.
The options are on the same stock and have the same expiration date.
This position is called a Box Spread.
Graph the payoff on this portfolio as a function of the future price of the stock.
2.10 (3 points) Amanda buys two $100 strike European call, sells three $110 strike European calls,
and buys a $130 strike European call.
The options are on the same stock and have the same expiration date.
This position is called a Asymmetric Butterfly Spread.
Graph the payoff on this portfolio as a function of the future price of the stock.
2.11 (3 points) Robert buys 1000 calls on a stock with a strike price of $120.
The premium per call is $8. Robert also pays a total commission of $100.
Determine the stock price at expiration at which Robert will break even.
Graph Robert!s profit as a percent of his initial investment, as a function of the stock price at
expiration of the call. (Ignore the time value of money.)
2.12 (2 points) Tiffany buys a $90 strike European call and sells a $90 strike European put.
The options are on the same stock and have the same expiration date.
Graph the payoff on this portfolio as a function of the future price of the stock.
2.13 (3 points) Heather buys a $70 strike European put and sells a $90 strike European call.
The options are on the same stock and have the same expiration date.
This position is called a Collar.
Graph the payoff on this portfolio as a function of the future price of the stock.
2014-MFE, Financial Economics 2 European Options, HCM 11/13/13, Page 35
2.14 (1 point) ABC stock costs $78.
ABC stock does not pay dividends.
Harold enters into a long position on a $80-strike two-year European call on ABC,
and enters into a short position on a $80-strike two-year European put on ABC.
Harold pays a net of $6.33 for entering these positions.
What is the continuously compounded risk free rate?
A. 5.0% B. 5.5% C. 6.0% D. 6.5% E. 7.0%
2.15 (3 points) Allen buys a $70 strike European put, sells four $100 strike European puts,
and buys three $110 strike European puts.
The options are on the same stock and have the same expiration date.
This position is called a Asymmetric Butterfly Spread.
Graph the payoff on this portfolio as a function of the future price of the stock.
2.16 (3 points) Kimberly buys 100 puts on a stock with a strike price of $80.
The premium per put is $5. Kimberly also pays a total commission of $60.
Determine the stock price at expiration at which Kimberly will break even.
Graph Kimberly!s profit as a percent of her initial investment, as a function of the stock price at
expiration of the put. (Ignore the time value of money.)
2.17 (2 points) Nicholas buys a share of stock, sells a $110 strike European call on that stock, and
buys a $110 strike European put on that stock. The options have the same expiration date.
Graph the value of this portfolio when the options expire as a function of the future price of the stock.
2.18 (2 points) Let S(t) be the price of a stock at time t.
The stock pays dividends at the continuously compounded rate #.
The continuously compounded risk free rate is r.
Assume a contract is purchased at time 0 and pays at time T: Max[S(T), 100 ].
Determine the premium for this contract in terms of the premium of a European option and other
known quantities.
2.19 (2 points) Kevin writes (sells) a 60 strike call and a 60 strike put. The options have the same
expiration date. Graph the value of this portfolio when the options expire as a function of the future
price of the stock. This position is called a written straddle.
2.20 (2 points) Aaron owns a share of stock of the Charming Prints Company.
The current price of Charming Prints Company stock is $100.
Briefly discuss why might Aaron buy a Collar.
2014-MFE, Financial Economics 2 European Options, HCM 11/13/13, Page 36
2.21 (1 point) Several years ago Warren bought 1000 shares of XYZ stock.
Fortunately for Warren the value of XYZ stock has increased substantially since then.
However, for tax reasons Warren does not wish to sell his XYZ stock and realize his capital gains.
Rather Warren plans to sell his XYZ stock one year from now.
Warren is worried that by time he is ready to sell his stock his capital gains may decrease or vanish.
Briefly describe how Warren could purchase a European option to hedge this risk.
2.22 (2 points) Lauren buys a 70 strike put and a 90 strike call. The options have the same
expiration date. Graph the value of this portfolio when the options expire as a function of the future
price of the stock. This position is called a strangle.
2.23 (1 point)
You buy a European call with a strike price of 80 and sell a European put with a strike price of 80.
You also sell a European call with a strike price of 100 and buy a European put with a strike price of
100. All of these options are on the same stock and have the same expiration date.
Which of the following is a graph of the payoff on this portfolio as a function of the future price of the
stock?
50 100 150 200
S
- 20
- 10
10
20
Payoff
A.

50 100 150 200
S
10
20
30
40
Payoff
B.
50 100 150 200
S
- 100
- 75
- 50
- 25
25
50
75
Payoff
C.

50 100 150 200
S
- 100
- 50
50
100
Payoff
D.
E. None of A, B, C, or D.
2014-MFE, Financial Economics 2 European Options, HCM 11/13/13, Page 37
2.24 (1 point) The Rich and Fine Stock Index has a current price of 800.
An insurer offers a contract that will pay the value of the Rich and Fine Stock Index two years from
now; however, the contract will pay a minimum of 750. The insurer buys the index.
Briefly describe how the insurer could purchase a European option to hedge its risk.
2.25 (2 points) Options traders often refer to straddles and butterflies. Here is an example of each.
Straddle: Buy a call with strike price of $100 and simultaneously buy a put with strike price of $100.
Butterfly spread: Simultaneously buy one call with strike price of $100, sell two calls with strike price
of $110, and buy one call with strike price of $120.
Draw position diagrams for the straddle and butterfly, showing the payoffs from the investor's net
position. Each strategy is a bet on variability. Explain briefly the nature of each bet.
2.26 (1 point) You sell a European call with a strike price of 110,
and buy a European put with a strike price of 90.
The put and call are on the same stock and have the same expiration date.
Which of the following is a graph of the payoff on this portfolio as a function of the future price of the
stock?
60 80 100 120 140
S
- 40
- 20
20
40
Payoff
A.

60 80 100 120 140
S
10
20
30
40
Payoff
B.
60 80 100 120 140
S
- 60
- 40
- 20
20
40
60
Payoff
C.

60 80 100 120 140
S
- 40
- 20
20
40
Payoff
D.
E. None of A, B, C, or D
2014-MFE, Financial Economics 2 European Options, HCM 11/13/13, Page 38
2.27 (1 point) Which of the following statements are true?
1. A European call option can be exercised on or before the expiration date.
2. A put option will be worthless at the expiration date if the share price at that time is less than the
strike price.
3. An investor who sells a stock short sells something that he or she does not yet own.
A. 1 and 2 only.
B. 1 and 3 only.
C. 2 and 3 only.
D. 1, 2, and 3.
E None of A, B, C, or D is correct.
2.28 (2 points) Let Y
+
equal the maximum of Y and zero.
Let S and Q be two random variables.
(a) Determine (S - Q)
+
+ (Q - S)
+
.
(b) Determine (S - Q)
+
- (Q - S)
+
.
2.29 (CAS5B, 11/94, Q.28) (2 points)
a. Graph the overall position diagram when an investor simultaneously buys one call with an strike
price of $80, sells two calls with strike prices of $90, and buys one call with an strike price of $100.
The current price of the stock is $90. Graph the payoff versus the stock price for the total transaction.
List both coordinates of all points where the slope of the graph changes. DO NOT include any other
graphs or lines in your final answer.
b. Assuming that markets are efficient and that the investor is rational with no superior knowledge,
what is the investor's prediction on the price movement of the stock described in (a)?
Briefly explain the logic underlying your answer.
2.30 (CAS5B, 11/94, Q.30) (2.5 points) Mr. Clean has hired Mr. Slob to run his hog farm.
Mr. Clean has given Mr. Slob the following incentive plan: if, in exactly one year, the price of hogs
has risen by more than 10% from their current price of $50 each, Mr. Clean will pay Mr. Slob a
$10,000 bonus. What is the best estimate of the cost of this incentive scheme for Mr. Clean given
the following values of one-year European call options on hogs for the strike prices?
Call Option Strike Price Call Option Price
$48 $5.38
$50 $4.11
$52 $3.05
$54 $2.20
$56 $1.54
$58 $1.04
Note: I have rewritten this past exam question in order to match the current syllabus.
2014-MFE, Financial Economics 2 European Options, HCM 11/13/13, Page 39
2.31 (CAS5B, 5/95, Q.13) (1 point) You own a share of XYZ stock and are concerned that the
price of the stock may fall. Of the following choices, which would allow you to offset (at least partially)
potential future losses?
1. Buy a put on the share of stock.
2. Sell the stock short.
3. Sell a call on the share of stock.
A. 1 B. 2 C. 1, 2 D. 1, 2, 3 E. None of 1, 2, 3
2.32 (CAS5B, 5/95, Q.32) (2 points) The RegLuar Firm, Inc., a publicly held corporation, having
current assets of $75 million and no liabilities, borrows $50 million by issuing a zero coupon bond
due in two years. Assume no other transactions occur after the bond is issued and before it is
redeemed.
a. (1/2 point) Briefly describe this transaction in terms of options.
b. (3/4 points) If the value of the company's assets falls to $40 million at the end of one year,
discuss whether the stock has a nonzero value.
c. (3/4 points) At the end of two years, if the value of the company's assets falls to $40 million just
before the debt is paid, discuss whether the stock has nonzero value.
2.33 (CAS5B, 5/95, Q.35) (1.5 points)
a. (1 point) Explain how a term life insurance policy on the life of an actively employed actuary can
function similarly to a put option owned by the actuary's dependents.
b. (1/2 point) Under what circumstances does the term life policy fall short of operating like a put?
2014-MFE, Financial Economics 2 European Options, HCM 11/13/13, Page 40
2.34 (CAS5B, 11/98, Q.15) (1 point) What combination of stocks, options and borrowing/lending
could be represented by the following position diagram?
50 100 150 200
SharePrice
- 100
- 80
- 60
- 40
- 20
Valueof Position
1. Sell one share of stock short and borrow the present value of $100.
2. Sell one call with strike price of $100 and sell one put with strike price of $100.
3. Sell one share of stock short, sell two puts with strike price of $100, and lend the present
value of $100.
A. 1 B. 2 C. 3 D. 2, 3 E. 1, 2, 3
2.35 (CAS5B, 11/99, Q.30) (2 points) ABC Insurance Company has purchased a reinsurance
contract from Reliable Reinsurer providing coverage for $10 million in excess of $20 million. In other
words, Reliable Reinsurer has agreed to pay up to, but no more than, $10 million beyond the initial
$20 million in loss dollars retained by ABC.
a. (1 point) Draw a position diagram showing the payoff to ABC from the reinsurance as a function of
the amount of ABC's total loss. Label both axes.
b. (1 point) If we think of ABC's total loss as the "underlying asset," we can model this reinsurance
contract as a mixture of simple options.
Describe the option position that replicates the payoffs from the reinsurance contract.
2.36 (CAS5B, 11/99, Q.31) (2 points) Norbert Corporation owns a vacant lot with a book value
of $50,000. By a stroke of luck, Norbert finds a buyer willing to pay $200,000 for the lot. However,
Norbert must also give the buyer a put option to sell the lot back to Norbert for $200,000 at the end
of two years. Moreover, Norbert agrees to pay the buyer $40,000 for a call option to repurchase
the lot for $200,000 at the end of two years.
a. (1 point) What would likely happen if the lot is worth more than $200,000 at the end of
two years? What if it is worth less than $200,000? Why?
b. (1 point) In effect, Norbert has borrowed money from the buyer.
What is the effective annual interest rate per year on the loan? Show all work.
2014-MFE, Financial Economics 2 European Options, HCM 11/13/13, Page 41
2.37 (FM Sample Exam, Q.3) Happy Jalapenos, LLC has an exclusive contract to supply
jalapeno peppers to the organizers of the annual jalapeno eating contest. The contract states that
the contest organizers will take delivery of 10,000 jalapenos in one year at the market price. It will
cost Happy Jalapenos 1,000 to provide 10,000 jalapenos and today!s market price is 0.12 for one
jalapeno. The continuously compounded risk-free interest rate is 6%.
Happy Jalapenos has decided to hedge as follows (both options are one-year, European):
Buy 10,000 0.12-strike put options for 84.30 and sell 10,000 0.14-strike call options for 74.80.
Happy Jalapenos believes the market price in one year will be somewhere between 0.10 and
0.15 per pepper. Which interval represents the range of possible profit one year from now for
Happy Jalapenos?
A. 200 to 100 B. 110 to 190 C. 100 to 200 D. 190 to 390 E. 200 to 400
2.38 (FM Sample Exam, Q.9) You are given the following information:
The current price to buy one share of ABC stock is 100
The stock does not pay dividends
The risk-free rate, compounded continuously, is 5%
European options on one share of ABC stock expiring in one year have the following prices:
Strike Price Call option price Put option price
90 14.63 0.24
100 6.80 1.93
110 2.17 6.81
A butterfly spread on this stock has the following profit diagram.
85 90 95 100 105 110 115 120
- 2
2
4
6
Which of the following will NOT produce this profit diagram?
A. Buy a 90 put, buy a 110 put, sell two 100 puts
B. Buy a 90 call, buy a 110 call, sell two 100 calls
C. Buy a 90 put, sell a 100 put, sell a 100 call, buy a 110 call
D. Buy one share of the stock, buy a 90 call, buy a 110 put, sell two 100 puts
E. Buy one share of the stock, buy a 90 put, buy a 110 call, sell two 100 calls.
2014-MFE, Financial Economics 2 European Options, HCM 11/13/13, Page 42
Solutions to Problems:
2.1. (20%)(60) + (30%)(80) + (30%)(100) + (20%)(120) = 90.
2.2. D. (20%)(0) + (30%)(0) + (30%)(100 - 85) + (20%)(120 - 85) = 11.5.
2.3. A. (20%)(85 - 60) + (30%)(85 - 80) + (30%)(0) + (20%)(0) = 6.5.
2.4. Graph of the future value of a European call with strike price of 100, E[(S - 100)
+
]:
50 100 150 200 250
Stock Price
20
40
60
80
100
120
140
Option Value
2.5. Graph of the future value of a European put with strike price of 100, E[(100 - S)
+
]:
50 100 150 200 250
Stock Price
20
40
60
80
100
Option Value
2014-MFE, Financial Economics 2 European Options, HCM 11/13/13, Page 43
2.6. Graph of the payoff of a European call plus put each with strike price of 100,
E[(S - 100)
+
] + E[(100 - S)
+
]:
50 100 150 200
Stock Price
20
40
60
80
100
Payoff
Comment: This straddle pays a large amount if the future stock price differs a lot from $100.
If $100 is the current price, this is one way to speculate on volatility.
Similar to Figure 3.10 in Derivatives Markets by McDonald.

2.7. The payoff for the portfolio is: (S
T
- 60)
+
- 2(S
T
- 70)
+
+ (S
T
- 80)
+
.
If S
T
# 60, then the payoff is nothing.
If 70 " S
T
> 60, then the payoff is: S
T
- 60.
If 80 " S
T
> 70, then the payoff is: (S
T
- 60) - 2(S
T
- 70) = 80 - S
T
.
If S
T
> 80, then the payoff is: (S
T
- 60) - 2(S
T
- 70) + (S
T
- 80) = 0.
A graph of the payoff:
50 60 70 80 90 100
S
2
4
6
8
10
Payoff
2014-MFE, Financial Economics 2 European Options, HCM 11/13/13, Page 44
2.8. The payoff for the portfolio is: (100 - S
T
)
+
- (120 - S
T
)
+
.
If S
T
" 120, then the payoff is nothing.
If 120 > S
T
" 100, then the payoff is: -(120 - S
T
) = S
T
- 120.
If 100 > S
T
, then the payoff is: (100 - S
T
) - (120 - S
T
) = -20.
A graph of the payoff:
60 80 100 120 140 160
S
- 20
- 15
- 10
- 5
Payoff
Comment: The premium for the 100 strike put is less than the premium for the 120 strike put.
Joe gained money from setting up this portfolio. Joe is hoping that the future stock price will be at
least 120. Similar to Figure 3.7 in Derivatives Markets by McDonald.
2.9. The payoff for the portfolio is: (S
T
- 90)
+
- (90 - S
T
)
+
- (S
T
- 130)
+
+ (130 - S
T
)
+
.
If S
T
# 90, then the payoff is: -(90 - S
T
) + (130 - S
T
) = 40.
If 130 " S
T
> 90, then the payoff is: (S
T
- 90) + (130 - S
T
) = 40.
If S
T
> 130, then the payoff is: (S
T
- 90) - (S
T
- 130) = 40. A graph of the payoff:
80 100 120 140
S
20
40
60
80
Payoff
Comment: The box-spread has a risk free payoff; buying a box-spread is equivalent to buying a
bond. Writing a box-spread is equivalent to borrowing money.
2014-MFE, Financial Economics 2 European Options, HCM 11/13/13, Page 45
2.10. The payoff for the portfolio is: 2(S
T
- 100)
+
- 3(S
T
- 110)
+
+ (S
T
- 130)
+
.
If S
T
# 100, then the payoff is nothing.
If 110 " S
T
> 100, then the payoff is: 2(S
T
- 100).
If 130 " S
T
> 110, then the payoff is: 2(S
T
- 100) - 3(S
T
- 110) = 130 - S
T
.
If S
T
> 130, then the payoff is: 2(S
T
- 100) - 3(S
T
- 110) + (S
T
- 130) = 0. A graph of the payoff:
80 100 120 140
S
5
10
15
20
Payoff
Comment: As will be discussed, such a Asymmetric Butterfly Spread may be used to take
advantage of certain arbitrage opportunities. + = (130 - 110)/(130 - 100) = 2/3.
Buy + of the lowest strike, sell 1 of the middle strike, and buy (1 - +) of the highest strike.
In this case, buy 2/3 of 100 strike, sell 1 of the 110 strike, and buy 1/3 of the 130 strike.
Here Amanda has multiplied this position by three.
2014-MFE, Financial Economics 2 European Options, HCM 11/13/13, Page 46
2.11. We want: (1000)(S
T
- 120)
+
= (1000)(8) + 100 = 8100. % S
T
= $128.10.
If S
T
< 120, the calls are worthless, and Robert!s profit is: -8100. This is equivalent to -100%.
If S
T
> 120, Robert!s profit is: (1000)(S
T
- 120) - 8100 = 1000S
T
- 128,100.
As a ratio to his initial investment of 8100, this is: 0.12346S
T
- 15.815.
Here is a graph of Robert!s profit as a percent of his initial investment as a function of the stock price
at expiration:
100 110 120 130 140 150
S
-1 0 0
- 50
50
100
150
200
250
Percent Profit
Comment: Notice the large leverage when one invests in an option.
A small change in the stock price at expiration has a large effect on Robert!s profit.
2.12. The payoff for the portfolio is: (S
T
- 90)
+
- (90 - S
T
)
+
.
If S
T
# 90, then the payoff is: -(90 - S
T
) = S
T
- 90. If S
T
> 90, then the payoff is: S
T
- 90.
A graph of the payoff:
60 70 80 90 100 110 120
S
- 30
- 20
- 10
10
20
30
Payoff
Comment: I only graphed from a future stock price of 60 to 120. If S
T
= 60, then the person to
whom Tiffany sold the put will require Tiffany to buy the stock for 90 from this person, even though
the stock is only worth 60. If S
T
= 60, then Tiffany has a payoff of 60 - 90 = -30.
2014-MFE, Financial Economics 2 European Options, HCM 11/13/13, Page 47
2.13. The payoff for the portfolio is: (70 - S
T
)
+
- (S
T
- 90)
+
.
If S
T
# 70, then the payoff is: 70 - S
T
.
If 90 " S
T
> 70, then the payoff is: 0.
If S
T
> 90, then the payoff is: S
T
- 90.
A graph of the payoff:
50 60 70 80 90 100 110
S
- 20
- 10
10
20
Payoff
Comment: Similar to Figure 3.8 in Derivatives Markets by McDonald.
2.14. B. Harold!s bought a call and sold the otherwise similar put.
The payoff on Harold!s position is: (S
2
- 80)
+
- (80 - S
2
)
+
= S
2
- 80.
Since ABC pays no dividends, the prepaid forward price for S
2
is just S
0
= 78.
The prepaid forward price to receive $80 two years from now is 80e
-2r
.
Therefore, the price for Harry!s position is: 78 - 80e
-2r
.
Set 6.33 = 78 - 80e
-2r
. % r = 5.5%.
2014-MFE, Financial Economics 2 European Options, HCM 11/13/13, Page 48
2.15. The payoff for the portfolio is: (70 - S
T
)
+
- 4(100 - S
T
)
+
+ 3(110 - S
T
)
+
.
If S
T
# 70, then the payoff is: (70 - S
T
) - 4(100 - S
T
) + 3(110 - S
T
) = 0.
If 100 " S
T
> 70, then the payoff is: -4(100 - S
T
) + 3(110 - S
T
) = S
T
- 70.
If 110 " S
T
> 100, then the payoff is: 3(110 - S
T
).
If S
T
> 110, then the payoff is nothing.
A graph of the payoff:
60 80 100 120
S
5
10
15
20
25
30
Payoff
2014-MFE, Financial Economics 2 European Options, HCM 11/13/13, Page 49
2.16. We want: (100)(80 - S
T
)
+
= (100)(5) + 60 = 560. % S
T
= $74.40.
If S
T
> 74.40, the puts are worthless, and Kimberly!s profit is: -560. This is equivalent to -100%.
If S
T
< 74.40, Kimberly!s profit is: (100)(80 - S
T
)
+
- 560 = 7440 - 100S
T
.
As a ratio to her initial investment of 560, this is: 13.2857 - 0.17857S
T
.
Here is a graph of Kimberly!s profit as a percent of her initial investment as a function of the stock
price at expiration:
50 60 70 80 90 100
S
- 100
100
200
300
400
Percent Profit
Comment: Notice the large leverage when one invests in an option.
A small change in the stock price at expiration has a large effect on Kimberly!s profit.
2.17. The value of this portfolio when the options expire is: S
T
- (S
T
- 110)
+
+ (110 - S
T
)
+
.
If S
T
# 110, then the value is: S
T
+ (110 - S
T
) = 110.
If S
T
> 110, then the payoff is: S
T
- (S
T
- 110) = 110.
A graph of the value:
80 90 100 110 120 130 140
S
50
100
150
200
Value
2014-MFE, Financial Economics 2 European Options, HCM 11/13/13, Page 50
2.18. Max[S(T), 100] = 100 + Max[S(T) - 100, 0] = 100 + (S(T) - 100)
+
.
(S(T) - 100)
+
is the payoff on a T-year 100-strike European call on this stock.
The present value of a payoff T years from now of 100 is: 100 e
-rT
.
Thus the premium for this contract is: 100 e
-rT
+ C,
where C is the premium on a T-year 100-strike European call.
Alternately, Max[S(T), 100] = S(T) + Max[100 - S(T), 0] = S(T) + (100 - S(T))
+
.
(100 - S(T))
+
is the payoff on a T-year 100-strike European put on this stock.
The prepaid forward price for S(T) is: S(0) e
-#T

Thus the premium for this contract is: S(0) e
-#T
+ P,
where P is the premium on a T-year 100-strike European call.
Comment: As will be discussed, the call premium and put premiums are connected via put-call
parity. Thus one can show that the two forms of the premium for this contract are equivalent.
2.19. The payoff is: -(S - 60)
+
- (60 - S)
+
.

2014-MFE, Financial Economics 2 European Options, HCM 11/13/13, Page 51
2.20. Collar: Purchase a put and sell a call with a higher strike price.
For example, Aaron might buy a 90 strike put and sell a 110 strike call, each of which expire 6
months from now. (There are many other possible Collars.)
Then the value of his portfolio 6 months from would be: S
.5
+ (90 - S
.5
)
+
- (S
.5
- 110)
+
.
If S
.5
# 90, then Aaron!s portfolio is worth 90. (Aaron will use his put to sell the stock for 90.)
If 90 < S
.5
< 110, then Aaron!s portfolio is S
.5
.
If S
.5
" 110, then Aaron!s portfolio is worth 110. (The person to whom Aaron sold the call, will use
the call to buy the stock for 110 from Aaron.)

By buying this collar, Aaron has limited the value of his position in 6 months to be between 90 and
110. Aaron can not make a lot, but also he can not lose a lot.
Comment: Using the Black-Scholes formula, to be discussed subsequently, if the stock pays no
dividends, the stock has a volatility of 30%, and r = 5%, then the premium for this collar is -2.60; in
other words, Aaron will make more money from selling the call than he spends buying the put.
If instead for example, Aaron had bought a 120 strike put and sold a 140 strike call, each of which
expire 2 years from now, then he would have limited the value of his position in 2 years to be
between 120 and 140.
2.21. Warren could buy one thousand 1-year at-the money European puts on XYZ stock.
If one year from now XYZ stock is worth more than its current price, then he can sell his stock and
make more in capital gains than he has currently.
If one year from now XYZ stock is worth less than its current price, then he could use his puts to sell
his stock at its price today and make in capital gains the amount he has currently.
Comment: Buying a put protects against the price of a stock you own going down.
Buying a call would protect against the price of a stock you shorted going up.
2014-MFE, Financial Economics 2 European Options, HCM 11/13/13, Page 52
2.22. The payoff is: (70 - S)
+
+ (S - 90)
+
.

2.23. B. (S - 80)
+
- (80 - S)
+
= S - 80.
(S - 100)
+
- (100 - S)
+
= S - 100.
(S - 80)
+
- (80 - S)
+
- {(S - 100)
+
- (100 - S)
+
} = S - 80 - (S - 100) = 20.
Comment: This position is called a Box Spread.
2.24. The insurer could buy a 2-year 750-strike European put on the Rich and Fine Stock Index.
If two years from now the index is worth more than 750, then the insurer can sell the index, pay off
the contract, and have some money left over.
If two years from now the index is worth less than 750, then the insurer can use its put to sell the
index for 750 and pay off the contract.
2014-MFE, Financial Economics 2 European Options, HCM 11/13/13, Page 53
2.25. Payoff diagram for the straddle:
90 100 110 120
S
5
10
15
20
Payoff
The purchaser of the straddle hopes that the stock price moves a lot; the purchaser is betting that
there will be high volatility in the stock price.
Payoff diagram for the butterfly spread:
100 110 120 130
S
2
4
6
8
10
Payoff
The purchaser of the butterfly spread hopes that the stock price does not move a lot; the purchaser
is betting that there will be low volatility in the stock price.
2.26. D. (90 - S)
+
- (110 - S)
+
is equal to:
110 - S, for S > 100,
0, for 90 < S < 110
90 - S, for S < 90.
Comment: This position is called a Collar.
2014-MFE, Financial Economics 2 European Options, HCM 11/13/13, Page 54
2.27. E. Statement number 1 is false. A European options can be exercised only on the expiration
date. The statement is the correct definition for an American option.
Statement number 2 is false. In fact, the opposite is true: the lower the market price on the expiration
date, the higher the value of the put option.
Statement number 3 is correct. Short sellers sell stock which they do not yet own.
2.28. (a) (S - Q)
+
=

S - Q if S & Q
0 if S < Q
'
(
)
. (Q - S)
+
=

0 if S & Q
Q - S if S < Q
'
(
)
.
Therefore, (S - Q)
+
+ (Q - S)
+
=

S - Q if S & Q
Q - S if S < Q
'
(
)
= |S - Q|.
(b) (S - Q)
+
=

S - Q if S & Q
0 if S < Q
'
(
)
. (Q - S)
+
=

0 if S & Q
Q - S if S < Q
'
(
)
.
Therefore, (S - Q)
+
- (Q - S)
+
=

S - Q if S & Q
S - Q if S < Q
'
(
)
= S - Q.
Comment: If Q were a constant, then (S - Q)
+
+ (Q - S)
+
would be the payoff on a Q-strike call and
the similar Q-Strike put; in other words the payoff on a straddle is: (S - K)
+
+ (K - S)
+
= |S - K|.
If Q were a constant, then (S - Q)
+
- (Q - S)
+
would be the payoff on a Q-strike call and the sale of a
similar Q-Strike put. The fact that (S - K)
+
- (K - S)
+
= S - K, is the basis of put-call parity, to be
discussed in a subsequent section.
2014-MFE, Financial Economics 2 European Options, HCM 11/13/13, Page 55
2.29. a. The payoff diagram:
80 90 100
S
10
Payoff
(90, 10)
(100, 0) (80, 0)
b. The investor is betting that the volatility of the stock will be less than the market expects.
The investor has a large payoff if the stock price moves a small amount from its initial price, and no
payoff if the stock price moves a large amount from its initial price.
Comment: This is a Butterfly spread; there would have been a net cost to setting up this position.
2.30. If the price of hogs in one year is $55 or more, then Mr. Slob gets his bonus.
We can approximate such a payoff by buying a 54-strike call and selling a 56-strike call.
The payoff will be:

0 if S < 54
S - 54 if 54 , S , 56
2 if S > 56
'
(
-
)
-
.
The cost of one such position is: $2.20 - $1.54 = $0.66.
If S > 56 the payoff is $2, so Mr. Clean would need to buy $10,000/$2 = 5000 such positions to
fund the bonus.
Thus the cost of this incentive scheme is: (5000)(0.66) = $3300.
Comment: The match between the bonus and the position of calls is approximate.
There are other combinations of calls that would also approximate the bonus.
An exact match would be provided by 55-strike cash-or-nothing calls, an exotic option to be
discussed in a subsequent section.
2014-MFE, Financial Economics 2 European Options, HCM 11/13/13, Page 56
2.31. D. If the stock price goes down, the put will have a positive payoff, so Statement #1 is true.
Owning a share and also selling a share short, you will be unaffected by stock price movements, so
Statement #2 is true.
If the stock price goes down, the call will have no payoff; you can use the money you got from
selling the call to offset some of the losses on the stock. Thus Statement #3 is true.
Comment: Buying a put would be the usual way to hedge the risk of the stock price declining.
2.32. a. When a firm borrows, the equity holders exchange their claim on the assets of the firm for a
call option on the whole firm with an strike price equal to the maturity value of the debt. In other
words, the option holder can either exercise the call option at the exercise date, pay the strike price,
and obtain the stock, or the option holder can choose not to exercise the call option and he or she is
left with no stock. Similarly, the equity holders can either repay the debt at the maturity date for the
par value and retain the assets of the corporation, or they can default on the debt, in which case they
are left with nothing since the bondholders take the assets of the corporation.
b. There is still a chance that the asset of the company will increase beyond $50 million by the end
of the second year, so the stock still has value. This is equivalent to an out-of-the money call with a
year until expiration; such a call has a positive if small value.
c. The company!s assets are less than the money owed to the bondholders, so the stock is now
worthless. An out-of-the money call at expiration is worthless.
2.33. a. The actuary's family has a claim on the future wages of the actuary. If the actuary dies, the
value of that claim falls to zero. The term life policy is like a put option which pays off if the actuary
dies over the term of the policy.
b. Events other than death can reduce the actuary's future wages, for example layoff or disability.
Under these events, the term life policy will not pay off.
Comment: The analogy is a little strained.
2.34. D. This is the diagram for selling (writing) a straddle, position #2.
If there are no dividends, then by put call parity, the buying a call is equivalent to buying one share
of stock, buying one puts, and borrowing the present value of the strike.
Thus, in this case, position #3 would be equivalent to selling a call and selling a put, position #2.
The value of position #1 at future time T is: S
T
- 100e
rT
, not the given graph.
2014-MFE, Financial Economics 2 European Options, HCM 11/13/13, Page 57
2.35. a. Payoff is: Min[10 million, (L - 20 million)
+
].
15 20 25 30 35 40
Loss
2
4
6
8
10
Payoff
b. Buy a 20 strike call and sell a 30 strike call.
(S - 20)
+
- (S - 30)
+
=

0, S < 20
S - 20, 20 < S < 30
10, S > 30
'
(
-
)
-
Comment: Many insurance and reinsurance arrangements can be thought of in terms of options.
2.36. a. If in two years the lot is worth more than $200,000, then Norbert will use its call to
repurchase the lot for $200,000. If in two years the lot is worth less than $200,000, then the put
Norbert gave the buyer will be used to sell the lot to Norbert for $200,000.
b. In either case, Norbert gets a net of $200,000 - $40,000 = $160,000 today and has $200,000
(without the lot) in two years.
Interest rate = (200,000/160,000)
1/2
- 1 = 11.8%.
2.37. D. The accumulated cost of the hedge is: (84.30 - 74.80)exp(.06) = 10.09.
Let x be the market price.
If x < 0.12, the put is in the money and the payoff is: 10,000(0.12 x) = 1,200 - 10,000x.
The sale of the jalapenos has a payoff of: 10,000x - 1,000.
The profit is: 1,200 - 10,000x + 10,000x - 1,000 - 10.09 = 190.
From 0.12 to 0.14 neither option has a payoff, and the profit is:
10,000x - 1,000 - 10.09 = 10,000x - 1,010. This ranges from 190 to 390.
If x > 0.14, the call is in the money and the payoff is: -10,000(x - 0.14) = 1,400 - 10,000x.
The profit is: 1,400 - 10,000x + 10,000x - 1,000 - 10.09 = 390.
The range of possible profit one year from now is: 190 to 390.
2014-MFE, Financial Economics 2 European Options, HCM 11/13/13, Page 58
2.38. D. The cost to set up portfolio A is: 0.24 + 6.81 - (2)(1.93) = 3.19.
If S
1
< 90, then the profit is: (90 - S
1
) + (110 - S
1
) - 2(100 - S
1
) - 3.19e
.05
= -3.35.
If 90 < S
1
< 100, then the profit is: (110 - S
1
) - 2(100 - S
1
) - 3.19e
.05
= S
1
- 93.35.
If S
1
= 100, then the profit is: 10 - 3.19e
.05
= 6.65.
If 100 < S
1
< 110, then the profit is: (110 - S
1
) - 3.19e
.05
= 106.65 - S
1
.
If 110 < S
1
, then the profit is: -3.19e
.05
= -3.35.
The cost to set up portfolio B is: 14.63 + 2.17 - (2)(6.80) = 3.20.
If S
1
< 90, then the profit is: -3.20e
.05
= -3.36.
If 90 < S
1
< 100, then the profit is: (S
1
- 90) - 3.20e
.05
= S
1
- 93.36.
If S
1
= 100, then the profit is: 10 - 3.20e
.05
= 6.64.
If 100 < S
1
< 110, then the profit is: (S
1
- 90) - 2(S
1
- 100) - 3.20e
.05
= 106.64 - S
1
.
If 110 < S
1
, then the profit is: (S
1
- 90) - 2(S
1
- 100) + (S
1
- 110) - 3.20e
.05
= -3.36.
The cost to set up portfolio C is: 0.24 + 2.17 - 6.80 - 1.93 = -6.32.
If S
1
< 90, then the profit is: (90 - S
1
) - (100 - S
1
) + 6.32e
.05
= -3.36.
If 90 < S
1
< 100, then the profit is: -(100 - S
1
) + 6.32e
.05
= S
1
- 93.36.
If S
1
= 100, then the profit is: 6.32e
.05
= 6.64.
If 100 < S
1
< 110, then the profit is: -(S
1
- 100) + 6.32e
.05
= 106.64 - S
1
.
If 110 < S
1
, then the profit is: -(S
1
- 100) + (S
1
- 110) + 6.32e
.05
= -3.36.
The cost to set up portfolio D is: 100 + 14.63 + 6.81 - (2)(1.93) = 117.68.
If S
1
< 90, then the profit is: S
1
+ (110 - S
1
) - 2(100 - S
1
) - 117.68e
.05
= 2S
1
- 213.71,
not matching the given graph.
If 90 < S
1
< 100, then the profit is: S
1
+ (S
1
- 90) + (110 - S
1
) - 2(100 - S
1
) - 117.68e
.05

= 3S
1
- 303.71, not matching the given graph.
If S
1
= 100, then the profit is: 100 + 10 + 10 - 117.68e
.05
= -3.71, not matching the given graph.
If 100 < S
1
< 110, then the profit is: S
1
+ (S
1
- 90) + (110 - S
1
) - 117.68e
.05
=
S
1
- 103.71, not matching the given graph.
If 110 < S
1
, then the profit is: S
1
+ (S
1
- 90) - 117.68e
.05
= 2S
1
- 213.71, not matching the given
graph.
The cost to set up portfolio E is: 100 + 0.24 + 2.17 - (2)(6.80) = 88.81.
If S
1
< 90, then the profit is: S
1
+ (90 - S
1
) - 88.81e
.05
= -3.36.
2014-MFE, Financial Economics 2 European Options, HCM 11/13/13, Page 59
If 90 < S
1
< 100, then the profit is: S
1
- 88.81e
.05
= S
1
- 93.36.
If S
1
= 100, then the profit is: 100 - 93.36 = 6.64.
If 100 < S
1
< 110, then the profit is: S
1
- 2(S
1
- 100) - 88.81e
.05
= 106.64 - S
1
.
If 110 < S
1
, then the profit is: S
1
- 2(S
1
- 100) + (S
1
- 110) - 88.81e
.05
= -3.36.
Comment: See Exercise 3.18 in Derivatives Markets by McDonald.
For each strike price, put-call parity holds. With no dividends, C - P = 100 - Ke
-.05
.
2014-MFE, Financial Economics 2 European Options, HCM 11/13/13, Page 60
Section 3, Properties of Premiums of European Options
The premium of an option is its current price, the actuarial present value of its future payoffs.
As will be discussed in subsequent sections, in order to properly price an option we have to use
risk-neutral probabilities; we can price options in the risk-neutral environment.
Actuarial Present Value of a Call Option:
The actuarial present value of a European call option is: E[(S
T
- K)
+
] e
-rT
.
If we let C(S
0
, K, T) be the actuarial present value of the call on a stock with current price S
0
,
strike price K, and time until expiration T: C
Eur
(S
0
, K, T) = E[(S
T
- K)
+
] e
-rT
.
Exercise: The price of a stock two years from now has the following distribution:
$50 @ 20%, $100 @ 40%, $150 @30%, $200 @10%.
The continuously compounded annual risk free rate is: r = 4%.
Determine the actuarial present value of a European call option on this stock with 2 years to
expiration and a strike price of $90.
[Solution: e
-0.08
{(20%)(0) + (40%)(10) + (30%)(60) + (10%)(110)} = $30.46.]
Exercise: In the previous exercise, change the strike price to $100.
[Solution: e
-0.08
{(20%)(0) + (40%)(0) + (30%)(50) + (10%)(100)} = $23.08.]
The actuarial present value of this European call option as a function of the strike price:
50 100 150 200
K
20
40
60
80
100
PV
The actuarial present value, in other words premium, of the call decreases as the strike
price increases and the curve is concave upwards.
2014-MFE, Financial Economics 3 Properties of Premiums, HCM 11/13/13, Page 61
General Properties of European Call Premiums:
These are general properties for the behavior of a call as one varies the strike price.
55

The value of an call option with strike price $90 is greater than an otherwise similar call with a strike
price of $100. It is less valuable to have the option to buy something at $100 than it is to have the
option to buy that same thing at $90. Call premiums decrease as the strike price increases:
For K
1
< K
2
, C(K
1
) " C(K
2
).
56

57


.C
.K
# 0.
For example, assume the future price of a stock at expiration of a call has the following distribution:
$50 @ 20%, $100 @ 40%, $150 @30%, $200 @10%.
Stock Payoff on Call Payoff on Call
Price with K = 90 with K = 100 Difference
50 0 0 0
100 10 0 10
150 60 50 10
200 110 100 10
The difference in payoffs is at most the difference in strike prices, 10.
Being able to buy at $90 is worth at most $10 more than being able to buy at $100.
58

Therefore the difference in call premiums is at most the difference in strike prices:
For K
1
< K
2
, C(K
1
) - C(K
2
) # K
2
- K
1
.
59

60

.C
.K
" -1.
Since European options can only be exercised at expiration, the difference in option premiums
cannot be more than the present value of the difference in payoffs. Thus:
For K
1
< K
2
, C(K
1
) - C(K
2
) # e
-rT
(K
2
- K
1
).
61


.C
.K
" -e
-rT
.
55
See Section 9.3 in Derivatives Markets by McDonald.
56
See equation 9.15 in Derivatives Markets by McDonald.
57
This equation holds both for European and American options, to be discussed subsequently.
58
Sometimes being able to buy at $90 and being able to buy at $100 are both worth nothing.
59
See equation 9.17 in Derivatives Markets by McDonald.
60
This equation holds both for European and American options, to be discussed subsequently.
61
See Appendix 9.B in McDonald, not on the syllabus.
2014-MFE, Financial Economics 3 Properties of Premiums, HCM 11/13/13, Page 62
The third property is referred to as the convexity of the option price with respect to the strike price.
62

For K
1
< K
2
< K
3
,

C(K
1
) - C(K
2
)
K
2
- K
1
"

C(K
2
) - C(K
3
)
K
3
- K
2
.
63

64


.
2
C
.K
2
" 0.
For example, let us assume that a call with strike price $90 is worth $7 more than a similar call with a
strike price of $100. Then a call with a strike price of $100 exceeds that of a similar call with a strike
price of $110, but by less than or equal to $7.
Convexity follows from the fact that the second derivative of the call premium with respect to K is
positive:

.
2
C
.K
2
" 0. C decreases as K increases and the slope is negative; as K increases the slope
increases, in other words gets closer to zero. As K decreases the slope decreases; however, the
slope can not get less than -1.
As mentioned before, the graph of C as a function of K is concave upwards.
62
A convex function has a curve that is concave upwards, shaped like a bowl.
63
See equation 9.19 in Derivatives Markets by McDonald. This equation holds both for European and American
options, to be discussed subsequently.
64
Increased Limits Factors share this same property for the same underlying mathematical reason.
See Sheldon Rosenberg!s review of On the Theory of Increased Limits and Excess of Loss Pricing, PCAS 1977.
2014-MFE, Financial Economics 3 Properties of Premiums, HCM 11/13/13, Page 63
Mathematically the three conditions for call premiums could be summarized as:
-1 #

.C
.K
# 0, and

.
2
C
.K
2
" 0.
This is illustrated in the following graph:
65

50 100 150 200
K
20
40
60
80
100
C
As K approaches zero, the slope of the above curve approaches -e
-rT
, which is close to but more
than minus one. As K approaches infinity, the slope of the above curve approaches 0.
65
The put premiums are computed via the Black-Scholes formula to be discussed subsequently.
This is for a 2-year European call, and / = 40%, r = 6%, # = 0.
2014-MFE, Financial Economics 3 Properties of Premiums, HCM 11/13/13, Page 64
Explaining the Behavior of the Value of Calls as a Function of K:
Let F(S) be the distribution of the future price of the stock.
The expected value of the call is:
E[(S
T
- K)
+
] =

{1 - F(x)} dx
K
0
1
.
66

Therefore, C(K) = e
-rT

{1 - F(x)} dx
K
0
1
.

.C
.K
= -e
-rT
{1 - F(K)} # 0.
Therefore, C declines as K increases.
C(K
1
) - C(K
2
) = e
-rT

{1 - F(x)} dx
K
1
K
2
1
# e
-rT
(K
2
- K
1
) {1 - F(K
1
)} # e
-rT
(K
2
- K
1
) # K
2
- K
1
.
67


.
2
C
.K
2
= e
-rT
f(K) " 0.
Therefore, C(K) is concave upwards.
66
See Mahler!s Guide to Loss Distributions, covering material on Exam C.
This is an expression for the expected excess losses.
67
Where I have used the fact that the survival function, 1 - F(x), is nonincreasing.
Thus on the interval from K
1
to K
2
, the survival function is at most its value at the lower endpoint: 1 - F(K
1
).
Thus the integral is at most the width of the interval times 1 - F(K
1
).
2014-MFE, Financial Economics 3 Properties of Premiums, HCM 11/13/13, Page 65
Lee Diagrams and Call Premiums:
68

One can present the same ideas graphically via Lee Diagrams.
69
Lee Diagrams have the x-axis
correspond to probability, while the y-axis corresponds to size of loss. Here we will graph the
distribution function of the future price of the stock, with the horizontal axis corresponding to
probability and the vertical axis corresponding to the stock price at expiration of the option.
The expected payoff of a European Call, is equal to E[(S
T
- K)
+
].
E[(X - K)
+
] is the expected losses excess of K, and corresponds to the area on the Lee Diagram
above the horizontal line at height K and also below the curve graphing F(x).

As K increases, the area above the horizontal line at height K decreases; in other words, the value of
the call decreases as K increases.
68
Not on the syllabus of your exam!
69
See The Mathematics of Excess of Loss Coverage and Retrospective Rating --- A Graphical Approach,
by Y.S. Lee, PCAS LXXV, 1988. Currently on the syllabus of CAS Advanced Ratemaking Exam.
See also Mahler!s Guide to Loss Distributions, covering material on Exam 4/C.
2014-MFE, Financial Economics 3 Properties of Premiums, HCM 11/13/13, Page 66
For an increase in K of $K, the value of the call decreases by Area A in the following Lee Diagram:
1
Prob.
Stock Price
A
K+$K
K
The absolute value of the change in the value of the call, Area A, is smaller than a rectangle of height
$K and width 1 - F(K). Thus Area A is smaller than $K {1 - F(K)} # $K. Thus a change of $K in the
strike price results in a absolute change in the value of the call option smaller than $K.
The following Lee Diagram shows the effect of raising the strike price by fixed amounts:

The successive absolute changes in the value of the call are represented by Areas A, B, C, and D.
We see that the absolute changes in the value of the call get smaller as the strike price increases.
2014-MFE, Financial Economics 3 Properties of Premiums, HCM 11/13/13, Page 67
Actuarial Present Value of a Put Option:
The actuarial present value of a European put option is: E[(K - S
T
)
+
] e
-rT
.
If we let P(S
0
, K, T) be the actuarial present value of the put on a stock with current price S
0
,
strike price K, and time until expiration T: P
Eur
(S
0
, K, T) = E[(K - S
T
)
+
] e
-Tr
.
Exercise: The price of a stock two years from now has the following distribution:
$50 @ 20%, $100 @ 40%, $150 @30%, $200 @10%.
r = 4%. Determine the actuarial present value of a European put option on this stock with 2 years to
expiration and a strike price of $120.
[Solution: e
-0.08
{(20%)(70) + (40%)(20) + (30%)(0) + (10%)(0)} = $20.31.]
The actuarial present value of the above European put option as a function of the strike price:
60 80 100 120 140 160 180 200
K
20
40
60
80
PV
The actuarial present value of the put increases as the strike price increases and the
curve is concave upwards.
These are general properties for the behavior of a put as one varies the strike price.
70

70
See Section 9.3 in Derivatives Markets by McDonald.
2014-MFE, Financial Economics 3 Properties of Premiums, HCM 11/13/13, Page 68
General Properties of European Put Premiums:
For K
1
< K
2
, P(K
1
) # P(K
2
) .
71

72


.P
.K
" 0.
For example, the value of an put option with strike price $100 is greater than an otherwise similar put
with a strike price of $90. It is more valuable to have the option to sell at $100 than to have the
option to sell at $90.
For K
1
< K
2
, P(K
2
) - P(K
1
) # K
2
- K
1
.
73

74

.P
.K
# 1.
For example, the value of a put option with strike price $100 is greater than an otherwise similar put
with a strike price of $90 by at most $10. Being able to sell at $100 is worth at most $10 more than
being able to sell at $90.
75

Since European options can only be exercised at expiration, the difference in option premiums
cannot be more than the present value of the difference in payoffs. Thus:
For K
1
< K
2
, P(K
2
) - P(K
1
) # e
-rT
(K
2
- K
1
).
76


.P
.K
# e
-rT
.
For K
1
< K
2
< K
3
,

P(K
2
) - P(K
1
)
K
2
- K
1
#

P(K
3
) - P(K
2
)
K
3
- K
2
.
77


.
2
P
.K
2
" 0.
For example, let us assume that a put with strike price $100 is worth $6 more than a similar put with a
strike price of $90. Then a put with a strike price of $110 exceeds that of a similar put with a strike
price of $100 by at least $6.
This is referred to as the convexity of the option price with respect to the strike price.
78
It follows from
the fact that the second derivative of the value of the put with respect to K is positive:

.
2
P
.K
2
" 0.
P increases as K increases and the slope is positive; as K increases the slope increases, in other
words gets further from zero. However, the slope can not exceed one.
71
See equation 9.16 in Derivatives Markets by McDonald.
72
This equation holds both for European and American options, to be discussed subsequently.
73
See equation 9.18 in Derivatives Markets by McDonald.
74
This equation holds both for European and American options, to be discussed subsequently.
75
Sometimes being able to sell at $90 and being able to sell at $100 are both worth nothing.
76
See Appendix 9.B in McDonald, not on the syllabus.
77
See equation 9.20 in Derivatives Markets by McDonald. This equation holds both for European and American
options, to be discussed subsequently.
78
A convex function has a curve that is concave upwards, shaped like a bowl.
2014-MFE, Financial Economics 3 Properties of Premiums, HCM 11/13/13, Page 69
As mentioned before, the graph of P as a function of K is concave upwards.
79
Mathematically the three conditions for put premiums could be summarized as:
1 "

.P
.K
" 0, and

.
2
P
.K
2
" 0.
This is illustrated in the following graph:
80

40 60 80 100
K
5
10
15
P
As K approaches infinity, the slope of the above curve approaches e
-rT
, which is close to but less
than one. As K approaches zero, the slope of the above curve approaches 0.
79
The graph of a function is concave upwards if it is shaped like a bowl. If the second derivative is positive, then the
graph of a function is concave upwards. A convex function is such that for any 0 # t # 1, and x # y,
f(tx + (1-t)y) # tf(x) + (1 -t)f(y). A convex function has a graph that is concave upwards.
80
The put premiums are computed via the Black-Scholes formula to be discussed subsequently.
This is for a 2-year European put, and / = 40%, r = 6%, # = 0.
2014-MFE, Financial Economics 3 Properties of Premiums, HCM 11/13/13, Page 70
Explaining the Behavior of the Value of Puts as a Function of K:
As before, let F(S) be the distribution of the future price of the stock.
The expected value of the put is:
81
E[(K - S
T
)
+
] =

F(x) dx
0
K
1
.
Therefore, P(K) = e
-rT

F(x) dx
0
K
1
.

.P
.K
= e
-rT
F(K) " 0.
Therefore, P increases as K increases.
P(K
2
) - P(K
1
) = e
-rT

F(x) dx
K
1
K
2
1
# e
-rT
(K
2
- K
1
) F(K
1
) # e
-rT
(K
2
- K
1
) # K
2
- K
1
.

.
2
P
.K
2
= e
-rT
f(K) " 0.
Therefore, P(K) is concave upwards.
81
See Mahler!s Guide to Loss Distributions, covering material on Exam C.
This is an expression for the expected amount by which X is less than K.
2014-MFE, Financial Economics 3 Properties of Premiums, HCM 11/13/13, Page 71
Lee Diagrams and Put Premiums:
82

The expected payoff of a European put is E[(K - S
T
)
+
], which corresponds to Area P below the
horizontal line at height K and also above the curve graphing F(x) in the following Lee Diagram:

As K increases, the area below the horizontal line at height K increases; in other words, the value of
the put increases as K increases.
82
Not on the syllabus of your exam!
2014-MFE, Financial Economics 3 Properties of Premiums, HCM 11/13/13, Page 72
For an increase in K of $K, the value of the put increases by Area A in the following Lee Diagram:
1
Prob.
Stock Price
K+$K
K
A
The change in the value of the put, Area A, is smaller than a rectangle of height $K and width
F(K +$K). Thus Area A is smaller than $K F(K +$K) # $K. Thus a change of $K in the strike price
results in a change in the value of the put option smaller than $K.
The following Lee Diagram shows the effect of raising the strike price by fixed amounts:

The successive changes in the value of the put are represented by Areas A, B, C, and D.
We see that the changes in the value of the put get larger as the strike price increases.
2014-MFE, Financial Economics 3 Properties of Premiums, HCM 11/13/13, Page 73
Arbitrage:
If there is a possible combination of buying and selling with no net investment that has
no risk but generates positive (or at least nonnegative) cashflows, this is an arbitrage
opportunity. Taking advantage of such an opportunity is called arbitrage. In other words,
arbitrage is free money.
If an arbitrage opportunity existed, clever traders would take advantage of it. Relatively quickly, the
prices would adjust so as to remove this opportunity for arbitrage.
Generally, we assume prices should be such that they do not permit arbitrage. In other words, we
assume that there is no free lunch. This is called no-arbitrage pricing.
Arbitrage Opportunities with Two Calls:
If one of the general properties of option premiums that has been discussed is violated, that creates
an opportunity for arbitrage.
For example, two otherwise similar calls have the following premiums:
100 strike call costs 10
110 strike call costs 12.
This violates the principal that call premiums should not increase as the strike price increases.
The 100 strike call is cheap relative to the 110 strike call; the 110 strike call is expensive relative to
the 100 strike call.
We can buy a 100 strike call and sell a 110 strike call.
We make 12 - 10 = 2 from this set of transactions.
We invest the 2 at the risk free rate, r.
At expiration of the calls at time T, we have:
If S # 100: 2e
rT
> 0
If 110 " S > 100: (S - 100) + 2e
rT
> 0.
If S > 110: (S - 100) - (S - 110) + 2e
rT
= 10 + 2e
rT
> 0.
Thus we always end up with a positive (or at least nonnegative) position, having taken no risk.
This demonstrates arbitrage.
2014-MFE, Financial Economics 3 Properties of Premiums, HCM 11/13/13, Page 74
In another example, two otherwise similar calls have the following premiums:
100 strike call costs 20
105 strike call costs 13.
This violates the principal that call premiums should not decrease by more than the increase of the
strike price. The 105 strike call is cheap relative to the 100 strike call.
We can sell a 100 strike call and buy a 105 strike call.
We make 20 - 13 = 7 from this set of transactions.
We invest the 7 at the risk free rate, r.
At expiration of the calls at time T, we have:
If S # 100: 7e
rT
> 0
If 105 " S > 100: -(S - 100) + 7e
rT
" 7e
rT
- 5 > 0.
If S > 105: (S - 100) - (S - 105) + 7e
rT
= 7e
rT
- 5 > 0.
Thus we always end up with a positive (or at least nonnegative) position, having taken no risk.
This demonstrates arbitrage.
If the principals for put premiums are violated, one can set up similar opportunities for arbitrage to
those illustrated for calls.
Arbitrage Opportunities When Convexity is Violated by Call Premiums:
Three otherwise similar calls have the following premiums:
100 strike call costs 20
110 strike call costs 17.
140 strike call costs 7.
(20 - 17) / (110 - 100) = 0.3. (17 - 7) / (140 - 110) = 0.333.
0.333 > 0.3. This violates the convexity of the call premium with respect to the strike price.
The arbitrage opportunity in such situations involves buying some of the low strike and high strike
calls, while selling some of the medium strike calls. While there are many possible positions that
demonstrate arbitrage when convexity is violated, McDonald has a technique of coming up with one
such a portfolio.
2014-MFE, Financial Economics 3 Properties of Premiums, HCM 11/13/13, Page 75
Let + =

K
3
- K
2
K
3
- K
1
.
83

+ is the amount of the distance between the high and medium strike prices as a fraction of the
distance between the high and low strike prices. In this example, + = (140 - 110) / (140 -100) = 3/4.
We note that K
2
= + K
1
+ (1 - +)K
3
, a weighted average of K
1
and K
3
with weights + and 1 - +.
In this example, 110 = (3/4)(100) + (1 - 3/4)(140).
The convexity relationship for call premiums was:
For K
1
< K
2
< K
3
, {C(K
1
) - C(K
2
)} / {K
2
- K
1
} " {C(K
2
) - C(K
3
)} / {K
3
- K
2
}.
% {C(K
1
) - C(K
2
)}{K
3
- K
2
} " {C(K
2
) - C(K
3
)}{K
2
- K
1
}.
% (K
3
- K
2
)C(K
1
) + (K
2
- K
1
)C(K
3
) " (K
3
- K
1
)C(K
2
). % C(K
2
) # + C(K
1
) + (1 - +) C(K
3
).
For convexity to hold, we require C(K
2
) # + C(K
1
) + (1 - +) C(K
3
).
84

In this example, we require:
C(110) # (3/4) C(100) + (1 - 3/4) C(140) = (3/4)(20) + (1/4)(7) = 16.75.
The height of the line between the points (K
1
, C(K
1
)) and (K
3
, C(K
3
)), at the strike price K is:
y = 2C(K
1
) + (1 - 2)C(K
3
), where 2 =

K
3
- K
K
3
- K
1
.
Thus, we require that the point (K
2
, C(K
2
)) not be above this line.
83
See equation 9.21 in Derivatives Markets by McDonald.
It is somewhat arbitrary that McDonald has a numerator of K
3
- K
2
rather than K
2
- K
1
.
84
See equation 9.22 in Derivatives Markets by McDonald.
2014-MFE, Financial Economics 3 Properties of Premiums, HCM 11/13/13, Page 76
For this example, here is the line, as well as the premiums for the three calls:
100 110 140
K
7
17
20
C
Convexity is violated because the point (110, 17) is above the line.
85

In other words, the curve of option premium as a function of strike price is not concave upwards.
86
85
The point (110. 16.75) is on the line.
86
If concave upwards, the curve should be below any chord drawn between two points on the curve.
2014-MFE, Financial Economics 3 Properties of Premiums, HCM 11/13/13, Page 77
In order to demonstrate arbitrage when convexity is violated, in general, one can buy + of the
lowest strike call, buy 1 - + of the high strike call, and sell 1 of the medium strike call.
87

88

In this example, we buy 3/4 of the 100 strike calls, buy 1/4 of the 140 strike calls, and sell 1 of the
110 strike calls. Equivalently, we can buy 3 of the 100 strike calls, buy 1 of the 140 strike calls, and
sell 4 of the 110 strike calls.
When we set up this portfolio, we get: (-3)(20) + (4)(17) + (-1)(7) = 1.
We invest this 1 at the risk free rate r.
At expiration of the calls at time T, we have:
If S # 100: e
rT
> 0
If 110 " S > 100: 3(S - 100) + e
rT
> 0.
If 140 " S > 110: 3(S - 100) - (4)(S - 110) + e
rT
= 140 - S + e
rT
> 0.
If S > 140: 3(S - 100) - (4)(S - 110) + (S - 140) + e
rT
= e
rT
> 0.
Thus we always end up with a positive (or at least nonnegative) position, having taken no risk.
This demonstrates arbitrage.
89

Arbitrage Opportunities When Convexity is Violated by Put Premiums:
If convexity for put premiums are violated, one can set up similar opportunities for arbitrage.
For example, three otherwise similar puts have the following premiums:
80 strike put costs 12
100 strike put costs 18.
110 strike put costs 20.
(18 - 12) / (100 - 80) = 0.3. (20 - 18) / (110 - 100) = 0.2.
0.2 < 0.3. This violates the convexity of the put premium with respect to the strike price.
As with calls, the arbitrage opportunity in such situations involves buying some of the low strike and
high strike option, while selling some of the medium strike option. While there are many possible
positions that demonstrate arbitrage when convexity is violated, one can use the same technique of
coming up with one such a portfolio as was discussed for calls.
87
One could multiply all of the amounts by a constant in order to make them integer.
88
Since convexity is violated, C(K
2
) > + C(K
1
) + (1 -+) C(K
3
). Therefore, the medium strike call is overpriced relative
to this weighted average of the low strike and high strike calls. Therefore, we sell the medium strike call.
89
Recall, that there are other portfolios that would also demonstrate arbitrage.
2014-MFE, Financial Economics 3 Properties of Premiums, HCM 11/13/13, Page 78
In this example, + = (110 - 100) / (110 - 80) = 1/3.
Thus, we buy 1/3 of the 80 strike puts, buy 2/3 of the 110 strike puts,
and sell 1 of the 100 strike puts.
Equivalently, we can buy 1 of the 80 strike puts, buy 2 of the 110 strike puts,
and sell 3 of the 100 strike puts.
When we set up this portfolio, we get: (-1)(12) + (3)(18) + (-2)(20) = 2.
We invest this 2 at the risk free rate r. At time T we get back 2e
rT
.
At expiration of the puts at time T, we have:
If S " 110: 2e
rT
> 0
If 110 > S " 100: (2)(110 - S) + 2e
rT
> 0.
If 100 > S " 80: (2)(110 - S) - (3)(100 - S) + 2e
rT
= S - 80 + e
rT
> 0.
If S < 80: (2)(110 - S) - (3)(100 - S) + (80 - S) + 2e
rT
= 2e
rT
> 0.
Thus we always end up with a positive (or at least nonnegative) position, having taken no risk.
This demonstrates arbitrage.
2014-MFE, Financial Economics 3 Properties of Premiums, HCM 11/13/13, Page 79
Chart of Arbitrage Opportunities:
90

Strike Prices: K
1
< K
2
< K
3
. C is the premium for a Call, while P is the premium for a put.
Condition Arbitrage if the Condition is Violated
C(K
1
) " C(K
2
). Buy the K
1
Call and Sell the K
2
Call (Call Bull Spread)
C(K
1
) - C(K
2
) # K
2
- K
1
. Sell the K
1
Call and Buy the K
2
Call (Call Bear Spread)

C(K
1
) - C(K
2
)
K
2
- K
1
Buy + = (K
3
- K
2
)/(K
3
- K
1
) of K
1
,

Sell 1 of K
2
,

Buy 1 - + of K
3
.
91
"

C(K
2
) - C(K
3
)
K
3
- K
2
.
(Asymmetric butterfly spread)
P(K
2
) " P(K
1
). Sell the K
1
Put and Buy the K
2
Put (Put Bear Spread)
P(K
2
) - P(K
1
) # K
2
- K
1
. Buy the K
1
Put and Sell the K
2
Put (Put Bull Spread)

P(K
2
) - P(K
1
)
K
2
- K
1
Buy + = (K
3
- K
2
)/(K
3
- K
1
) of K
1
,

Sell 1 of K
2
,

Buy 1 - + of K
3
.
92

#

P(K
3
) - P(K
2
)
K
3
- K
2
.
(Asymmetric butterfly spread)
90
See page 283 of Derivatives Markets by McDonald.
91
There are other sets of amounts of each call that would also demonstrate arbitrage.
92
There are other sets of amounts of each put that would also demonstrate arbitrage.
2014-MFE, Financial Economics 3 Properties of Premiums, HCM 11/13/13, Page 80
Problems:
Use the following information for the next 4 questions:
The price of the stock of the Willy Wonka Chocolate Company 6 months from now has the
following distribution:
Price Probability
100 20%
150 40%
200 30%
250 10%
The continuously compounded annual rate of interest is 5%.
3.1 (1 point) Determine the expected payoff of a 6 month European call option on one share of
Willy Wonka Chocolate Company, with a strike price of 180.
A. 12 B. 13 C. 14 D. 15 E. 16
3.2 (1 point) Determine the expected payoff of a 6 month European put option on one share of
Willy Wonka Chocolate Company, with a strike price of 160.
A. 12 B. 13 C. 14 D. 15 E. 16
3.3 (1 point) Determine the actuarial present value of a 6 month European call option on one share
of Willy Wonka Chocolate Company, with a strike price of 170.
A. 16.0 B. 16.2 C. 16.4 D. 16.6 E. 16.8
3.4 (1 point) Determine the actuarial present value of a 6 month European put option on one share
of Willy Wonka Chocolate Company, with a strike price of 170.
A. 19.5 B. 20.0 C. 20.5 D. 21.0 E. 21.5
3.5 (1 point) 3 European put options on a stock are otherwise similar except for their strike price.
A put with a strike price of 150 has a premium of 30, in other words costs 30.
A put with a strike price of 160 has a premium of 34, in other words costs 34.
A put with a strike price of 180 has a premium of 40, in other words costs 40.
What general property of the value of puts is violated?
3.6 (3 points) Briefly describe an opportunity for arbitrage presented by the situation in the
previous question.
2014-MFE, Financial Economics 3 Properties of Premiums, HCM 11/13/13, Page 81
3.7 (1 point) Two European put options on a stock are otherwise similar except for their strike price.
A put with a strike price of 60 has a premium of 7.
A put with a strike price of 80 has a premium of 15.
Which of the following is true about the premium of a similar 70 strike put?
A. The smallest possible premium is 8
B. The smallest possible premium is 11
C. The largest possible premium is 8
D. The largest possible premium is 11
E. None of A, B, C, or D
3.8 (1 point) Two European call options on a stock are otherwise similar except for their strike price.
A call with a strike price of 100 has a premium of 27, in other words costs 27.
A call with a strike price of 110 has a premium of 15, in other words costs 15.
What general property of the value of calls is violated?
3.9 (2 points) Briefly describe an opportunity for arbitrage presented by the situation in the
previous question.
3.10 (1 point) Two European call options on a stock are otherwise similar except for their strike price.
A call with a strike price of 80 has a premium of 12.
A call with a strike price of 85 has a premium of 10.
Which of the following is true about the premium of a similar 100 strike call?
A. The smallest possible premium is 2
B. The smallest possible premium is 4
C. The largest possible premium is 2
D. The largest possible premium is 4
E. None of A, B, C, or D
3.11 (1 point) 2 European put options on a stock are otherwise similar except for their strike price.
A put with a strike price of 150 has a premium of 22, in other words costs 22.
A put with a strike price of 170 has a premium of 46, in other words costs 46.
What general property of the value of puts is violated?
3.12 (2 points) Briefly describe an opportunity for arbitrage presented by the situation in the
previous question.
2014-MFE, Financial Economics 3 Properties of Premiums, HCM 11/13/13, Page 82
3.13 (3 points) European call and put prices for options on a given stock are available as follows:
Strike Price Call Price Put Price
$50 $21 $3
$60 $16 $7
$80 $8 $14
All six options have the same expiration date, which is no more than two years from now.
After reviewing the information above, Danielle, Eric, and Felicia agree that arbitrage opportunities
arise from these prices.
Danielle believes that one could use the following portfolio to obtain arbitrage profit: Long two calls
with strike price 50; short four calls with strike price 60; long two calls with strike price 80; and either
lend or borrow some money.
Eric believes that one could use the following portfolio to obtain arbitrage profit: Long two puts with
strike price 50; short three puts with strike price 60; long one put with strike price 80; and either lend
or borrow some money.
Felicia believes that one could use the following portfolio to obtain arbitrage profit: Short one call with
strike price 50; long two calls with strike price 60; short one call with strike price 80; long one put with
strike price 50; short two puts with strike price 60; long one put with strike price 80; and either lend or
borrow some money.
Which of the following statements is true?
(A) Danielle and Eric are correct, while Felicia is not.
(B) Danielle and Felicia are correct, while Eric is not.
(C) Eric and Felicia are correct, while Danielle is not.
(D) All of them are correct.
(E) None of A, B, C, or D.
3.14 (1 point) 3 European call options on a stock are otherwise similar except for their strike price.
A call with a strike price of 100 has a premium of 25, in other words costs 25.
A call with a strike price of 110 has a premium of 20, in other words costs 20.
A call with a strike price of 115 has a premium of 16, in other words costs 16.
What general property of the value of calls is violated?
3.15 (3 points) Briefly describe an opportunity for arbitrage presented by the situation in the
previous question.
3.16 (1 point) Two European call options on a stock are otherwise similar except for their strike price.
A call with a strike price of 120 has a premium of 22, in other words costs 22.
A call with a strike price of 125 has a premium of 16, in other words costs 16.
What general property of the value of calls is violated?
3.17 (2 points) Briefly describe an opportunity for arbitrage presented by the situation in the
previous question.
2014-MFE, Financial Economics 3 Properties of Premiums, HCM 11/13/13, Page 83
3.18 (1 point) Two European call options on a stock are otherwise similar except for their strike price.
A call with a strike price of 80 has a premium of 12, in other words costs 12.
A call with a strike price of 90 has a premium of 14, in other words costs 14.
What general property of the value of calls is violated?
3.19 (2 points) Briefly describe an opportunity for arbitrage presented by the situation in the
previous question.
3.20 (2 points) Two European call options are otherwise similar except for their strike prices.
A call with a strike price of 85 has a premium of 13.
A call with a strike price of 90 has a premium of 7.
In order to take advantage of arbitrage, you buy 1000 of the 90 strike calls and sell x of the 85 strike
calls. If r = 0%, what is the smallest possible value of x?
A. 800 B. 825 C. 850 D. 875 E. 900
2014-MFE, Financial Economics 3 Properties of Premiums, HCM 11/13/13, Page 84
3.21 (2 points) A long range forward contract consists of buying a call, and selling a similar put with a
lower strike but the same time until expiration, where the strikes are chosen so that the contract has
no initial cost.
The current exchange rate is 1.58 Canadian Dollars per British Pound.
The company Bright, Light, and Powers is based in Canada. It will have to make a payment of
100,000 British Pounds in 4 months.
In order to limit its risk, the company buys a 4-month long range forward contract on 100,000 British
Pounds. The strike of the call is 1.65 Canadian Dollars. The strike of the put is 1.50 Canadian Dollars.
Graph as a function of the future exchange rate the amount of Canadian Dollars that Bright, Light, and
Powers has to spend 4 months from now in order to make its payment of 100,000 British Pounds.
3.22 (1 point)
The price of a stock one year from now has the following distribution in the risk neutral environment:
Price Probability
50 10%
100 20%
150 40%
200 20%
250 10%
Let X be the expected future value of a 1 year European call option on 100 shares of this stock with
a strike price of 170.
Let Y be the expected future value of a 1 year European put option on 100 shares of this stock with
a strike price of 140.
Determine Y - X.
A. 150 B. 200 C. 250 D. 300 E. 350
2014-MFE, Financial Economics 3 Properties of Premiums, HCM 11/13/13, Page 85
3.23 (MFE Sample Exam, Q.2) Near market closing time on a given day, you lose access to
stock prices, but some European call and put prices for a stock are available as follows:
Strike Price Call Price Put Price
$40 $11 $3
$50 $6 $8
$55 $3 $11
All six options have the same expiration date.
After reviewing the information above, John tells Mary and Peter that no arbitrage opportunities can
arise from these prices.
Mary disagrees with John. She argues that one could use the following portfolio to obtain arbitrage
profit: Long one call option with strike price 40; short three call options with strike price 50; lend $1;
and long some calls with strike price 55.
Peter also disagrees with John. He claims that the following portfolio, which is different from Mary!s,
can produce arbitrage profit: Long 2 calls and short 2 puts with strike price 55; long 1 call and short 1
put with strike price 40; lend $2; and short some calls and long the same number of puts with strike
price 50.
Which of the following statements is true?
(A) Only John is correct.
(B) Only Mary is correct.
(C) Only Peter is correct.
(D) Both Mary and Peter are correct.
(E) None of them is correct.
3.24 (IOA, CT8, 4/09, Q.1) (2.25 points) Describe what is meant by an arbitrage opportunity.
2014-MFE, Financial Economics 3 Properties of Premiums, HCM 11/13/13, Page 86
Solutions to Problems:
3.1. B. E[(S - 180)
+
] = (20%)(0) + (40%)(0) + (30%)(20) + (10%)(70) = 13.
Comment: If the future price is low, then the option to buy at 180 is worthless.
We have ignored the time value of money.
3.2. E. E[(160 - S)
+
] = (20%)(60) + (40%)(10) + (30%)(0) + (10%)(0) = 16.
Comment: If the future price is high, then the option to sell at 160 is worthless.
3.3. D. E[(S - 170)
+
] e
-rT
= {(20%)(0) + (40%)(0) + (30%)(30) + (10%)(80)} e
-(.05)(1/2)
= 16.58.
3.4. E. E[(170 - S)
+
] e
-rT
= {(20%)(70) + (40%)(20) + (30%)(0) + (10%)(0)} e
-(.05)(1/2)
= 21.46.
3.5. & 3.6. The absolute value of the changes in the value of the option over the absolute value of
the changes in the strike price are: (34 - 30)/(160 - 150) = 0.4, and (40 - 34)/(180 - 160) = 0.3.
0.4 > 0.3, which violates the proposition that the rate of change of the put option premium must
increase as the strike price rises. In other words, convexity is violated.
+ = (180 - 160)/(180 - 150) = 2/3. We can buy 2/3 puts with strike price 150, buy 1/3 puts with a
strike price of 180, and sell 1 put with strike price 160. Equivalently, we can buy 2 puts with strike
price 150, buy 1 put with a strike price of 180, and sell 3 puts with strike price 160.
Then you collect a net of: (3)(34) - (2)(30) - (1)(40) = 2.
You could invest this 2 at the risk free rate and have 2e
rT
at the expiration of the options.
If S
T
" 180, then all the puts turn out to worthless. You end up with 2e
rT
" 2.
If 180 > S
T
" 160, then the put that you have bought with strike price of 180 will be exercised.
You will make 180 - S
T
by exercising this put. You end up with: 2e
rT
+ (180 - S
T
) " 2.
If 160 > S
T
" 150, then the puts that you have sold will be exercised. You will pay (3)(160) = 480
for the 3 shares you buy from the person holding the puts. Then you can sell 1 share at 180 and 2
shares at S
T
.
You end up with: 2e
rT
+ 180 + 2S
T
- 480 = 2e
rT
+ 2S
T
- 300 " 2 + (2)(150) - 300 = 2.
If S
T
< 150, then you will pay (3)(160) = 480 for the 3 shares you buy from the person holding the
puts. Then you can sell 1 share at 180 and 2 shares at 150. You end up with:
2e
rT
+ 180 + 300 - 480 = 2e
rT
" 2.
In all possible situations you end up with a positive amount at time T, not having invested a positive
amount of money.
Comment: See Example 9.6 in Derivatives Markets by McDonald.
2014-MFE, Financial Economics 3 Properties of Premiums, HCM 11/13/13, Page 87
3.7. D. If the premiums were on a straight line, then the premium at a 70 strike would be 11.
For convexity to hold, the line between (60, 15) and (80, 7) must not be below the point
(70, premium). Therefore, the premium for a 70 strike must be 11 or less.
3.8. & 3.9. The absolute value of the change in the value of the option is: 27 -15 = 12, which is
greater than the absolute value of the change in the strike price: 110 - 100 = 10.
This should not occur.
You could buy a call with strike price 110 and sell a call with strike price 100.
Then you collect a net of: 27 -15 = 12. You could invest this 12 at the risk free rate and have 12e
rT
,
at the expiration of the call options.
If S
T
# 100, then both calls turn out to worthless. You end up with 12e
rT
> 12.
If 110 " S
T
> 100, then the call that you have sold will be exercised. You can buy a share at S
T
and
then will get 100 for the share when you sell it to the person holding the call you sold.
You end up with: 12e
rT
- (S
T
- 100) > 12 - 10 = 2.
If S
T
" 110, then the call that you have sold will be exercised, buying a share at 110. Then will get
100 for the share when you sell it to the person holding the call.
You end up with: 12e
rT
- 10 > 12 - 10 = 2.
In all possible situations you end up with a positive amount at time T, not having invested a positive
amount of money.
Comment: See Example 9.4 in Derivatives Markets by McDonald.
Arbitrage is a transaction where you always end up with a positive position, with no net investment
and no risk; in other words, arbitrage represents a free lunch. Arbitrage involves the simultaneous
buying and selling of related assets.
3.10. B. If the premiums were on a straight line, then the premium for a 100 strike would be 4.
For convexity to hold, the line between (80, 12) and (100, premium) must not be below the point
(85, 10). Therefore, the premium for a 100 strike must be 4 or more.
2014-MFE, Financial Economics 3 Properties of Premiums, HCM 11/13/13, Page 88
3.11. & 3.12. The absolute value of the change in the value of the option is: 46 - 22 = 24, which is
greater than the absolute value of the change in the strike price: 170 - 150 = 20.
This should not occur.
You could buy a put with strike price 150 and sell a put with strike price 170.
Then you collect a net of: 46 - 22 = 24. You could invest this 24 at the risk free rate and have 24e
rT
,
at the expiration of the options.
If S
T
" 170, then both puts turn out to worthless. You end up with 24e
rT
> 24.
If 170 > S
T
" 150, then the put that you have sold will be exercised. You buy a share at 170 from
the person who owns the put you sold, and then you sell the share for S
T
.
You end up with: 24e
rT
- (170 - S
T
) > 24 - 20 = 4.
If S
T
< 150, then the put that you have sold will be exercised. You buy a share at 170 from the
person who owns the put you sold, and then you sell the share for 150, using the put you own.
You end up with: 24e
rT
- (170 - 150) > 24 - 20 = 4.
In all possible situations you end up with a positive amount at time T, not having invested a positive
amount of money.
2014-MFE, Financial Economics 3 Properties of Premiums, HCM 11/13/13, Page 89
3.13. C. The call premiums satisfy all three properties, and thus there is no arbitrage opportunity.
If Danielle buys her portfolio of calls, then she would get: (4)(16) - (2)(21) - (2)(8) = 6.
She would lend out 6.
If for example, S
T
= 80, then the payoff from the calls is: (2)(30) + (-4)(20) + (2)(0) = -20.
Thus she would have 6e
rT
- 20, which is almost surely not positive.
Danielle!s portfolio does not demonstrate arbitrage.
The put premiums do not satisfy convexity, and thus there are opportunities for arbitrage.
If Eric buys his portfolio of puts, then he would get: (3)(7) - (2)(3) - (1)(14) = 1.
He would lend out 1.
If S
T
# 50, then the puts payoff: (2)(50 - S) - (3)(60 - S) + (1)(80 - S) = 0.
He ends up with e
rT
> 0.
If 50 < S
T
# 60, then the puts payoff: (-3)(60 - S) + (1)(80 - S) = 2S - 100 > 0.
He ends up with more than e
rT
> 0.
If 60 < S
T
# 80, then the puts payoff: (1)(80 - S) > 0.
He ends up with more than e
rT
> 0.
If 80 < S
T
, then all of the puts expire worthless. He ends up with e
rT
> 0.
Eric!s portfolio does demonstrate arbitrage.
If Felicia buys her portfolio of puts and calls, then she would spend:
(-1)(21) + (2)(16) + (-1)(8) + (1)(3) + (-2)(7) + (1)(14) = 6.
She would borrow 6.
If S
T
# 50, then the payoff is: (1)(50 - S) + (-2)(60 - S) + (1)(80 - S) = 10.
If 50 < S
T
# 60, then the payoff is: (-1)(S - 50) + (-2)(60 - S) + (1)(80 - S) = 10.
If 60 < S
T
# 80, then the payoff is: (-1)(S - 50) + (2)(S - 60) + (1)(80 - S) = 10.
If 80 < S
T
, then the payoff is: (-1)(S - 50) + (2)(S - 60) + (-1)(S - 80) = 10.
So she always ends up with 10 - 6e
rT
" 10 - 6e
r2
> 0.
Felicia!s portfolio does demonstrate arbitrage.
Comment: Similar to MFE Sample Exam, Q.2.
I have assumed that 10 > 6e
r2
. 3 r < 25.5%.
2014-MFE, Financial Economics 3 Properties of Premiums, HCM 11/13/13, Page 90
3.14. & 3.15. The absolute value of the changes in the value of the option over the absolute value
of the changes in the strike price are: (25 - 20)/(110 - 100) = 0.5, and (20 - 16)/(115 - 110) = 0.8.
0.5 < 0.8, which violates the proposition that the rate of absolute change of the call option premium
must decrease as the strike price rises. In other words, convexity is violated.
+ = (115 - 110)/(115 - 100) = 1/3. We can buy 1/3 calls with strike price 100, buy 2/3 calls with a
strike price of 115, and sell 1 call with strike price 110. Equivalently, we can buy 1 call with strike
price 100, buy 2 calls with a strike price of 115, and sell 3 calls with strike price 110.
Then you collect a net of: (3)(20) - (1)(25) - (2)(16) = 3.
You loan out this 3 and collect interest at the risk free rate.
At expiration of the calls at time T, we have:
If S # 100: 3e
rT
> 0
If 110 " S > 100: (S - 100) + 3e
rT
> 0.
If 115 " S > 110: (S - 100) - (3)(S - 110) + 3e
rT
= 230 - 2S + 3e
rT
> 0.
If S > 115: (S - 100) - (3)(S - 110) + (2)(S - 115) + 3e
rT
= 3e
rT
> 0.
Thus we always end up with a positive (or at least nonnegative) position, having taken no risk.
This demonstrates arbitrage.
Comment: See Example 9.5 in Derivatives Markets by McDonald.
The given call premiums as a function of strike price:
100 110 115
K
16
20
25
C
Since (110, 20) is above the line between (100, 25) and (115, 16), convexity is violated.
The line between (100, 25) and (115, 16) can be written as a weighted average of 25 and 16:
y = 25 2 + 16 (1 - 2), where 2 = (115 - K)/(115 - 100).
At K = K
2
= 110, 2 = 1/3 = (K
3
- K
2
)/(K
3
- K
1
) = +,
and the height of the line is: (25)(1/3) + (16)(2/3) = 19 < 20.
For convexity to hold, we require that: + C(K
1
) + (1 - +) C(K
3
) " C(K
2
).
There are other possible portfolios that would demonstrate arbitrage, but the one given in my
solution is based on how in the textbook arbitrage is demonstrated for this situation.
2014-MFE, Financial Economics 3 Properties of Premiums, HCM 11/13/13, Page 91
3.16. For K
1
# K
2
, C(K
1
) - C(K
2
) # K
2
- K
1
.
However, here 22 - 16 = 6 > 5 = 125 - 120.
The absolute change in call premium, should not be greater than the absolute change in the strike
prices.
3.17. Sell one 120 strike call and buy one 125 strike call. You get a net of: 22 - 16 = 6.
If the future stock price is less than 120, then both calls are worthless.
If the future stock price is between 120 and 125, then you lose money from the first call, while the
second call is worthless. Your position is worth: 6e
rT
- (S - 120) > 6 - 5 = 1.
If the future stock price is more than 125, then you lose money from the 120 strike call that you sold:
S - 120, while making money on the second call: S - 125.
Your position is worth: 6e
rT
- (S - 120) + (S - 125) > 6e
rT
- 5 > 1.
For an initial gain, you make money or break even, demonstrating arbitrage.
3.18. The premium for the call should not increase as the strike price increases.
3.19. Buy one 80 strike call and sell one 90 strike call. You get a net of: 14 - 12 = 2.
If the future stock price is less than 80, then both calls are worthless.
If the future stock price is between 80 and 90, then you make money from the first call, while the
second call is worthless.
If the future stock price is more than 90, then you make money from the first call: S - 80, while losing
money on the second call: S - 90. Overall, you make: (S - 80) - (S - 90) = 10.
For an initial gain, you make money or break even, demonstrating arbitrage.
2014-MFE, Financial Economics 3 Properties of Premiums, HCM 11/13/13, Page 92
3.20. D. If the future stock price is S
T
< 85, then both calls are worthless.
We want 13x - (1000)(7) " 0. % x > 538.
If the future stock price is 90 > S
T
> 85, then the 90 strike calls are worthless, but we lose money on
the 85 strike calls we sold. The worst case is when S
T
= 90 and we lose 5 on each 85 strike call.
We want 13x - (1000)(7) " 5x. % x " 875.
If the future stock price is S
T
> 90, then we make money on the 90 strike calls, and we lose money
on the 85 strike calls we sold.
We want 13x - (1000)(7) " x(S
T
- 85) - (1000)(S
T
- 90). % x(98 - S
T
) " (1000)(97 - S
T
).
If 98 - S
T
> 0, then we require that x " (1000)(97 - S
T
)/(98 - S
T
) = 1000{1 - 1/(98 - S
T
)}.
For 90 < S
T
< 98. The left hand side is largest for S
T
= 90; (1000)(97 - 90)/(98 - 90) = 875.
We need x " 875.
If 98 - S
T
< 0, then we require that x # (1000)(97 - S
T
)/(98 - S
T
) = 1000{1 + 1/(S
T
- 98)}.
Thus we need x # 1000.
We conclude that we can take 875 # x # 1000.
The smallest possible value of x is 875.
Comment: For example, if x = 900, then we get for setting up the portfolio: (900)(13) - (1000)(7) =
4700. If S
T
< 85 both calls are worthless. If S
T
= 90, we lose (5)(900) = 4500 on the 85 strike calls
we sold, but still come out ahead. If for example, S
T
= 100, we lose (15)(900) = 13,500 on the 85
strike calls we sold, and make (1000)(10) = 10,000 on the 90 strike calls we bought, for a net loss of
3500; but we still come out ahead due to the 4700 we got for setting up the portfolio.
2014-MFE, Financial Economics 3 Properties of Premiums, HCM 11/13/13, Page 93
3.21. If the exchange rate is greater than 1.65, it uses its call to buy 100,000 British Pounds for
165,000 Canadian Dollars. If the exchange rate is less than 1.50, the person who bought the put
uses it to sell 100,000 British Pounds to Bright, Light, and Powers for 150,000 Canadian Dollars.
For exchange rates in the middle, both options expire worthless, and Bright, Light, and Powers
pays the current exchange rate to buy 100,000 British Pounds.
1.4 1.5 1.6 1.7 1.8
rate
150000
155000
160000
165000
Amount
3.22. D. E[(S - 170)
+
] = (10%)(0) + (20%)(0) + (40%)(0) + (20%)(30) + (10%)(80) = 14.
E[(140 - S)
+
] = (10%)(90) + (20%)(40) + (40%)(0) + (20%)(0) + (10%)(0) = 17.
Y - X = 100 E[(140 - S)
+
] - 100 E[(S - 170)
+
] = 1700 - 1400 = 300.
2014-MFE, Financial Economics 3 Properties of Premiums, HCM 11/13/13, Page 94
3.23. D. Both Mary and Peter are correct; the prices are not arbitrage-free.
The call option prices do not satisfy their convexity condition:

11 - 6
50 - 40
= 0.5 >

6 - 3
55 - 50
= 0.6.
Mary buys one 40-strike calls; sells three 50-strike calls; lends $1; and buys some 55-strike calls.
+ =

55 - 50
55 - 40
= 1/3.
Buy + of K
1
. Sell 1 of K
2
. Buy 1 - + of K
3
.
So Mary can buy two of the 55-strike calls.
To buy and sell the calls brings in $1: (3) (6) - 11 - (2) (3) = 1.
Thus Mary does indeed lend out $1.
The prices are not arbitrage-free. To show that Mary!s portfolio yields arbitrage profit:
Time 0 Time T Time T Time T Time T
S
T
< 40 40 # S
T
< 50 50 # S
T
< 55 S
T
" 55
Buy 1 call -11 0 S
T
40 S
T
40 S
T
40
Strike 40
Sell 3 calls +18 0 0 -3(S
T
50) -3(S
T
50)
Strike 50
Lend $1 -1 e
rT
e
rT
e
rT
e
rT
Buy 2 calls -6 0 0 0 2(S
T
55)
Strike 55
Total 0 e
rT
> 0 e
rT
+ S
T
- 40 e
rT
+ 2(55 - S
T
) e
rT
> 0
The total at time 0 is zero, while in all cases, the total at time T is positive, proving arbitrage.
2014-MFE, Financial Economics 3 Properties of Premiums, HCM 11/13/13, Page 95
Strike Price Call Price Put Price
$40 $11 $3
$50 $6 $8
$55 $3 $11
Peter buys 1 40-strike call & sells 1 40-strike put: 11 - 3 = 8 money out the door.
Buys 2 55-strike calls & sells 2 55-strike puts: (2)(11) - (2)(3) = 16 money in the door.
Sells x 50-strike calls & buys x 50-strike puts: x8 - x6 = 2x money out the door.
We are told that Peter lends $2. % He must get $2 in the door net.
% 2 = 16 - 8 - 2x. % 2x = 6. % x = 3.
Let a position be the purchase of a K-strike call and sale of a K-strike put.
Peter buys one 40-strike position, sells three 50-strike positions, buys two 55-strike positions, and
lends $2.
The payoff on the purchase of a K-strike call and sale of a K-strike put is:
(S
T
- K)
+
- (K - S
T
)
+
=

S
T
- K if S
T
> K
-(K - S
T
) if S
T
< K
'
(
)
= S
T
- K.
At expiration, the payoff on his positions is:
S
T
- 40 - (3) (S
T
- 50) + (2) (S
T
- 55) = 0.
Thus at time T he has: 2 e
rT
> 0.
The total at time 0 is zero, while the total at time T is positive, proving arbitrage.
Alternately, Peter!s portfolio makes arbitrage profit, because:
Time-0 cash flow Time-T cash flow
Buy 1 call & sell 1 put Strike 40 -11 + 3 = -8 S
T
- 40
Sell 3 calls & buy 3 puts Strike 50 3(6 - 8) = -6 3(50 - S
T
)
Lend $2 -2 2e
rT
Buy 2 calls & sells 2 puts Strike 55 2(-3 + 11) = 16 2(S
T
- 55)
Total 0 2e
rT
The total at time 0 is zero, while the total at time T is positive, proving arbitrage.
Comment: See Table 9.7 in Derivatives Markets by McDonald.
The call option prices do not satisfy their convexity condition:
(11 - 6)/(50 -40) = 0.5, while (6 - 3)/(55 - 50) = 0.6 which is larger.
2014-MFE, Financial Economics 3 Properties of Premiums, HCM 11/13/13, Page 96
These call premiums as a function of strike price:
40 50 55
K
11
6
3
C
Since the point (50, 6) is above the line between (40, 11) and (55, 3), convexity is violated.
In other words, the curve of option premium versus strike price is not concave upwards.
The line between (40, 11) and (55, 3) can be written as a weighted average of 11 and 3:
y = 11 2 + 3 (1 - 2), where 2 =

55 - K
55 - 40
.
At K = K
2
= 50, 2 = 1/3 =

K
3
- K
2
K
3
- K
1
= +, and the height of the line is: (11)(1/3) + (3)(2/3) = 5.67 < 6.
For convexity to hold, we require that: + C(K
1
) + (1 - +) C(K
3
) " C(K
2
).
2014-MFE, Financial Economics 3 Properties of Premiums, HCM 11/13/13, Page 97
In contrast, the put option prices do satisfy their convexity condition:

8 - 3
50 - 40
= 0.5 >

11 - 8
55 - 50
= 0.6 which is larger.
These put premiums as a function of strike price:
40 50 55
K
11
8
3
P
Since the point (50, 8) is below the line between (40, 3) and (55, 11), convexity is satisfied.
In other words, the curve of option premium versus strike price is concave upwards.
Long 3 Buy. Short 3 Sell.
The payoff on Peter!s combination of positions is always zero; thus its correct price is 0.
Since its price is not 0, there is an opportunity for arbitrage.
Depending on the sign of this price, either we would buy and sell Peter!s combination of positions,
or we would do the exact opposite of what Peter did.
2014-MFE, Financial Economics 3 Properties of Premiums, HCM 11/13/13, Page 98
3.24. Put in simple terms, an arbitrage opportunity is a situation where we can make a sure profit
with no risk. This is sometimes described as a free lunch.
Put more precisely an arbitrage opportunity means that:
(a) We can start at time 0 with a portfolio which has a net value of zero (implying that we are long in
some assets and short in others).
(b) At some future time T:
the probability of a loss is 0.
the probability that we make a strictly positive profit is greater than 0.
2014-MFE, Financial Economics 3 Properties of Premiums, HCM 11/13/13, Page 99
Section 4, Put-Call Parity
The value of an otherwise similar call and put option on the same asset are related.
Adam and Eve Example:
XYZ stock pays no dividends.
Adam buys a 2 year call on XYZ stock with strike price $100.
Adam also loans out $100 e
-2r
at the risk free rate.
In two years, Adam will have $100.
If XYZ stock has a price > $100, Adam will use $100 and his call to buy a share of XYZ.
Otherwise, Adam keeps his $100.
Two years from now, Adam ends up with a share of XYZ or $100, whichever is worth more.
Eve buys a 2 year put on XYZ stock with strike price $100 from Seth.
Eve also buys a share of XYZ stock.
In two years, if XYZ stock has a price < $100, Eve will use her put to sell a share of XYZ stock to
Seth for $100. Otherwise, Eve does not exercise her put.
Two years from now, Eve ends up with a share of XYZ or $100, whichever is worth more.
Future Stock Price Adam Eve
< $100 $100 $100
> $100 Stock Stock
Two years from now Adam and Eve have the same position.
Therefore, Adam and Eve!s initial positions must have the same price.
Call + K e
-rT
= Put + Stock.
If the stock had paid dividends, then Eve would have collected them, while Adam will not. If we
subtract the present value of these dividends, then their two positions would still be equal.
PV[F
0,T
] = S
0
- PV[Div].
Setting equal the values of Adam!s position and Eve!s position minus any dividends we have:
Call + K e
-rT
= Put + PV[F
0,T
].
2014-MFE, Financial Economics 4 Put-Call Parity, HCM 11/13/13, Page 100
Put-Call Parity formula:
Thus we have the following put-call parity formula:
93
C
Eur
(K, T) = P
Eur
(K, T) + PV[F
0 , T
] - PV[K].
Exercise: In the absence of stock dividends, determine the future value of the sum of:
(a) A share of stock.
(b) A European put option on that stock with a strike price of K.
(c) Having sold a call option on that stock with a strike price of K and the same expiration as the put.
[Solution: Future value is: S
T
+ (K - S
T
)
+
- (S
T
- K)
+
.
If S # K, then this future value is: S
T
+ K - S
T
- 0 = K.
If S " K, then this future value is: S
T
+ 0 - (S
T
- K) = K.
Comment: (X-d)
+
- (d-X)
+
= X - d. % X + (d-X)
+
- (X-d)
+
= d.]
Thus we again have put-call parity. As in the Adam and Eve example, in the absence of dividends:
Share of Stock at Time T + Put - Call = K.
3 Value of Call at Time T - Value of Put at Time T = Value of Stock at Time T - Strike Price.

Stocks With Discrete Dividends:
If dividends are paid at discrete times, then PV[F
0,T
] = S
0
- PV[Div].
94
C
Eur
(K, T) = P
Eur
(K, T) + S
0
- PV[Div] - K e
-rT
.
95

Stocks With Continuous Dividends:
If dividends are paid continuously, then PV[F
0,T
] = S
0
e
-#T
.
C
Eur
(K, T) = P
Eur
(K, T) + S
0
e
-#T
- K e
-rT
.
96

93
See Equation 3.1 in Derivatives Markets by McDonald.
94
F
0,T
, the future value of the share of stock, is the expected value of owning a share at time T, excluding any
dividends that may have been paid from time 0 to time T.
95
See equation 9.2 in Derivatives Markets by McDonald.
96
See below equation 9.2 in Derivatives Markets by McDonald. In the case of no dividends, # = 0.
2014-MFE, Financial Economics 4 Put-Call Parity, HCM 11/13/13, Page 101
Exercise: S
0
= $100, r = 6%, # = 2%, T = 1/2.
The present value of a European put option with a strike price of $120 is $30.
Determine the present value of a European call option with the same strike price of $120.
[Solution: $30 + $100 e
-0.01
- $120 e
-0.03
= $12.55.]
Thus given r, #, S
0
, K and T, if we know the value of either the put or call, then we know the value of
the other one.
Exercise: S
0
= $100, r = 6%, # = 2%, T = 1/2.
The present value of a European call option with a strike price of $110 is $15.
Determine the present value of a European put option with the same strike price of $110.
[Solution: $15 - $100 e
-0.01
+ $110 e
-0.03
= $22.74.]
Exercise: On a stock that pays continuous dividends, at what strike price are a European put and call
worth the same?
[Solution: 0 = C
Eur
(K, T) - P
Eur
(K, T) = S
0
e
-#T
- K e
-rT
. % K = S
0
e
(r-#)T
.
Comment: In a risk-neutral environment, the expected stock price at time T is: S
0
e
(r-#)T
.]
Strike Price Equal to Current Stock Price:
97

If the stock pays no dividends, then C
Eur
(K, T) - P
Eur
(K, T) = S
0
- K e
-rT
.
Let us assume S
0
= K = $100. Then C - P = $100(1 - e
-rT
).
Let us assume we have $100 today available to spend.
If we buy the stock now for $100, we can hold it until time T.
If we instead buy the call and sell the put, then this will cost $100(1 - e
-rT
).
Subtracting this from our $100, we would have $100 e
-rT
left.
Assuming we invest $100 e
-rT
at the risk free rate, when time T comes we would have $100.
At time T if S
T
" 100, we can buy a share for 100 using our call.
At time T if S
T
< 100, then we buy a share for 100 from the person who bought our put.
In both situations, we end up with the stock and have invested $100 at time 0 for it.
By buying the call and selling the put, we have deferred the payment of $100 until T.
This deferral costs interest on the $100 of: $100(1 - e
-rT
).
Thus the option premiums differ by interest on the deferral of the payment for the stock.
97
See Example 9.1 in Derivatives Markets by McDonald. When S = K, the option is said to be at the money.
2014-MFE, Financial Economics 4 Put-Call Parity, HCM 11/13/13, Page 102
Derivation of Put-Call Parity:
The payoff on a call is (S - K)
+
.
The payoff on a put is (K - S)
+
.
If one buys a call and sells a similar put, the payoff is: (S - K)
+
- (K - S)
+
= S - K.
Therefore, taking the prepaid forward prices, in other words the actuarial present values:
C - P =

0,T
P
F [S
T
] -

0,T
P
F [K] = PV[F
0,T
] - PV[K].
Bonds:
A bond pays coupons to its owner. These coupon payments act mathematically like stock
dividends paid at discrete times. Therefore, if B
0
is the current price of the bond:
98
C
Eur
(K, T) = P
Eur
(K, T) + B
0
- PV[Coupons] - K e
-rT
.
Summary:
99

As will be discussed in subsequent sections, there are similar relationships for other assets.
Asset Parity Relationship
Stock, No Dividends S
0
= C - P + e
-rT
K
Stock, Discrete Dividends S
0
- PV[Div] = C - P + e
-rT
K
Stock, Continuous Dividends e
-#T
S
0
= C - P + e
-rT
K
Bond B
0
- PV[Coupons] = C - P + e
-rT
K.
Futures Contract e
-rT
F
0,T
= C - P + e
-rT
K
Currency exp[-r
f
T] x
0
= C - P + e
-rT
K
Exchange Options

F
0,T
P
(S
0
) = C - P +

F
0,T
P
(Q
0
) 3
exp[-#
S
T] S
0
= C - P + exp[-#
Q
T] Q
0
98
While listed by McDonald, he does not discuss further this formula involving bonds.
In this formula, e
-rT
is the price of a zero-coupon bond that pays 1 at time T.
99
See Table 9.9 at page 305 of Derivatives Markets by McDonald.
2014-MFE, Financial Economics 4 Put-Call Parity, HCM 11/13/13, Page 103
Dividend Forward Contract
:
100

Put-call parity states that:
C
Eur
(K, T) = P
Eur
(K, T) + S
0
- PV[Div] - K e
-rT
.
Therefore, the present value of future dividends paid from time 0 to T is:
101

PV[Div] = S
0
- K e
-rT
+ P
Eur
(K, T) - C
Eur
(K, T).
Therefore, the forward price of future dividends paid from time 0 to T is:
102
F
0,T
[Div] = e
rT
PV[Div] = e
rT
{S
0
+ P
Eur
(K, T) - C
Eur
(K, T)} - K.
Exercise: A 90-strike 2-year European call on ABC stock costs $19.42.
A 90-strike 2-year European put on ABC stock costs $9.96.
The current price of ABC stock is $92. r = 6%.
Determine the forward price of future dividends paid over the next two years by ABC stock.
[Solution: exp[(6%)(2)] (92 + 9.96 - 19.42) - 90 = $3.06.]
Usually, we treat future dividends as a known quantity, either as a dollar amount or a percent of the
stock price. However, the magnitude of future dividends are uncertain, particularly over longer
periods of time.
One could buy a dividend forward contract in order to hedge against or speculate on the amount of
future dividends. The payoff to the buyer of such a contract would be the difference between the
accumulated value of the dividends paid between time 0 and T and the dividend forward price:
FV
0,T
(D) - F
0,T
(D).
For example, the payoff for a 2 year forward contract on dividends on ABC stock would be:
FV
0,2
(D) - $3.06.
If for example it turns out that there are dividend payments of $1.70 each at times 1 and 2, then
FV
0,2
(D) = e
0.06
1.7 + 1.7 = $3.51. The buyer of the dividend forward contract would be paid by
the seller at time 2: $3.51 - $3.06 = $0.45.
If instead it turns out that there are dividend payments of $1.20 at time 0.5 and $1.40 at time 1.5,
then FV
0,2
(D) = e
0.09
1.2 + e
0.03
1.4 = $2.76. The buyer of the dividend forward contract would
be paid at time 2: $2.76 - $3.06 = -$0.30; the buyer would pay the seller $0.30.
100
See page 269-270 of Derivative Markets by McDonald.
101
See equation 9.6 in Derivative Markets by McDonald.
102
See equation 9.7 in Derivative Markets by McDonald.
2014-MFE, Financial Economics 4 Put-Call Parity, HCM 11/13/13, Page 104
Problems:
4.1 (2 points) Jay Corp. common stock is priced at $80 per share.
The company just paid its $1 quarterly dividend. Continuously compounded Interest rate is 6.0%.
A $75 strike European call, maturing in 8 months, sells for $10.
What is the price of a 8-month, $75 strike European put option?
A. 3 B. 4 C. 5 D. 6 E. 7
4.2 (2 points) A bond currently costs $830. The bond has $10 quarterly coupons.
A coupon has just been paid. r = 4%. What is the difference in price between a 2-year $800 strike
European call option and the corresponding put?
A. 15 B. 20 C. 25 D. 30 E. 35
Use the following information for the next two questions:
The Rich and Fine stock index is priced at 1300. Dividends are paid at the rate of 2%.
The continuously compounded risk free rate is 5%.
4.3 (1 point) A 1800 strike European call, maturing in 4 years, sells for 196.
What is the price of a 4-year, 1800 strike European put option?
A. Less than 400
B. At least 400, but less than 450
C. At least 450, but less than 500
D. At least 500, but less than 550
E. At least 550
4.4 (1 point) A 1500 strike European put, expiring in 3 years, sells for 293.
What is the price of a 3-year, 1500 strike European call option?
A. 150 B. 175 C. 200 D. 225 E. 250
4.5 (2 points) For a stock that does not pay dividends, the difference in price between otherwise
similar European call and put options is 29.05. The current stock price is 100. r = 6%.
The time until expiration is 2 years. Determine the strike price.
A. 80 B. 85 C. 90 D. 95 E. 100
4.6 (2 points) 160 = current market price of the stock
130 = strike price of the option
5%

= annual risk free force of interest
18 months = time until the exercise date of the option
3% = (continuously compounded) annual rate at which dividends are paid on this stock
If a European put has an option premium of 13, determine the premium for a similar European call.
A. 35 B. 40 C. 45 D. 50 E. 55
2014-MFE, Financial Economics 4 Put-Call Parity, HCM 11/13/13, Page 105
Use the following information for the next four questions:
You have the following bid and ask prices on 3.25 month European options on HAL stock with a
strike price of $160.
Call Bid Call Ask Put Bid Put Ask
$13.00 $13.40 $2.90 $3.20
You buy at the ask and sell at the bid.
Your continuously compounded lending rate is 1.9% and your borrowing rate is 2%.
The current price of HAL stock is $169.70.
Ignore transaction costs on the stock.
HAL will pay a $0.36 dividend one month from today.
4.7 (2 points) You sell the call, buy the put, buy the stock, and borrow the present value of the
strike price plus dividend. What is the cost?
4.8 (2 points) Briefly discuss whether parity is violated by the situation in the previous question.
4.9 (2 points) What is the cost if you buy the call, sell the put, short the stock, and lend the present
value of the strike price plus dividend?
4.10 (2 points) Briefly discuss whether parity is violated by the situation in the previous question.
4.11 (2 points) The price of a non-dividend paying stock is $85 per share. A 18 month, at the
money European put option is trading for $5. If the interest rate is 6.5%, what is the price of a
European call at the same strike and expiration?
A. 12 B. 13 C. 14 D. 15 E. 16
4.12 (2 points) As a function of the future stock price, graph the future value of the sum of a share of
the stock and a European put option on that same stock with a strike price of 100.
4.13 (2 points) A stock that pays continuous dividends at 1.5%, has a price of 130. r = 4%.
At what strike price are a 2 year European put and call worth the same?
A. Less than 126
B. At least 126, but less than 128
C. At least 128, but less than 132
D. At least 132, but less than 136
E. At least 136
2014-MFE, Financial Economics 4 Put-Call Parity, HCM 11/13/13, Page 106
4.14 (2 points) The prices for otherwise similar European puts and calls are:
Strike 100 120
Call premium 27 23
Put premium 25 26
Describe a spread position involving puts and calls that you can use to affect arbitrage, and
demonstrate that arbitrage occurs.
4.15 (2 points) Kay Corp. common stock is priced at $120 per share.
Yesterday, the company paid its $2 quarterly dividend.
The continuously compounded interest rate is 5.0%.
The company just paid its $2 quarterly dividend. Interest rates are 5.0%.
A $130 strike European put, maturing in 4 months, sells for $15.
What is the price of a 4-month, $130 strike European call option?
A. 3 B. 4 C. 5 D. 6 E. 7
4.16 (3 points) You are given the following premiums for European options:
Strike Price Call Price Put Price
$60 $3.45
$80 $23.85 $9.88
$90 $19.56 $14.38
$110 $13.09
All of the options are on the same stock and have the same expiration date.
Determine the sum of the price of the 60 strike call plus the price of the 110 strike put.
A. 59.0 B. 59.5 C. 60.0 D. 60.5 E. 61.0
4.17 (2 points) Joe buys a share of a stock, buys a European put option on that stock with a strike
price of 100, and sells a European call option on that stock with a strike price of 100. Graph the
future value of Joe!s investments as a function of the stock price at expiration of the options.

4.18 (2 points) 150 = current market price of the stock
140 = strike price of the option
6%

= annual risk free force of interest
6 months = time until the exercise date of the option
3% = (continuously compounded) annual rate at which dividends are paid on this stock
If a European call has an option premium of 23, determine the premium for a similar European put.
A. 7 B. 8 C. 9 D. 10 E. 11
4.19 (2 points) A European put and call on the same stock have the same time until maturity and the
same strike price. S
0
= 100. K = 115. r = 6%. # = 2%.
If the put and the call have the same premium, what is the time until maturity?
A. 1.5 years B. 2.0 years C. 2.5 years D. 3.0 years E. 3.5 years
2014-MFE, Financial Economics 4 Put-Call Parity, HCM 11/13/13, Page 107
4.20 (2 points) A stock is priced at $100 per share.
The stock is forecasted to pay dividends of $0.80, $1.20, and $1.50 in 3, 6, and 9 months,
respectively. r = 5.5%.
A $125 strike European call, maturing in 9 months, sells for $8.
What is the price of a 9-month, $125 strike European put option?
A. Less than 25
B. At least 25, but less than 30
C. At least 30, but less than 35
D. At least 35, but less than 40
E. At least 40
4.21 (2 points) Consider 6 month 80 strike European options on a nondividend paying stock.
Two months after being purchased, the call and the put would have the same value for a stock price
of 78.68. Determine r.
A. 4.0% B. 4.5% C. 5.0% D. 5.5% E. 6.0%
4.22 (2 points) Near market closing time on a given day, you lose access to stock prices, but some
European call and put prices for a stock are available as follows:
Strike Price Call Price Put Price
$70 $11 $3
$80 $6 $7
$90 $3
All of the options have the same expiration date.
Determine the price of the 90 strike put.
A. 12.0 B. 12.5 C. 13.0 D. 13.5 E. 14.0
4.23 (2 points) A one year European put and call on the same stock with the same strike price have
the same premium.
The current stock price is 84. The stock will pay dividends of 2, one month from now, four months
from now, seven months from now, and ten months from now.
If r = 5%, what is the strike price?
A. 75 B. 80 C. 85 D. 90 E. 95
4.24 (2 points) A two-year Call Bull Spread has strike prices that differ by 10.
The premium for this Call Bull Spread is 4.19.
r = 5.6%.
Determine the premium for a similar Put Bear Spread.
A. 3.75 B. 4.00 C. 4.25 D. 4.50 E. 4.75
2014-MFE, Financial Economics 4 Put-Call Parity, HCM 11/13/13, Page 108
4.25 (2 points) You are given the following information on European options on a given stock index
that does not pay dividends.
All the options have the same strike price and all expire on July 1, 2008.
As of January 1, 2008 As of March 1, 2008
Stock Index Price 860 1020
Call Premium 66.67 154.64
Put Premium 86.64 21.24
Determine the continuously compounded risk rate of interest, r.
A. 4.5% B. 5.0% C. 5.5% D. 6.0% E. 6.5%
4.26 (2 points) A stock is priced at $93 per share.
The stock pays dividends at a continuously compounded rate #.
r = 5.2%.
The premium for a $90-strike 2-year European call is $18.50.
The premium for a $90-strike 2-year European put is $9.36.
Determine #.
A. 0.5% B. 1.0% C. 1.5% D. 2.0% E. 2.5%
4.27 (3 points) You observe the prices of various European call and put options all on the same
stock and all with the same expiration date:
Strike Price Call Price Put Price
$60 $21 $6
$70 $17 $11
$90 $12 $25
After reviewing the information above, Andrew tells Brittany and Christopher that no arbitrage
opportunities can arise from these prices.
Brittany disagrees with Andrew. She argues that one could use a portfolio of various calls to obtain
arbitrage profit.
Christopher also disagrees with Andrew. He claims that one could use a portfolio of various puts to
obtain arbitrage profit.
Which of the following statements is true?
(A) Only Andrew is correct.
(B) Only Brittany is correct.
(C) Only Christopher is correct.
(D) Both Brittany and Christopher are correct.
(E) None of them is correct.
2014-MFE, Financial Economics 4 Put-Call Parity, HCM 11/13/13, Page 109
4.28 (3 points) A nondividend paying stock has a current price of 76.
You observe the prices of various 2-year European options on this stock:
70-strike call has a premium of 19.77.
80-strike put has a premium of 8.61.
100-strike call has a premium of 7.93.
Determine the possible values of r.
4.29 (3 points) Consider European options all on the same stock and all with the same time until
expiration.
Dustin buys a 90-strike call and sells a 100-strike put. His net cost is 24.47.
Edith buys a 100-strike call and sells a 90-strike put. Her net cost is 16.85.
Frank buys a 120-strike call and sells a 120-strike put. His net cost is 1.61.
Gabrielle buys a 110-strike call and sells a 110-strike put.
What is Gabrielle!s net cost?
A. 8.4 B. 8.6 C. 8.8 D. 9.0 E. 9.2
4.30 (2 points) Consider European options all on the same stock and all with the same time until
expiration.
Ashley buys a 60-strike call and sells an 80-strike put. Her net cost is 1.22.
Chelsea buys an 80-strike call and sells a 60-strike put. Her net cost is 4.52.
Benjamin buys an 70-strike call and sells a 70-strike put.
What is Benjamin!s net cost?
A. 2.8 B. 2.9 C. 3.0 D. 3.1 E. 3.2
4.31 (2 points) A stock pays dividends at a continuously compounded rate of 2%.
The current price of the stock is 82.
The effective annual risk-free interest rate is 10%.
You buy a three-year 80-strike European call option on the stock and sell a similar put.
Determine the premium for this position.
(A) 16 (B) 17 (C) 18 (D) 19 (E) 20
4.32 (3 points)
Consider European options all on the same stock and all with the same time until expiration.
Let C(K) be the premium for a call with strike K.
Let P(K) be the premium for a put with strike K.
Let A(K) = C(K) - C(K + 10).
Let B(K) = P(K + 10) - P(K).
A(95) = 3.49. B(95) = 5.36. B(80) = 3.87.
Determine A(80).
A. 4.7 B. 4.8 C. 4.9 D. 5.0 E. 5.1
2014-MFE, Financial Economics 4 Put-Call Parity, HCM 11/13/13, Page 110
4.33 (CAS5B, 11/92, Q.64) (1 point) For a certain non-dividend paying stock, the price of a put
option with a strike price of $100 exercisable in one year is $25.
The current stock price is $120. The cost to borrow money is 10% for one year.
What is the price of a call with an strike price of $100 exercisable in one year?
A. Less than $50
B. At least $50, but less than $55
C. At least $55, but less than $60
D. At least $60, but less than $65
E. $65 or greater
4.34 (CAS5B, 11/94, Q.29) (4.5 points) An investor would like to sell short ABC company's
stock, currently valued at $100, and will close out this position in one year.
a. (1 point) What does it mean "to sell short ABC!s stock"?
b. (1 point) In a graph, depict the cumulative payoff in one year to this investor, relative to ABC's
stock price. Ignore interest and transaction costs. Assume ABC pays no dividends.
For (c) and (d) below, assume the investor does not want to lose more than $100 on this transaction
when she closes out her position at the end of the year. Further assume a European options market
exists for ABC stock. Assume ABC pays no dividends.
c. (1.5 points) Construct an option that would achieve this goal, ignoring the cost of the option itself,
interest charges, and any related transaction costs. Draw two graphs: the first to show the payoff of
this option, and the second to show the investor's net payoff after implementing this strategy.
d. (1 point) Given the following table, what is the expected cost of this "insurance"?
Assume a continuously compounded risk-free interest rate of 10% per year, and ignore transaction
costs and potential broker margin calls.
Term: One Year
Strike Price Value of Put
50 5
75 10
100 15
125 30
150 50
175 72
200 95
2014-MFE, Financial Economics 4 Put-Call Parity, HCM 11/13/13, Page 111
4.35 (CAS5B, 5/99, Q.1) (1 point) You sell a 1-year European call option on Greystokes Inc.
with an strike price of 110 and buy a 1-year European put option with the same strike price and
term.
The current continuously compounded risk-free rate is 12% and the value of your combined position
is zero.
Greystokes Inc. is a non-dividend-paying stock. What is the price of a share of Greystokes Inc.?
A. Less than 95.00
B. At least 95.00, but less than 97.50
C. At least 97.50, but less than 100.00
D. At least 100.00, but less than 102.50
E. At least 102.50
4.36 (IOA 109, 9/00, Q.1) (8.25 points)
(i) (1.5 points) State what is meant by put-call parity.
(ii) (4.5 points) Derive an expression for the put-call parity of a European option that has a dividend
payable prior to the exercise date.
(iii) (2.25 points) If the equality in (ii) does not hold, explain how an arbitrageur can make a riskless
profit.
4.37 (MFE Sample Exam, Q.1) Consider a European call option and a European put option on a
nondividend-paying stock. You are given:
(i) The current price of the stock is $60.
(ii) The call option currently sells for $0.15 more than the put option.
(iii) Both the call option and put option will expire in 4 years.
(iv) Both the call option and put option have a strike price of $70.
Calculate the continuously compounded risk-free interest rate.
(A) 0.039 (B) 0.049 (C) 0.059 (D) 0.069 (E) 0.079
4.38 (FM Sample Exam, Q.1) Which statement about zero-cost purchased collars is FALSE?
A. A zero-width, zero-cost collar can be created by setting both the put and call strike prices
at the forward price.
B. There are an infinite number of zero-cost collars.
C. The put option can be at-the-money.
D. The call option can be at-the-money.
E. The strike price on the put option must be at or below the forward price.
2014-MFE, Financial Economics 4 Put-Call Parity, HCM 11/13/13, Page 112
4.39 (FM Sample Exam, Q.2) You are given the following information:
The current price to buy one share of XYZ stock is 500.
The stock does not pay dividends.
The risk-free interest rate, compounded continuously, is 6%.
A European call option on one share of XYZ stock with a strike price of K that expires in
one year costs 66.59.
A European put option on one share of XYZ stock with a strike price of K that expires in
one year costs 18.64.
Using put-call parity, determine the strike price, K.
A. 449 B. 452 C. 480 D. 559 E. 582
4.40 (FM Sample Exam, Q.5) You are given the following information:
One share of the PS index currently sells for 1,000.
The PS index does not pay dividends.
The effective annual risk-free interest rate is 5%.
You want to lock in the ability to buy this index in one year for a price of 1,025. You can do this by
buying or selling European put and call options with a strike price of 1,025.
Which of the following will achieve your objective and also gives the cost today of establishing this
position?
A. Buy the put and sell the call, receive 23.81
B. Buy the put and sell the call, spend 23.81
C. Buy the put and sell the call, no cost
D. Buy the call and sell the put, receive 23.81
E. Buy the call and sell the put, spend 23.81
2014-MFE, Financial Economics 4 Put-Call Parity, HCM 11/13/13, Page 113
4.41 (CAS3, 5/07, Q.3) (2.5 points) For a dividend paying stock and European options on this
stock, you are given the following information:
The current stock price is $49.70.
The strike price of options is $50.00.
The time to expiration is 6 months.
The continuous risk-free rate is 3% annually.
The continuous dividend yield is 2% annually.
The call price is $2.00.
The put price is $2.35.
Using put-call parity, calculate the present value arbitrage profit per share that could be generated,
given these conditions.
A. Less than $0.20
B. At least $0.20 but less than $0.40
C. At least $0.40 but less than $0.60
D. At least $0.60 but less than $0.80
E. At least $0.80
4.42 (CAS3, 5/07, Q.4) (2.5 points)
The price of a European call that expires in six months and has a strike price of $30 is $2.
The underlying stock price is $29, and a dividend of $0.50 is expected in two months and in five
months. The term structure is flat, with all continuously compounded, risk-free rates being 10%.
Calculate the price of a European put option on the same stock with expiration in six months and a
strike price of $30.
A. 0.57 B. 1.06 C. 1.54 D. 2.02 E. 2.51
4.43 (MFE, 5/07, Q.1) (2.6 points) On April 30, 2007, a common stock is priced at $52.00.
You are given the following:
(i) Dividends of equal amounts will be paid on June 30, 2007 and September 30, 2007.
(ii) A European call option on the stock with strike price of $50.00 expiring in six months
sells for $4.50.
(iii) A European put option on the stock with strike price of $50.00 expiring in six months
sells for $2.45.
(iv) The continuously compounded risk-free interest rate is 6%.
Calculate the amount of each dividend.
(A) $0.51 (B) $0.73 (C) $1.01 (D) $1.23 (E) $1.45
2014-MFE, Financial Economics 4 Put-Call Parity, HCM 11/13/13, Page 114
4.44 (MFE, 5/07, Q.4) (2.6 points) For a stock, you are given:
(i) The current stock price is $50.00.
(ii) # = 0.08
(iii) The continuously compounded risk-free interest rate is r = 0.04.
(iv) The prices for one-year European calls (C) under various strike prices (K) are shown below:
K C
$40 $ 9.12
$50 $ 4.91
$60 $ 0.71
$70 $ 0.00
You own four special put options each with one of the strike prices listed in (iv).
Each of these put options can only be exercised immediately or one year from now.
Determine the lowest strike price for which it is optimal to exercise these special put option(s)
immediately.
(A) $40 (B) $50 (C) $60 (D) $70
(E) It is not optimal to exercise any of these put options.
4.45 (CAS3, 11/07, Q.14) (2.5 points)
Given the following information about a European call option on Stock Z:
The call price is 5.50.
The call has a strike price of 47.
The call expires in two years.
The current stock price is 45.
The continuously compounded risk-free rate is 5%.
Stock Z will pay a dividend of 1.50 in one year.
Calculate the price of a European put option on Stock Z with a strike price of 47 that expires in two
years.
A. Less than 3.00
B. At least 3.00, but less than 3.50
C. At least 3.50, but less than 4.00
D. At least 4.00, but less than 4.50
E. At least 4.50
2014-MFE, Financial Economics 4 Put-Call Parity, HCM 11/13/13, Page 115
4.46 (CAS3, 11/07, Q.16) (2.5 points) An investor has been quoted a price on European
options on the same non-dividend paying stock. The stock is currently valued at 80 and the
continuously compounded risk-free interest rate is 3%. The details of the options are:
Option 1 Option 2
Type Put Call
Strike 82 82
Time to expiration 180 days 180 days
Based on his analysis, the investor has decided that the prices of the two options do not present
any arbitrage opportunities. He decides to buy 100 calls and sell 100 puts.
Calculate the net cost of this transaction.
(Hint: A positive net cost means the investor pays money from the transaction. A negative cost
means the investor receives money.)
A. Less than -60
B. At least -60, but less than -20
C. At least -20, but less than 20
D. At least 20, but less than 60
E. At least 60
4.47 (MFE/3F, 5/09, Q.12) (2.5 points) You are given:
(i) C(K, T) denotes the current price of a K-strike T-year European call option on a
nondividend-paying stock.
(ii) P(K, T) denotes the current price of a K-strike T-year European put option on the same stock.
(iii) S denotes the current price of the stock.
(iv) The continuously compounded risk-free interest rate is r.
Which of the following is (are) correct?
(I) 0 # C(50, T) - C(55, T) # 5e
-rT
(II) 50e
-rT
# P(45, T) - C(50, T) + S # 55e
-rT
(III) 45e
-rT
# P(45, T) - C(50, T) + S # 50e
-rT
(A) (I) only
(B) (II) only
(C) (III) only
(D) (I) and (II) only
(E) (I) and (III) only
2014-MFE, Financial Economics 4 Put-Call Parity, HCM 11/13/13, Page 116
4.48 (CAS8, 5/10, Q.16) (4 points) Given the following stock option information:
The current stock price is $60.
The strike price is $57.
The time to expiration is six months.
The risk-free rate is 3.0% compounded continuously.
European call option price is $3.
Dividend amount to be paid in three months is $2.
European put option price is $2.
Assume there are no transaction costs, it is possible to borrow or lend at the risk-free rate, and there
are no taxes to consider.
a. (1 point) Use put-call parity to demonstrate why an arbitrage opportunity exists with respect to
this stock option.
b. (3 points) Assume an investor takes a position in one option. Fully discuss how an investor would
take advantage of the arbitrage opportunity, including the investments that an investor would buy
and sell, the decision points during the arbitrage period, and the risk-free profit that would be made.
4.49 (IOA, CT8, 4/10, Q.2) (6 points) Consider a stock paying a dividend at a rate # and denote
its price at any time t by S
t
.
The dividend earned between t and T, T " t, is: S
t
{e
#(T-t)
- 1}.
Let C
t
and P
t
be the price at time t of a European call option and European put option
respectively, written on the stock S, with strike price K and maturity T " t .
The instantaneous risk-free rate is denoted by r.
Prove put-call parity in this context by adapting the proof of standard put-call parity that applies to
put and call options on a non-dividend paying stock.
4.50 (IOA, CT8, 9/10, Q.8) (5.25 points) Consider a particular stock and denote its price at any
time t by S
t
. This stock pays a dividend D at time T!.
Let C
t
and P
t
be the price at time t of a European call option and European put option respectively,
written on S, with strike price K and maturity T " T! " t .
The instantaneous risk-free rate is denoted by r.
Prove the put-call parity in this context by adapting the proof of standard put-call parity.
Hint: assume that when the dividend is paid it is used to pay off any borrowed positions required as
part of the proof.
2014-MFE, Financial Economics 4 Put-Call Parity, HCM 11/13/13, Page 117
Solutions to Problems:
4.1. B. C
Eur
(K, T) = P
Eur
(K, T) + S
0
- PV[Div] - K e
-rT
.
10 = P + 80 - {(1)e
-.015
+ (1)e
-.03
} - (75)e
-.04
. P = $4.01.
4.2. A. C
Eur
(K, T) = P
Eur
(K, T) + B
0
- PV[Coupons] - K e
-rT
.
C - P = 830 - (10)(e
-.01
+ e
-.02
+ e
-.03
+ e
-.04
+ e
-.05
+ e
-.06
+ e
-.07
+ e
-.08
) - (800)e
-.08
=
$15.01.
4.3. C. C
Eur
(K, T) = P
Eur
(K, T) + S
0
e
-#T
- K e
-rT
.
196 = P + 1300e
-.08
- 1800e
-.2
. P = 469.66.
4.4. D. C
Eur
(K, T) = P
Eur
(K, T) + S
0
e
-#T
- K e
-rT
.
C = 293 + 1300e
-.06
- 1500e
-.15
= 226.23.
4.5. A. With no dividends, # = 0 and: C
Eur
(K, T) - P
Eur
(K, T) = S
0
- K e
-rT
.
29.05 = 100 - K e
-.12
. K = 80.00.
4.6. C. C
Eur
(K, T) = P
Eur
(K, T) + S
0
e
-#T
- K e
-rT
= 13 + (160)e
-(.03)(1.5)
- (130)e
-(.05)(1.5)
=
45.35.
2014-MFE, Financial Economics 4 Put-Call Parity, HCM 11/13/13, Page 118
4.7. & 4.8. Sell the call for $13.00. Buy the put at $3.20.
We buy a share of stock for $169.70.
Borrowing the present value of the $160 strike price we receive:
160 exp[-(2%)(3.25/12)] = $159.136.
Borrowing the present value of the $0.36 dividend we receive:
.36 exp[-(2%)(1/12)] = $0.359.
The cost of our position is: 3.20 - 13.00 + 169.70 - 159.136 - 0.359 = $0.405.
So we need to come up with a positive amount of money in order to set up this position.
We get the dividend of $0.36 in one month and repay the lender.
At expiration if S
T
> K, then the person to whom we sold the call will exercise it and buy our share of
HAL for K. We will then pay K to the lender. If instead S
T
# K, then we sell our share of HAL for K to
the person from whom we bought the put. We will then pay K to the lender.
In either case, we end up with nothing at time T.
We needed to come up with a positive amount of money in order to set up a position which turns
out to be worth nothing.
Thus this is not a case of arbitrage.
Put-call parity would have been violated if there was arbitrage available.
Thus parity was not violated.
Comment: Similar to question 9.17 in Derivatives Markets by McDonald.
Beyond what I expect you to be asked on your exam.
2014-MFE, Financial Economics 4 Put-Call Parity, HCM 11/13/13, Page 119
4.9. & 4.10. Buy the call for $13.40. Sell the put at $2.90.
We borrow a share of stock, sell it for $169.70, and will give this person a share of stock in 3.25
months when the option expires. We also must pay this person the stock dividend they would
have gotten on the stock, when they would have gotten it.
Lending the present value of the $160 strike price we receive:
160 exp[-(1.9%)(3.25/12)] = $159.179.
Lending the present value of the $0.36 dividend we receive: .36 exp[-(1.9%)(1/12)] = $0.359.
The cost of our position is: 13.40 - 2.90 - 169.70 + 159.179 + 0.359 = $0.338.
So we need to come up with a positive amount of money in order to set up this position.
We will receive from the person to whom we made the loan the money to pay the stock dividend to
the person from whom we borrowed the stock. We will receive from the person to whom we made
the loan the strike price at expiration of the options 3.25 months from now. If S
T
" K, then we
exercise our call and buy a share of HAL for K, and give the share to the person from whom we
borrowed the stock. If instead S
T
< K, then we buy a share of HAL for K from the person who
exercises the put option we sold, and give the share to the person from whom we borrowed the
stock. In either case, we end up with nothing at time T.
We needed to come up with a positive amount of money in order to set up a position which turns
out to be worth nothing.
Thus this is not a case of arbitrage.
Put-call parity would have been violated if there was arbitrage available.
Thus parity was not violated.
4.11. B. With no dividends, # = 0. At the money means: K = S
0
= 85.
C
Eur
(K, T) = P
Eur
(K, T) + S
0
e
-#T
- K e
-rT
= 5 + 85 - 85e
-(1.5)(.065)
= 12.90.
Comment: For a non-dividend paying stock the difference between similar at the money options is:
C
Eur
(K, T) - P
Eur
(K, T) = S
0
(1 - e
-RT
).
2014-MFE, Financial Economics 4 Put-Call Parity, HCM 11/13/13, Page 120
4.12. Graph of the future value of a share of the stock plus a European put with a strike price of
100, S + E[(100 - S)
+
] =

S+100 4 S=100, for S<100
S + 0 = S, for S & 100
'
(
)
= Max[S, 100]:
50 100 150 200 250
Stock Price 100
120
140
160
180
200
220
240
Option Value
Comment: If today one buys a share of the stock plus a put option to sell the stock at 100, then at
the expiration of the option, you can sell the stock for 100 (by exercising the put option) or the
market price of the stock, whichever is higher.
4.13. E. 0 = C
Eur
(K, T) - P
Eur
(K, T) = S
0
e
-#T
- K e
-rT
.
% K = S
0
e
(r-#)T
= (130)exp[(.04 - .015)(2)] = 136.67.
2014-MFE, Financial Economics 4 Put-Call Parity, HCM 11/13/13, Page 121
4.14. Buy a call with strike 100: cost 27. Sell a put with strike 100: get 25.
Sell a call with strike 120: get 23. Buy a put with strike 120: cost 26.
Cost is: 27 + 26 - 25 - 23 = 5.
We borrow 5 at the risk free rate.
The payoff on the options is: (S
T
- 100)
+
- (100 - S
T
)
+
- {(S
T
- 120)
+
- (120 - S
T
)
+
} =
S
T
- 100 - (S
T
- 120) = 20.
For any reasonable values of r and T, 5e
rT
< 20.
After repaying the loan, we have: 20 - 5e
rT
> 0.
Thus for no investment, we end up with a position whose value is always positive.
Comment: Similar to Q. 9.8 in Derivatives Markets by McDonald.
The given call premiums are OK by themselves. The given put premiums are OK by themselves.
The given 100 strike premiums are OK by themselves.
The given 120 strike premiums are OK by themselves.
The arbitrage results from the relationship between all of the given premiums.
By put-call parity, C - P = PV[F
0,T
] - PV[K].
For the 100 strike options: 2 = PV[F
0,T
] - 100e
-rT
.
For the 120 strike options: -3 = PV[F
0,T
] - 120e
-rT
.
% e
-rT
= 5/20. % rT = ln(4) = 1.386.
If in fact rT = 1.386, then there is no arbitrage. (For example, T = 10 years and r = 13.86%.)
We assume that for any application to a real world situation, rT < 1.386.
Therefore, put-call parity must be violated. Therefore, there must be arbitrage.
2014-MFE, Financial Economics 4 Put-Call Parity, HCM 11/13/13, Page 122
4.15. C. C
Eur
(K, T) = P
Eur
(K, T) + S
0
- PV[Div] - K e
-rT
.
C = 15 + 120 - (2)e
-.05/4
- (130)e
-.05/3
. C = $5.17.
4.16. D. By Put-Call Parity, C - P = Se
-#T
- Ke
-rT
.
Therefore, 23.85 - 9.88 = 13.97 = Se
-#T
- 80e
-rT
, and 19.56 - 14.38 = 5.18 = Se
-#T
- 90e
-rT
.
Therefore subtracting these two equations, 10e
-rT
= 8.79. % e
-rT
= 0.879.
Therefore, Se
-#T
= 13.97 + (80)(.879) = 84.29.
Therefore, for K = 60, C = P - Ke
-rT
+ Se
-#T
= 3.45 - (60)(.879) + 84.29 = 35.00.
Therefore, for K = 110, P = C + Ke
-rT
- Se
-#T
= 13.09 + (110)(.879) - 84.29 = 25.49.
Sum of the prices of the 60 strike call and the 110 strike put is: 35.00 + 24.49 = $60.49.
4.17. S + E[(100 - S)
+
] - E[(100 - S)
+
] =

S+1004S - 0 =100, for S<100
S + 0 - (S - 100) = 100, for S & 100
'
(
)
= 100:
50 100 150 200 250
Stock Price
50
100
150
200
Option Value
Comment: If today you buy a share of the stock plus a put option to sell the stock at 100, and also
sell to Fred a call option with a strike price of 100, then at the expiration of the options:
(a) If the future stock price is below 100, you can sell the stock for 100 by exercising the put option.
Fred has no interest in exercising his call option and buying your share of the stock for 100, since 100
is higher than the market price of the stock.
(b) If the future stock price is above 100, Fred will exercise his call option to buy your share of the
stock for 100, which is lower than the market price of the stock.
In either case (a) or (b), you end up with 100.
By buying a share, buying a put, and selling a call, you have removed the uncertainty in the future
value of your investment. An example of put-call parity.
2014-MFE, Financial Economics 4 Put-Call Parity, HCM 11/13/13, Page 123
4.18. E. C
Eur
(K, T) = P
Eur
(K, T) + S
0
e
-#T
- K e
-rT
.
23 = P + (150)e
-.03/2
- (140)e
-.06/2
. % P = 11.10.
4.19. E. 0 = C - P = S
0
e
-#T
- K e
-rT
. % lnS
0
- #T = lnK - rT.
% T = ln[K/S
0
]/(r - #) = ln[1.15]/.04 = 3.49 years.
Comment: S
0
e
-#T
= 100 e
-(.02)(3.49)
= 93.3 = 115 e
-(.06)(3.49)
= K e
-rT
.
4.20. C. C
Eur
(K, T) = P
Eur
(K, T) + S
0
- PV[Div] - K e
-rT
.
8 = P + 100 - {(.8)e
-.055/4
+ (1.2)e
-.055/2
+ (1.5)e
-(.055)(.75)
} - (125)e
-(.055)(.75)
. P = $31.34.
4.21. C. In 2 months, the options will have 4 months until expiration.
By put call parity, C - P = S e
-#T
- K e
-rT
= 78.68 - 80 exp[-r/3].
Setting C = P: 78.68 = 80 exp[-r/3]. % r = 5.0%.
4.22. C. By Put-Call Parity, C - P = Se
-#T
- Ke
-rT
.
Therefore, 11 - 3 = 8 = Se
-#T
- 70e
-rT
,
and 6 - 7 = -1 = Se
-#T
- 80e
-rT
.
Therefore subtracting these two equations, 10e
-rT
= 9. % e
-rT
= 0.9.
Therefore, Se
-#T
= 8 + (70)(.9) = 71.
Therefore, for K = 90, P = C + Ke
-rT
- Se
-#T
= 3 + (90)(.9) - 71 = 13.
Alternately, if there were discrete dividends, then by Put-Call Parity, C - P = S - PV[Div]

- Ke
-rT
.
Therefore, 11 - 3 = 8 = 10.29 = S - PV[Div]

- 70e
-rT
,
and 6 - 7 = -1 = S - PV[Div]

- 80e
-rT
. Proceed as before.
4.23. B. By Put-Call parity, C - P = S
0
- PV[Div] - K e
-rT
.
0 = 84 - 2e
-.05/12
- 2e
-.05(4/12)
- 2e
-.05(7/12)
- 2e
-.05(10/12)
- K e
-.05(1)
. % K = 80.09.
2014-MFE, Financial Economics 4 Put-Call Parity, HCM 11/13/13, Page 124
4.24. E. Let us assume the strikes for the Call Bull Spread are K and K + 10.
Call Bull Spread: Buy a K strike call and sell a K + 10 strike call.
Premium for Call Bull Spread is: C(K) - C(K+10) = 4.19.
Put Bear Spread: Sell a K strike put and buy a K + 10 strike put.
By put-call parity: P(K) = C(K) + K e
-rT
- S
0
e
-#T
.
By put-call parity: P(K+10) = C(K+10) + (K+10) e
-rT
- S
0
e
-#T
.
Premium for the Put Bear Spread is:
P(K+10) - P(K) = C(K+10) - C(K) + 10e
-rT
= 10e
-(0.056)(2)
- 4.19 = 4.75.
Comment: Premium for the Put Bull Spread is -4.75.
Bull Spread: The purchase of an option together with the sale of an otherwise identical option with a
higher strike price. One can construct a bull spread using either puts or calls.
Bear Spread: The sale of an option together with the purchase of an otherwise identical option with a
higher strike price. One can construct a bear spread using either puts or calls.
4.25. A. Applying put call parity on January 1:
66.67 = 86.64 + 860 - Ke
-r/2
. % Ke
-r/2
= 879.97.
Applying put call parity on March 1:
154.64 = 21.24 + 1020 - Ke
-r/3
. % Ke
-r/3
= 886.60.
Therefore dividing the two equations, e
r/6
= 886.60/879.97 = 1.00753. % r = 0.045.
% K = 879.97 e
.045/2
= 900.
4.26. C. e
-#T
S
0
= C - P + e
-rT
K. % e
-#2
93 = 18.50 - 9.36 + e
-(0.052)(2)
90. % # = 1.5%.
2014-MFE, Financial Economics 4 Put-Call Parity, HCM 11/13/13, Page 125
4.27. E. The call premiums satisfy all three properties.
The put premiums satisfy all three properties.
However, applying put-call parity:

F
0,T
P
[S] = Ke
-rT
+ C - P = 60 e
-rT
+ 21 - 6.

F
0,T
P
[S] = 70 e
-rT
+ 17 - 11.

F
0,T
P
[S] = 90 e
-rT
+ 12 - 25.
Combining the first two equations: 60 e
-rT
+ 21 - 6 = 70 e
-rT
+ 17 - 11. % e
-rT
= 0.90.
Combining the last two equations: 70 e
-rT
+ 17 - 11 = 90 e
-rT
+ 12 - 25. % e
-rT
= 0.95.
Thus the sets of prices for the puts and calls are not consistent.
Thus one could use a portfolio of puts and calls to obtain arbitrage profit.
Comment: Similar to MFE Sample Exam, Q.2.
A graph of the call premiums:
60 70 90
S
12
17
21
C
A graph of the put premiums:
60 70 90
S
6
11
25
P
2014-MFE, Financial Economics 4 Put-Call Parity, HCM 11/13/13, Page 126
One can demonstrate arbitrage by: buying two 60-strike calls, selling two 60-strike puts,
selling three 70-strike calls, buying three 70-strike puts,
buying one 90-strike call, and selling one 90-strike put.
You get in the door initially for setting up this portfolio:
(2)(6) - (2)(21) + (3)(17) - (3)(11) + (1)(25) - (1)(12) = 1.
We invest this 1 at the risk free rate, and on the expiration date of the options we have e
rT
.
If S
T
< 60, all of the calls turn out to be worthless, and the payoffs from the puts are:
-(2)(60 - S) + (3)(70 - S) - (1)(90 - S) = 0.
If 60 # S
T
< 70, then the payoffs from the options are:
(2)(S - 60) + (3)(70 - S) - (1)(90 - S) = 0.
If 70 # S
T
< 90, then the payoffs from the options are:
(2)(S - 60) - (3)(S - 70) - (1)(90 - S) = 0.
If 90 # S
T
, then the puts are all worthless and the payoffs from the calls are:
(2)(S - 60) - (3)(S - 70) + (1)(S - 90) = 0.
So regardless, the payoffs are zero, and we end up with e
rT
, demonstrating arbitrage.
(S - K)
+
- (K - S)
+
= S - K.
Thus the payoff from buying a call and selling the similar put is: S - K.
Thus the payoff for this portfolio of puts and calls is: 2 (S
T
- 60) - 3 (S
T
- 70) + (S
T
- 90) = 0.
4.28. Let C(80) be the premium of the 80-strike call.
By put-call parity, C(80) = 8.61 + 76 - 80 e
-2r
= 84.61 - 80 e
-2r
.
This is positive.
The various properties of call premiums must hold.
7.93 < C(80) < 19.77. % 7.93 < 84.61 - 80 e
-2r
< 19.77. % 64.84 < 80 e
-2r
< 76.68.
% 0.8105 < e
-2r
< 0.9585. % 10.5% > r > 2.1%
The difference between the premiums must be less than the discounted difference in strikes.
19.77 - C(80) < 10 e
-2r
. % 19.77 - (84.61 - 80 e
-2r
) < 10 e
-2r
. % 70 e
-2r
< 64.84. % r > 3.8%.
C(80) - 7.93 < 20 e
-2r
. % 84.61 - 80 e
-2r
- 7.93 < 20 e
-2r
. % 76.68 < 100 e
-2r
. % r < 13.3%.
For convexity to hold, we must have C(80) < (2/3)(19.77) + (1/3)(7.93) = 15.82.
% 84.61 - 80 e
-2r
< 15.82. % 68.79 < 80 e
-2r
. % r < 7.5%.
Combining the conditions: 3.8% < r < 7.5%.
Comment: For example, if r = 6%, then C(80) = 84.61 - 80 e
-0.12
= 13.66.
Then C(80) < 15.82, so convexity holds. 19.77 > 13.66 > 7.93.
19.77 - C(80) = 6.11 < 8.87 = 10 e
-2r
. C(80) - 7.93 = 5.73 < 17.74 = 20 e
-2r
.
Thus all three properties of call premiums would hold.
2014-MFE, Financial Economics 4 Put-Call Parity, HCM 11/13/13, Page 127
4.29. E. By put-call parity, P(100) = C(100) + 100e
-rT
- PV[S].
Therefore, Dustin!s premium is: C(90) - C(100) - 100e
-rT
+ PV[S].
% 24.47 = C(90) - C(100) - 100e
-rT
+ PV[S].
By put-call parity, P(90) = C(90) + 90e
-rT
- PV[S].
Therefore, Edith!s premium is: C(100) - C(90) - 90e
-rT
+ PV[S].
% 16.85 = C(100) - C(90) - 90e
-rT
+ PV[S].
Adding the equations from Dustin and Edith: 41.32 = 2 PV[S] - 190e
-rT
.
By put-call parity, Frank!s premium is: C(120) - P(120) = PV[S] - 120e
-rT
= 1.61.
Therefore, 41.32 - (2)(1.61) = 240e
-rT
- 190e
-rT
. % e
-rT
= 0.762. % PV[S] = 93.05.
By put-call parity, Gabrielle!s premium is:
C(110) - P(110) = PV[S] - 110e
-rT
= 93.05 - (110)(0.762) = 9.23.
Alternately, Gabrielle!s premium is equal to Frank!s premium plus 10e
-rT
:
1.61 + (10)(0.762) = 9.23.
4.30. B. By put-call parity, P(80) = C(80) + 80e
-rT
- PV[S].
Therefore, Ashley!s premium is: C(60) - C(80) - 80e
-rT
+ PV[S].
% 1.22 = C(60) - C(80) - 80e
-rT
+ PV[S].
By put-call parity, P(60) = C(60) + 60e
-rT
- PV[S].
Therefore, Chelsea!s premium is: C(80) - C(60) - 60e
-rT
+ PV[S].
% 4.52 = C(80) - C(60) - 60e
-rT
+ PV[S].
Adding the equations from Ashley and Chelsea: 5.74 = 2 PV[S] - 140e
-rT
.
By put-call parity, Benjamin!s premium is: C(70) - P(70) = PV[S] - 70e
-rT
= 5.74/2 = 2.87.
Comment: Benjamin!s premium is an average of Ashley!s and Chelsea!s.
2014-MFE, Financial Economics 4 Put-Call Parity, HCM 11/13/13, Page 128
4.31. B. By put-call parity:
Call Premium - Put Premium = 82 e
-(3)(0.02)
-

80
1.1
3
= 17.11.
Comment: Assume Adam bought a three year call and loaned out

80
1.1
3
.
Then at time 3, Adam can use 80 to exercise his call if it makes sense to do so.
Assume Eve bought a three year put and a three-year prepaid forward contract on the stock.
Then at time 3, Eve could use her put to sell the stock for 80 if it makes sense to do so.
Then if S
2
> 80, both Adam and Eve end up with the stock.
Then if S
2
< 80, both Adam and Eve end up with 80.
Therefore, Call +

80
1.1
3
= Put + S
0
e
-3#
. A form of put-call parity.
4.32. D. A(K) + B(K) = C(K) - C(K + 10) + P(K + 10) - P(K)
= {C(K) - P(K)} - {C(K + 10) - P(K + 10)} = S
0
e
-#T
- K e
-rT
- {S
0
e
-#T
- (K + 10) e
-rT
} = 10e
-rT
.
Thus A(K) + B(K) is independent of K.
A(95) + B(95) = 3.49 + 5.36 = 8.85.
% A(80) = 8.85 - B(80) = 8.85 - 3.87 = 4.98.
4.33. B. C = P + S - K e
-rt
= 25 + 120 - 100 e
-0.1
= $54.52.
Comment: If instead you assume the 10% is an effective annual rate, then:
C = P + S - K e
-rt
= 25 + 120 - 100/1.1 = $54.09.
2014-MFE, Financial Economics 4 Put-Call Parity, HCM 11/13/13, Page 129
4.34. a. One borrows shares of ABC stock from someone who owns them, and then sell the
shares. You promise to return the shares of stock in the future, in this case in one year, along with the
present value of any dividends issued on the stock.
b. Assuming ABC stock pays no dividends, then one year from now the investor will have to buy a
share of ABC stock with price S
1
; ignoring interest the payoff is: 100 - S
1
.
50 100 150 200 250
S
- 150
- 100
- 50
50
100
Payoff
c. Buying a one-year 200-strike European call would limit her lose to $100. The payoff on the call:
50 100 150 200 250
S
10
20
30
40
50
Payoff
2014-MFE, Financial Economics 4 Put-Call Parity, HCM 11/13/13, Page 130
The investor's net payoff:
50 100 150 200 250
S
- 100
- 50
50
100
Payoff
d. Assuming no dividends, C = P + S - Ke
-rT
= 95 + 100 - 200 e
-0.1
= 14.03.
4.35. C. C = P + Se
-#T
- Ke
-rT
. Since C = P and # = 0, S = Ke
-rT
= 100e
-.12
= 97.56.
2014-MFE, Financial Economics 4 Put-Call Parity, HCM 11/13/13, Page 131
4.36. (i) Put-call parity expresses a relationship between the price of a put option and the price of a
call option on a stock where the options have the same exercise dates and strike prices.
(ii) Consider a portfolio A which contains one European call and an amount of cash D + Xe
-r(T-t)
,
where X = exercise price, r = risk-free rate, T - t = time to exercise of the option,
D = present value of dividends payable.
At the exercise date if the share price S
T
" X then the call will be exercised and portfolio A will have
a value of D e
r(T-t)
+ S
T
.
If at T we have S
T
< X then the call will not be exercised and portfolio A will be worth D e
r(T-t)
+ X.
Now consider portfolio B consisting of one European put and a share. At the exercise date if S
T
" X
then the put will not be exercised and portfolio B will have value of S
T
+ D e
r(T-t)
If at the exercise date T, we have S
T
< X then the put will be exercised and portfolio B will have a
value of X + D e
r(T-t)
.
Portfolios A and B have the same value in all circumstances at the exercise date T.
Hence they must be equivalent at all earlier times. % the portfolios are of equal value.
Therefore c + D + Xe
-r(T-t)
= p + S
t
. c = value of European call with strike X and exercise date T.
p = value of European put with strike X and exercise date T. S
t
= value of stock at time t.
(iii) Let D be the present value of dividends payable and consider: c + D + X e
-r(T-t)
< p + S
t
.
Then for some amount A: A + c + D + X e
-r(T-t)
= p + S
t
.
Hence we can short one share and sell a put and receive p + S
t

At the exercise date we know the value of this portfolio is: max[S
T
+ D e
r(T-t)
, X + D e
r(T-t)
].
However we know that the value of a portfolio invested in a European call and D + Xe
-r(T-t)
at time t
will be worth max[S
T
+ D e
r(T-t)
, X + D e
r(T-t)
] at time T.
This is the same as the amount we must repay at time T.
Hence we are left with a profit of Ae
r(T-t)
.
Therefore, the strategy in order to make a riskless profit is short 1 share and sell a put, buy 1 call and
put on deposit A + D + X e
-r(T-t)
.
The net investment is zero at time t, and we end up with Ae
r(T-t)
at time T.
If the inequality is reversed also reverse investment (i.e. swap long positions for short positions and
vice versa).
4.37. A. The put-call parity formula for a European call and a European put on a
nondividend-paying stock with the same strike price and maturity date is: C - P = S
0
- Ke
-rT
.
0.15 = 60 - 70e
-r4
. % r = 0.039.
2014-MFE, Financial Economics 4 Put-Call Parity, HCM 11/13/13, Page 132
4.38. D. By put call parity, C - P = S - PV[Div] - Ke
-rT
. For at the money options, S = K, and C - P
= K - PV[Div] - Ke
-rt
. Assuming the dividend rate is less than r, C - P < 0. Therefore, the premium of
the at-the-money put is less than the premium of the at-the-money call.
Therefore, if the call is at-the-money, the put option with the same cost will have a higher strike price.
However, a purchased collar requires that the put have a lower strike price.
Thus statement D is false.
Comment: On the syllabus of an earlier exam; beyond the level of detail you are likely need for
your exam. A collar is the purchase of a put option and the sale of a similar call with a higher strike;
both options are on the same stock and have the same expiration date. If the collar is written rather
than purchased, then the put is sold and the call is bought. In the case of a zero-cost collar, the two
option premiums are the same, and the net cost of the collar is zero.
4.39. C. For no dividends, by put call parity: C - P = S - Ke
-rT
.
% 66.59 - 18.64 = 500 - Ke
-0.06
. % K = 480.
4.40. E. You want to buy the call and sell the put.
Let x be the future value of the index one year from now.
If x > 1,025, then you will receive x - 1,025 from the call. After buying the index for x you will have
spent a total of 1,025.
If x < 1,025, then you will pay 1,025 x to the person who bought the put from you.
After buying the index for x you will have spent a total of 1,025.
One way to get the cost is to note that the forward price is: 1,000(1.05) = 1,050. You want to pay
1,025, which is 25 less. Therefore, you must spend 25/1.05 = 23.81 today, in order to guarantee
that outcome.
Alternately, by put-call parity with no dividends, using effective annual rates rather than continuously
compounded rates: C - P = S - K/(1 + r)
T
= 1000 - 1025/1.05 = 23.81.
2014-MFE, Financial Economics 4 Put-Call Parity, HCM 11/13/13, Page 133
4.41. B. Based on put-call parity, we would expect the call to have a price of:
P - Ke
-rT
+ Se
-#T
= 2.35 - 50 e
-.03/2
+ 49.70 e
-.02/2
= $2.30.
Therefore, the call is underpriced at $2.00.
The present value arbitrage profit is: $2.30 - $2.00 = $0.30.
Alternately, based on put-call parity, we would expect the put to have a price of:
C + Ke
-rT
- Se
-#T
= 2.00 + 50 e
-.03/2
- 49.70 e
-.02/2
= $2.05.
Therefore, the put is overpriced at $2.35.
The present value arbitrage profit is: $2.35 - $2.05 = $0.30.
Comment: We can take advantage of the arbitrage by:
buying a call, selling a put, investing Ke
-rT
, and shorting e
-#T
= e
-.02/2
= .990 shares of stock.
(We borrow .990 shares of stock from someone at time 0 and return 1 share of stock at time 1/2.
If one owned .990 shares and reinvested the dividends, one would have .990e
.02/2
= 1 share at
time 1/2.)
When we set up the position we receive: 2.35 - 2.00 + (49.70)e
-.02/2
= 49.556.
We invest this at the risk free rate, and end up with 49.556 e
.03/2
= 50.305, at time 1/2.
At time 1/2 we need 1 share of stock to return to the person we borrowed it from.
If S
1/2
> K = 50, then we use our call to buy a share of stock at 50.
If S
1/2
< K = 50, then the person to whom we sold the put uses it to sell us a share of stock at 50.
In either case, we use $50 to buy a share of stock, and are left with $50.305 - $50 = $0.305.
The present value is: $0.305 e
-.03/2
= $0.30.
Put another way, one can buy a synthetic put by:
buying a call, lending at the risk free rate K e
-rT
, and shorting e
-#T
= e
-.02/2
shares of stock.
The cost of this synthetic put is: 2.00 + 50 e
-.03/2
- 49.70 e
-.02/2
= $2.05.
We can sell an actual put for $2.35 and buy the synthetic put for $2.05, making $0.30.
4.42. E. Based on put-call parity,
P = C + Ke
-rT
- (S - PV[Dividends]) = 2 + 30 e
-.10/2
- {29 - .50e
-.10/6
- .50e
-.10(5/12)
} = $2.51.
4.43. B. By put-call parity, C = P + S - PV[Div] - K e
-rT
. Let x be the size of each dividend.
4.50 = 2.45 + 52 - x{e
-(.06)(2/12)
+ e
-(.06)(5/12)
} - (50)e
-(.06)(6/12)
. % x = 0.726.
2014-MFE, Financial Economics 4 Put-Call Parity, HCM 11/13/13, Page 134
4.44. E. By put-call parity, P = C - Se
-#T
+ Ke
-rT
= C - 50e
-.08
+ Ke
-.04
.
K C Immediate Payoff P Exercise Immediately
$40 $9.12 $0 $1.40 No
$50 $4.91 $0 $6.79 No
$60 $0.71 $10 $12.20 No
$70 $0.00 $20 $21.10 No
We exercise immediately if the payoff for the put, (K - 50)
+
, is more than the premium for a
European Put. That is not the case for any of the four examples here.
Comment: The present value of an option if one does not exercise it, is called the continuation value.
American options can be exercised any time up through expiration. Thus unlike here, one would
need to compare the payoff and the continuation value at more times than just initially, in order to
decide whether it is optimal to exercise early.
4.45. D. P = C + PV[K] + PV[Div] - S = 5.50 + 47 e
-(2)(.05)
+ 1.5 e
-.05
- 45 = 4.45.
Alternately, the prepaid forward price for the stock is: 45 - 1.5 e
-.05
= 43.57.
P = 5.50 + 47 e
-(2)(.05)
- 43.57 = 4.46.
4.46. A. C - P = S - K exp[-rT] = 80 - 82 e
-(.03)(1/2)
= -0.7792.
100(C - P) = -77.92.
Comment: Since the call is worth less than the put, the investor receives money.
4.47. E. A 55 strike call is worth less than a 50 strike call, so 0 # C(50, T) - C(55, T).
The difference in payoffs between a 50 strike and 55 strike call is at most 5.
Since these are European options, these payoffs takes place T years in the future.
Therefore, C(50, T) - C(55, T) # 5e
-rT
. Statement I is true.
By put-call parity, P(45, T) = C(45, T) + 45e
-rT
- S.
Therefore, P(45, T) - C(50, T) + S = C(45, T) - C(50, T) + 45e
-rT
.
However, 0 # C(45, T) - C(50, T) # 5e
-rT
. 3 45e
-rT
# C(45, T) - C(50, T) + 45e
-rT
# 50e
-rT
.
3 45e
-rT
# P(45, T) - C(50, T) + S

# 50e
-rT
. Statement III is true.
Since Statement III is true, Statement II is not true.
2014-MFE, Financial Economics 4 Put-Call Parity, HCM 11/13/13, Page 135
4.48. (a) Using put-call parity, the price of the call should be:
2 + 60 - 2e
-0.03/4
- 57 e
-0.03/2
= $3.86.
Thus the call is underpriced at $3.
Alternately, using put-call parity, the price of the put should be:
2 + 57 e
-0.03/2
- (60 - 2e
-0.03/4
)

= $1.14.
Thus the put is overpriced at $2.
(b) We can buy the call and sell a synthetic call.
To sell the synthetic call, we sell a put, short sell a share of stock, lend 2e
-0.03/4
the present value of
dividends, and lend the strike price for 6 months.
We net $0.86 from setting up this position.
In six months, if the stock price is greater than the strike price of $57, we use the call to buy the stock
for $57, repay the stock and accumulated value of dividends to the person from whom we
borrowed the stock.
We are left with $0.86 accumulated for 6 months of interest.
In six months, if the stock price is less than the strike price of $57, then the person to whom we sold
the put uses it to sell us a share of stock for $57; we repay the stock and accumulated value of
dividends to the person from whom we borrowed the stock.
We are left with $0.86 accumulated for 6 months of interest.
4.49. Consider two portfolios.
Portfolio A: buying the call and selling the put at time t.
Its value at time t is C
t
- P
t
and at time T, it is S
T
- K in all states of the universe.
Portfolio B: buying a fraction exp[-#(T - t)] of the underlying asset for S
t
exp[-#(T - t)]
and borrowing K exp[-r(T - t)] at time t.
Its value at time t is then S
t
exp[-#(T - t)] - K exp[-r(T - t)].
Its value at maturity is then S
T
- K by taking into account the dividends which are paid
continuously at rate #.
Using the absence of arbitrage opportunity, both portfolios should have the same value at any
intermediate time, in particular at time t. Hence:
C
t
- P
t
= S
t
exp[-#(T - t)] - K exp[-r(T - t)].
Comment: There are other sets of two portfolios of equal value one could set up in order to prove
put-call parity. For example:
Portfolio A: At time t, buying a call option and lending K exp[-r(T - t)].
Portfolio B: At time t, buying the put option and buying exp[-#(T - t)] shares of stock.
2014-MFE, Financial Economics 4 Put-Call Parity, HCM 11/13/13, Page 136
4.50. Adam buys the call and loans out Ke
-r(T-t)
.
At time T, Adam has the call and K.
Eve buys the stock, the put, and borrows D exp[-r(T!-t)].
At time T!, Eve gets the dividend of size D, and pays off her loan.
At time T, Eve has the stock and the put.
If S
T
> K, Adam uses his call to buy the stock for K, and he ends up with the stock.
If S
T
> K, Eve!s put expires worthless, and she ends up with the stock.
If S
T
< K, Adam!s call expires worthless, and he ends up with K.
If S
T
< K, Eve uses her put to sell the stock for K, and she ends up with K.
Since they end up in the same position, the initial costs of their portfolios must have been the same.
% C
t
+ Ke
-r(T-t)
= P
t
+ S
t
- D exp[-r(T!-t)] = P
t
+ S
t
- PV[Dividends] = P
t
+

F
t,T
P
[S].
2014-MFE, Financial Economics 4 Put-Call Parity, HCM 11/13/13, Page 137
Section 5, Bounds on Premiums of European Options
Option premiums must stay within certain bounds.
Maximum and Minimum Prices of Calls:
103

Since an option does not require its owner to do anything, it never has a negative value.
E[(S
T
- K)
+
] " 0, and E[(K - S
T
)
+
] " 0. Therefore, E[(S
T
- K)
+
] e
-rT
" 0, and E[(K - S
T
)
+
] e
-rT
" 0.
If Joe buys a call option, then he must pay K > 0 if he wants to own a share of stock at time T in the
future.
For S
0
Mary can buy a share of the stock now and if she holds on to it, guarantee she owns a share
at time T in the future.
After Mary buys her share of stock and after Joe buys his option, Mary is in a better position than
Joe, even ignoring any dividends Mary may get on the stock. Therefore, the value of the call option
is less than (or equal to) S
0
, the current market share of the stock.
Owning a share of stock now is worth at least as much as the option to buy a share of stock in the
future. % A call is never worth more than the current stock price.
S
0
" C(S
0
, K, T) " 0.
In fact, one can pay the prepaid forward price,

F
0,T
P
(S
0
), and will own a share of stock at time T.
This is more valuable than the option to buy a share of stock at time T in the future.
For example, Tim pays Howard the prepaid forward price for a share of ABC stock and in exchange
Howard agrees to give Tim a share of ABC stock two years from now. By paying this amount of
money today, Tim will own a share of ABC stock two years from now. Instead, Tim could buy a two
year $100 strike call on ABC stock from John. Two years from now Tim will have the option to pay
$100 to buy a share of ABC stock from John.
Owning a share of stock two years from now is more valuable than having the option to buy a share
of stock two years from now for $100. The future value of the former is S
2
. The future value of the
latter is: (S
2
- 100)
+
< S
2
.

Therefore, a somewhat more stringent inequality holds:

F
0,T
P
(S
0
) " C
Eur
(S
0
, K, T).
104

103
See pages 276-277 of Derivatives Markets by McDonald.
104
McDonald does not mention this in Derivatives Markets.
In the absence of dividends, the prepaid forward price is just S
0
.
2013-MFE/3F, Financial Economics 5 Bounds on Premiums Euro., HCM 11/13/13, Page 138
Phyllis can buy a forward contract to buy a share of stock at T in the future.
At time T in the future Phyllis will pay F
0,T
for a share of stock.
Harry can instead buy a call option for this same stock at time T in the future, for a price of
C(S
0
, K, T), and can in addition invest PV
0,T
[K] in a risk-free investment. At time T Harry will then
have the money to buy a share of stock if he chooses to exercise his option.
Phyllis will own a share of stock at time T. Harry can choose to own a share of stock at time T if it is
advantageous to him to exercise his option. Therefore, Harry!s position is worth more than Phyllis!s.
C(S
0
, K, T) + PV
0,T
[K] " PV
0,T
[F
0,T
]. % C(S
0
, K, T) " PV
0,T
[F
0,T
] - PV
0,T
[K].
105

Therefore, recalling that an option can never have a negative value:
S
0
" C(S
0
, K, T) " (PV
0 , T
[F
0 , T
] - PV
0 , T
[K])
+
.
106

However, PV
0,T
[F
0,T
] = S
0
- PV[Div]. Therefore, C(S
0
, K, T) " (S
0
- PV[Div] - PV
0,T
[K])
+
.
Exercise: K = 100, T = 2, r = 6%, and # = 2%.
What are the bounds on the call premium as a function of S
0
?
[Solution: PV
0,T
[F
0,T
] - PV
0,T
[K] = S
0
e
-#T
- K e
-rT
= S
0
e
-0.04
- 100 e
-0.12
= 0.961 S
0
- 88.69.
S
0
" C(S
0
, K, T) " (0.961 S
0
- 88.69)
+
.
Comment: 0.961 S
0
- 88.69 = 0, for S
0
= 92.3 = Ke
-(r-#)T
.]
For these inputs, here is a graph of the possible call premiums as a function of S
0
:
50 92.3 150
S0
20
40
60
80
100
120
140
C
50 92.3 150
S0
20
40
60
80
100
120
140
C
105
From put-call parity: C = P + PV[F
0,T
] - PV[K]. Also P " 0. % C " PV[F
0,T
] - PV[K].
106
See Equation 9.11 in Derivatives Markets by McDonald.
2013-MFE/3F, Financial Economics 5 Bounds on Premiums Euro., HCM 11/13/13, Page 139
Exercise: S
0
= 90, T = 2, r = 6%, and # = 2%.
What are the bounds on the call premium as a function of K?
[Solution: PV
0,T
[F
0,T
] - PV
0,T
[K] = S
0
e
-#T
- K e
-rT
= 90 e
-0.04
- K e
-0.12
= 86.47 - 0.887 K.
90 " C(S
0
, K, T) " (86.47 - 0.887 K)
+
.
Comment: 86.47 - 0.887 K = 0, for K = 97.5 = S
0
e
(r-#)T
.]
For these inputs, here is a graph of the possible call premiums as a function of K:
50 97.5 150
K
20
40
60
80
C
50 97.5 150
K
20
40
60
80
C
Maximum and Minimum Prices of Puts:
If one owns a put option, then the most one can gain is K, if the stock price goes to zero.
Therefore, the value of a put is at most its strike price K.
107

K " P(S
0
, K, T).
In fact, since the possible gain from the put occurs at time T, the value of the put is at most the
present value of K, e
-rT
K: PV[K] " P(S
0
, K, T).
Susan buys a put option on a share of stock with expiration T, and also enters a forward contract to
buy the stock at T, at price F
0,T
. Then if Susan invests PV
0,T
[F
0,T
], she will have the money at time
T to buy the stock as per her forward contract. If the price of the stock is greater than K she will not
exercise her put option and instead she keeps the stock. If the price of the stock at time T is less
than K she will exercise her option, and sell the share of stock for K. In either case, her position at
time T is worth at least K.
107
P
Eur
(S
0
, K, T) = E[(K - S
T
)
+
] e
-Tr
# E[(K - S
T
)
+
] # K.
2013-MFE/3F, Financial Economics 5 Bounds on Premiums Euro., HCM 11/13/13, Page 140
Therefore, P(S
0
, K, T) + PV
0,T
[F
0,T
] " PV
0,T
[K]. % P(S
0
, K, T) " PV
0,T
[K] - PV
0,T
[F
0,T
].
108

Therefore, recalling that an option can never have a negative value:
K " P(S
0
, K, T) " (PV
0 , T
[K] - PV
0 , T
[F
0 , T
])
+
.
109

However, PV
0,T
[F
0,T
] = S
0
- PV[Div].
Therefore, P(S
0
, K, T) " (PV
0,T
[K] - S
0
+ PV[Div])
+
.
Exercise: K = 80, T = 3, r = 5%, and # = 1%.
What are the bounds on the put premium as a function of S
0
?
[Solution: PV
0,T
[K] - PV
0,T
[F
0,T
] = K e
-rT
- S
0
e
-#T
= 80 e
-0.15
- S
0
e
-0.03
= 68.86 - 0.970 S
0
.
80 " P(S
0
, K, T) " (68.86 - 0.970 S
0
)
+
.
Comment: 68.86 - 0.970 S
0
= 0, for S
0
= 71.0 = Ke
-(r-#)T
.]
For these inputs, here is a graph of the possible put premiums as a function of S
0
:
50 71 150
S0
20
40
60
80
P
50 71 150
S0
20
40
60
80
P
Exercise: S = 70, T = 3, r = 5%, and # = 1%.
What are the bounds on the put premium as a function of K?
[Solution: PV
0,T
[K] - PV
0,T
[F
0,T
] = K e
-rT
- S
0
e
-#T
= K e
-0.15
- 70 e
-0.03
= 0.861 K - 67.93.
K " P(S
0
, K, T) " (0.861 K - 67.93)
+
.
Comment: 0.861 K - 67.93 = 0, for K = 78.9 = S
0
e
(r-#)T
.]
108
From put-call parity: P = C + PV[K] - PV[F
0,T
]. Also C " 0. % P " PV[K] - PV[F
0,T
].
109
See Equation 9.10 in McDonald.
2013-MFE/3F, Financial Economics 5 Bounds on Premiums Euro., HCM 11/13/13, Page 141
For these inputs, here is a graph of the possible put premiums as a function of K:
50 78.9 150
K
20
40
60
80
100
120
140
P
50 78.9 150
K
20
40
60
80
100
120
140
P
For a European put, the most you can be paid is K at time T, which has present value Ke
-rT
.
Therefore, Ke
-rT
" P(S
0
, K, T).
110

Relationship to Put-Call Parity:
From Put-Call Parity, we have:
C
Eur
(K, T) = P
Eur
(K, T) + PV[F
0,T
] - PV[K].
Since, P
Eur
(K, T) " 0, C
Eur
(K, T) " PV[F
0,T
] - PV[K], as discussed previously.
Since, C
Eur
(K, T) " 0, P
Eur
(K, T) + PV[F
0,T
] - PV[K] " 0.
Therefore, P
Eur
(K, T) " PV[K] - PV[F
0,T
], as discussed previously.
110
McDonald does not mention this in Derivatives Markets.
See for example, Options, Futures, and Other Derivatives by John C. Hull, not on the syllabus.
2013-MFE/3F, Financial Economics 5 Bounds on Premiums Euro., HCM 11/13/13, Page 142
An Increasing Strike Price:
111

Rather than being fixed as is usual, the strike price could increase with the time until expiration.
Let the strike price for an option with time until expiration T be: K
T
= Ke
rT
.
In other words, we keep the present value of the strike price constant.
112


On a stock that does not pay dividends, when the strike price grows at the risk free rate, then the
premiums on European puts increase with time to expiration.
113

Exercise: Demonstrate that an arbitrage opportunity exists in the following situation.
The risk free rate is 6%.
The premium for a one-year 100-strike European put on a non-dividend paying stock is 5.
The premium for the two-year 106.18-strike European put on the same stock is 4.
[Solution: Buy the two-year put and sell the one-year put.
You get 1, which you invest at the risk-free rate. At time 1 you have e
0.06
= 1.0618.
If S
1
> 100 then you do not pay off on the one-year put you sold.
You have 1.0618 plus the two year put; your position is positive.
If S
1
< 100 then you pay off on the one-year put: 100 - S
1
.
However, the premium of the 106.18-strike put that you still own is greater than or equal to:
(PV[K] - PV[F
0,T
])
+
= (106.18 e
-0.06
- S
1
)
+
= 100 - S
1
.
Thus the put you still own is worth at least as much as the amount you pay off on the put you sold.
Thus you have at least 1.0618; your position is positive. Demonstrating arbitrage.
Comment: Note that the strike for the two year put is: (100) e
0.06
= 106.18.
Since the stock pays no dividends, its prepaid forward price is just its current price.
The result did not depend on the particular values other than the fact that 4 < 5.]
If the options start at the money, then the increasing strike price is: S
0
e
rT
.
From put-call parity, C - P = Ke
-rT
- Se
-#T
.
Thus, in this situation with no dividends and K = S
0
e
rT
: C - P = 0.
The call premium is equal to the put premium. They both increase with T.
In general, for a European call on a non-dividend paying stock, when the strike price grows at the risk
free rate, then the premiums increase with time to expiration.
114
111
See page 298 of Derivatives Markets by McDonald.
112
This is analogous to keeping a insurance policy deductible or limit up with inflation.
113
Since the put premium increases with the strike price, if the strike price increases with T at a rate greater than r,
then the put premium increases even faster with T.
114
Since the call premium decreases with the strike price, if the strike price increases with T at a rate less than r, then
the call premium increases even faster with T.
2013-MFE/3F, Financial Economics 5 Bounds on Premiums Euro., HCM 11/13/13, Page 143
Exercise: For a stock that does not pay dividends, demonstrate that when the strike price grows at
the risk free rate, the premium of a European call increases with time to expiration.
[Solution: K
T
= Ke
rT
. Take two similar calls, with different times to expiration T < U.
Assume the premium of the first call with less time to expiration is more than that of the second call.
Buy the U-year call and sell the T-year call
Since we have assumed that the premium of the T-year call is greater than the premium of the
U-year call, we get money in the door, which we invest at the risk-free rate.
If S
T
# Ke
rT
then we do not pay off on the T-year call we sold.
We have money plus the T-year call; our position is positive.
If S
T
> Ke
rT
then we pay off on the T-year call: S
T
- Ke
rT
.
However, the premium of the call we own with strike Ke
rU
, that has U - T until expiration, is greater
than or equal to: (PV[Forward contract on the stock] - PV[K])
+
= (S
T
- e
-r(U-T)
Ke
rT
)
+

= S
T
- e
-r(U-T)
Ke
rU
= S
T
- Ke
rT
.
Thus the call we still own is worth at least as much as the amount we pay off on the call we sold.
Thus we have at least the money we invested at time 0, plus the interest we earned on it;
your position is positive. Demonstrating arbitrage.]

Portfolio Insurance for the Long Run:
115
If you owned a share of a stock that pays no dividends and purchased a put with strike price S
0
e
rT
,
then if at time T: S
T
< S
0
e
rT
, you could use your put to sell the stock for S
0
e
rT
. You would
guarantee that over the period time from 0 to T you would earn at least the risk free rate on the stock
plus put, not including the cost to buy the put.
However, as we have seen the cost of such puts increases with T.
116
The longer the period of time
we wish to insure, the more the premium. One can apply the same idea to a portfolio of different
stocks.
115
See page 299 of Derivatives Markets by McDonald. See also page 603, to be discussed subsequently.
116
When there are no dividends. If there are dividends, we would instead have the strike increase at the rate r - #;
after taking into account the dividends we receive, we would earn at least the risk free rate.
2013-MFE/3F, Financial Economics 5 Bounds on Premiums Euro., HCM 11/13/13, Page 144
Problems:
5.1 (1 point) A European 140 strike 2 year call is an option on a stock with current price 100 and
forward price of 108. r = 5%.
Which of the following intervals represents the range of possible prices of this option?
A. [0, 100] B. [0, 108] C. [9.05, 100] D. [9.05, 108] E. [8, 108]
5.2 (1 point) A European 140 strike 2 year put is an option on a stock with current price 100 and
forward price of 108. r = 5%.
Determine the range of possible prices of this option.
5.3 (3 points) One has a European option on a stock with current market price of 100.
The time until expiration is 2 years. r = 5%.
The two year forward price of the stock is 110.
Which the following statements are true?
A. The premium of a call option with a strike price of 80 could not be 80.
B. The premium of a put option with a strike price of 140 could not be 105.
C. The premium of a call option with a strike price of 80 could not be 25.
D. The premium of a put option with a strike price of 140 could not be 30.
E. The premium of an at-the-money put must be at least the premium of an at-the-money call.
5.4 (1 point) A European 110 strike 2 year put is an option on a stock with current price 100 and
forward price of 108. r = 5%.
Which of the following intervals represents the range of possible prices of this option?
A. [0, 100] B. [0, 110] C. [1.81, 108] D. [1.81, 110] E. [2, 108]
5.5 (1 point) A European 105 strike 2 year call is an option on a stock with current price 100 and
forward price of 108. r = 5%.
Which of the following intervals represents the range of possible prices of this option?
A. [0, 100] B. [0, 108] C. [2.72, 100] D. [2.72, 108] E. [8, 108]
2013-MFE/3F, Financial Economics 5 Bounds on Premiums Euro., HCM 11/13/13, Page 145
5.6 (2 points) ABC stock pays no dividends. r = 8%.
For a 90 strike 6 month European put on ABC Stock, which of the following graphs represents the
bounds on the put premium as a function of S
0
?
25 50 75 100 125 150
S0
20
40
60
80
P
A.
25 50 75 100 125 150
S0
20
40
60
80
P
25 50 75 100 125 150
S0
20
40
60
80
100
120
140
P
B.
25 50 75 100 125 150
S0
20
40
60
80
100
120
140
P
25 50 75 100 125 150
S0
20
40
60
80
P
C.
25 50 75 100 125 150
S0
20
40
60
80
P
25 50 75 100 125 150
S0
20
40
60
80
100
120
P
D.
25 50 75 100 125 150
S0
20
40
60
80
100
120
P
25 50 75 100 125 150
S0
20
40
60
80
P
E.
25 50 75 100 125 150
S0
20
40
60
80
P
2013-MFE/3F, Financial Economics 5 Bounds on Premiums Euro., HCM 11/13/13, Page 146
5.7 (2 points) XYZ stock pays dividends at a continuously compounded rate of 2%.
XYZ stock has a current price of 80. r = 10%.
For a 3 year European call on XYZ Stock, which of the following graphs represents the bounds on
the call premium as a function of K?
25 50 75 100 125 150
K
20
40
60
80
C
A.
25 50 75 100 125 150
K
20
40
60
80
C
25 50 75 100 125 150
K
20
40
60
80
100
120
140
C
B.
25 50 75 100 125 150
K
20
40
60
80
100
120
140
C
25 50 75 100 125 150
K
10
20
30
40
50
60
70
C
C.
25 50 75 100 125 150
K
10
20
30
40
50
60
70
C
25 50 75 100 125 150
K
10
20
30
40
50
60
70
C
D.
25 50 75 100 125 150
K
10
20
30
40
50
60
70
C
25 50 75 100 125 150
K
20
40
60
80
C
E.
25 50 75 100 125 150
K
20
40
60
80
C
5.8 (CAS5B, 11/94, Q.29) (1 point): Explain why the price of a European call option approaches
the prepaid forward price of the underlying stock less the present value of the strike price as the
stock rises further and further beyond the strike price.
2013-MFE/3F, Financial Economics 5 Bounds on Premiums Euro., HCM 11/13/13, Page 147
Solutions to Problems:
5.1. A. S
0
" C(S
0
, K, T) " (PV
0,T
[F
0,T
] - PV
0,T
[K])
+
.
100 " C " (108 e
-.1
- 140 e
-.1
)
+
= 0.
Comment: While McDonald does not mention this in Derivatives Markets, for a European call,
C #

F
0,T
P
(S
0
) = PV
0,T
[F
0,T
] = 108 e
-.1
= 97.72.
5.2. K " P(S
0
, K, T) " (PV
0,T
[K] - PV
0,T
[F
0,T
])
+
.
140 " P " (140 e
-.1
- 108 e
-.1
)
+
= 28.95.
Comment: While McDonald does not mention this in Derivatives Markets, for a European put,
P # Ke
-rt
= 140 e
-.1
= 126.68.
5.3. C. S
0
" C(S
0
, K, T) " (PV
0,T
[F
0,T
] - PV
0,T
[K])
+
.
Therefore, 100 " the premium of the call.
More precisely, C #

F
0,T
P
(S
0
) = PV
0,T
[F
0,T
] = 110 e
-.1
= 99.53.
Statement A is not true.
The premium of the call " (110 - 80)/e
.1
= 27.15.
Statement C is true.
K " P(S
0
, K, T) " (PV
0,T
[K] - PV
0,T
[F
0,T
])
+
.
Therefore, 140 " the premium of the put.
More precisely, P # PV
0,T
[K] = 140 e
-.1
= 126.67.
Statement B is not true.
The premium of put " (140 -110)/e
.1
= 27.15.
Statement D is not true.
By put-call parity: P - C = PV
0,T
[K] - PV
0,T
[F
0,T
] = 100e
-.1
- 110e
-.1
= -9.05.
Statement E is not true.
5.4. D. K " P(S
0
, K, T) " (PV
0,T
[K] - PV
0,T
[F
0,T
])
+
.
110 " P " (110 e
-.1
- 108 e
-.1
)
+
= 1.81.
Comment: While McDonald does not mention this in Derivatives Markets, for a European put,
P # Ke
-rt
= 110 e
-.1
= 99.53.
2013-MFE/3F, Financial Economics 5 Bounds on Premiums Euro., HCM 11/13/13, Page 148
5.5. C. S
0
" C(S
0
, K, T) " (PV
0,T
[F
0,T
] - PV
0,T
[K])
+
.
100 " C " (108 e
-.1
- 105 e
-.1
)
+
= 2.72.
Comment: While McDonald does not mention this in Derivatives Markets, for a European call,
C #

F
0,T
P
(S
0
) = PV
0,T
[F
0,T
] = 108 e
-.1
= 97.72.
5.6. C. PV
0,T
[K] - PV
0,T
[F
0,T
] = K e
-rT
- S
0
e
-#T
= 90 e
-.04
- S
0
= 86.47 - S
0
.
90 " P " (86.47 - S
0
)
+
.
5.7. A. PV
0,T
[F
0,T
] - PV
0,T
[K] = S
0
e
-#T
- K e
-rT
= 80 e
-.06
- K e
-.3
= 75.34 - .741 K.
80 " C(S
0
, K, T) " (75.34 - .741 K)
+
.
20 40 60 80 100 120 140
K
20
40
60
80
C
20 40 60 80 100 120 140
K
20
40
60
80
C
2013-MFE/3F, Financial Economics 5 Bounds on Premiums Euro., HCM 11/13/13, Page 149
5.8. As the current stock price increases, the probability that the call will be in the money at
expiration approaches one. The payoff at expiration will be S
T
- K.
The price for this is:

F
0,T
P
[S] - K e
-rT
.
Comment: Note that if it is positive,

F
0,T
P
[S] - K e
-rT
is the lower bound for the call premium.
As S
0
increases, the call premium approaches this lower bound.
For example, for a 2-year 100-strike call, with reasonable other inputs, here is the call premium from
the Black-Scholes formula versus this lower bound, as a function of the initial stock price, S
0
:
120 140 160 180 200
S 0
20
40
60
80
100
C
Black-Scholes
Lower Bound
2013-MFE/3F, Financial Economics 5 Bounds on Premiums Euro., HCM 11/13/13, Page 150
Section 6, Options on Currency
For example, one may be able today to buy one Euro for 1.25 U.S. Dollars.
1% = $1.25. 3 0.8% = $1. Instead one could buy one U.S. Dollar for 0.8 Euros.
Let r
$
be the interest rate on dollars and r
%
be the interest rate on Euros.
Then the forward price in dollars for one Euro in the future is: (current exchange rate) exp[(r
$
- r
%
)T].
117

The forward price in Euros for one dollar in the future is: (current exchange rate) exp[(r
%
- r
$
)T].
118

In general, one can exchange one currency for another currency at an exchange rate.
119

However, exchange rates change over time. Therefore, it is useful for businesses engaged in
international trade to buy and sell options related to currencies.
Dollar Denominated Options:
For example, one could purchase a call option to buy one euro in 2 years for $1.20.
The owner of this call would have the option in 2 years to use $1.20 to obtain one Euro.
This is an example of a dollar-denominated option, since the strike price and premium (cost) are in
dollars. A dollar-denominated call on Euros would give one the option to obtain Euros at some time
in the future for a specified number of dollars.
If the price of one euro in two years is more than $1.20, then one would exercise the above option.
For example if the exchange rate two years from now is $1.50 per Euro, then one would make
$1.50 - $1.20 = $0.30 by exercising this call.
A dollar-denominated put on Euros would give one the option to sell Euros at some time in the
future for a specified number of dollars. For example, a 2 year $1.20 strike put on Euros would give
its owner the option in 2 years to sell one Euro for $1.20. The owner would exercise this put if
2 years from now the exchange rate is less than $1.20 per Euro.
The payoff of the call option is: (x
2
- $1.20)
+
, where x
2
is the exchange rate in two years.
The payoff of the put option is: ($1.20 - x
2
)
+
, where x
2
is the exchange rate in two years.
The payoff on a call on currency has the same mathematical expression as for a call on stocks, with
x
T
substituted for S
T
: (x
T
- K)
+
. Similarly, the payoff on a put on currency is: (K - x
T
)
+
.
117
See Equation 5.18 in Derivatives Markets by McDonald, on the syllabus of an earlier exam.
118
Since the current exchange rates are reciprocals of each other, so are the forward prices.
119
For current exchange rates see for example www.xe.com/ucc/
2014-MFE, Financial Economics 6 Options on Currency HCM 11/13/13, Page 151
Put-Call Parity for Dollar Denominated Options:
The same put-call parity relationship holds as was discussed previously:
C(K, T) = P(K, T) + PV[F
0,T
] - PV[K].
If the option is dollar-denominated, then the strike price K is in dollars. We discount it back to the
present using the interest rate for dollars r
$
, usually just written as r in the United States.
The present value of one euro at time T, PV[F
0,T
], is the amount we would need to invest now in a
euro-denominated risk free bond in order to have 1 euro at time t. A euro-denominated risk free
bond earns interest continuous at a rate of r
%
.
120

If we spend x
0
exp[-Tr
%
] dollars to buy exp[-Tr
%
] euros at time 0, and invest in a
euro-denominated risk free bond, we will have 1 euro at time T.
Therefore, PV[F
0,T
] = x
0
exp[-Tr
%
].
Therefore, for dollar-denominated options on euros:
C
$
( x
0
, K, T) = P
$
( x
0
, K, T) + x
0
exp[-Tr
$
] - K e
-Tr
.
121

x
0
is the exchange rate for Euros in terms of dollars, $1.25 = 1 Euro, and K is in dollars.
This is analogous to what we had for options on stock: C(K, T) = P(K, T) + S
0
e
-T#
- K e
-Tr
.
x
0
3 S
0
. x
0
is the price of the asset euros. 3 S
0
is the price of the asset stock.
Dollars act as money: r
$
3 r.
Euros act as the asset: r
$
3 #.
Exercise: x
0
= $1.25/% . T = 2 years. K = $1.20/% .
The dollar-denominated interest rate is 6% and the euro-denominated interest rate is 4%.
The option premium for a dollar-denominated put on one euro is $0.10.
Determine the premium for a dollar-denominated call on one euro.
[Solution: $0.10 + ($1.25)e
-(2)(0.04)
- ($1.20)e
-(2)(0.06)
= $0.190.
Comment: Note how every term in the equation is in dollars. This is one way to check your work.]
120
We assume that r
$
will usually differ from r
%
. Similarly, each currency will have its own associated interest rate.
121
See equation 9.4 of Derivatives Markets by McDonald.
This is similar to the parity equation for stocks paying continuous dividends, with r
%
taking the place of #.
2014-MFE, Financial Economics 6 Options on Currency HCM 11/13/13, Page 152
Foreign Denominated Options:
A Euro-denominated call on dollars would give one the option to obtain dollars at some time in the
future for a specified number of Euros.
For example, a 2 year % 0.70 strike call on dollars would give its owner the option in 2 years to buy
one dollar for % 0.70. The owner would exercise this call if 2 years from now the exchange rate is
more than % 0.70 per dollar.
A Euro-denominated put on dollars would give one the option to sell dollars at some time in the
future for a specified number of Euros.
For example, a 2 year % 0.60 strike put on dollars would give its owner the option in 2 years to sell
one dollar for % 0.60. The owner would exercise this put if 2 years from now the exchange rate is
less than % 0.60 per dollar.
Put-Call Parity for Foreign Denominated Options:
For Euro-denominated options on dollars:
C
$
( x
0
, K, T) = P
$
( x
0
, K, T) + x
0
exp[-T r
$
] - K exp[-T r
$
].
x
0
is the exchange rate for dollars in terms of Euros, 0.8 Euro = $1, and K is in Euros.
This is analogous to what we had for options on stock: C(K, T) = P(K, T) + S
0
e
-T#
- K e
-Tr
.
x
0
3 S
0
. x
0
is the price of the asset dollars. 3 S
0
is the price of the asset stock.
Euros act as money: r
$
3 r.
Dollars act as the asset: r
$
3 #.
Exercise: x
0
= 0.8% /$. T = 2 years. K = 0.833% /$.
The dollar-denominated interest rate is 6% and the euro-denominated interest rate is 4%.
The option premium for a euro-denominated put on one dollar is 0.08 euro.
Determine the premium for a euro-denominated call on one dollar.
[Solution: 0.08 euros + (0.8 euros)e
-(2)(0.06)
- (0.833 euros)e
-(2)(0.04)
= 0.0206 euro.
Comment: Since the options are euro-denominated, the roles of the two interest rates are reversed.
Here euros act as the currency, while dollars act as the asset. Therefore, the interest rate on dollars is
analogous to the dividend rate on a stock.
Note how every term in the equation is in euros. This is one way to check your work.]
2014-MFE, Financial Economics 6 Options on Currency HCM 11/13/13, Page 153
Different Points of View:
Assume that currently 1% = $1.25. 3 0.8% = $1.
Sam is in the United States. Sam has 1000 dollar denominated calls with strike price $1.30, giving
him the option one year from now to buy 1000 Euros for 1300 dollars.
Exercise: In one year, 1% = $1.4. What is the payoff on Sam!s calls?
[Solution: $1.4 - $1.30 = $0.10. For 1000 calls: (1000)($0.10) = $100.]
From Sam!s point of view, the future value of each of his calls in dollars is: (x
1
- 1.30)
+
.
Johann is in Germany. Johann has 1300 Euro denominated puts with strike price: 1/1.30 = 0.7692,
giving him the option one year from now to sell 1300 dollars for: (1300)(0.7692) = 1000 Euros.
Exercise: In one year, 1% = $1.4. What is the payoff on Johann!s puts?
[Solution: $1 = 1/1.4 = 0.7143 euros. 0.7692 - 0.7143 = 0.0549 euros.
(1300)(0.0549) = 71.4 euros.
Comment: At this future exchange rate, 71.4 euros = (71.4%) ($1.4/%) = $100.]
From Johann!s point of view, the payoff on each of his puts in euros is: (0.7692 - 1/x
1
)
+
,
where x
1
is the exchange rate to buy euros with dollars, and therefore 1/x
1
is the exchange rate to
buy dollars with euros. The payoff in dollars on each of Johann!s puts is:
x
1
(0.7692 - 1/x
1
)
+
= (0.7692x
1
- 1)
+
= (x
1
- 1.30)
+
/ 1.30.
Sam!s calls have value if x
1
> 1.30. Johann!s puts have value if 0.7682 > 1/x
1
3 x
1
> 1.30.
Their options have value under the same conditions.

If x
1
= 1.4, then as calculated above, Sam!s calls are worth $100 and Johann!s puts are worth 71.4
euros or $100 at this future exchange rate. Johann!s puts are worth the same as Sam!s calls.
Both Sam!s and Johann!s positions give them the option one year from now to use 1300 dollars to
obtain 1000 Euros. Therefore, their positions must be worth the same amount.
2014-MFE, Financial Economics 6 Options on Currency HCM 11/13/13, Page 154
Exercise: If each of Johann!s euro-denominated puts for one dollar with a strike price of 0.7692 euros
has a price of 0.0523 euros, determine the price of Sam!s dollar denominated one year call for one
euro, with a strike price of $1.30.
[Solution: Johann!s position costs (1300)(0.0523) = 68 euros.
Thus each of Sam!s calls should cost: 68 euros/1000 = 0.068 euros.
At the current exchange rate, this is: (0.068 euros)($1.25/ euro) = $0.085.
Comment: Sam!s position is worth: (1000)($0.085) = $85.]
In general, in order to replicate the dollar denominated call, one needs to buy K euro denominated
puts. Thus the value of the call is K times the value of the put. In order to put the price of the call in
dollars rather than euros, one must multiply by the current exchange rate x
0
.
C
$
( x
0
, K, T) = x
0
K P
f
(1/x
0
, 1/K, T).
122

Exercise: Use the above formula to solve the previous exercise.
[Solution: C
$
(1.25, 1.3, T) = (1.25) (1.3) P
%
(1/1.25, 1/1.3, T) = (1.25)(1.3)(0.0523) = $0.085.]
Calls from Sam!s point of view was equivalent to puts from Johann!s point of view.
A call from one point of view is a put from an opposite point of view.
Similarly, P
$
( x
0
, K, T) = x
0
K C
f
(1/x
0
, 1/K, T).
For a dollar denominated call, we have the option to use dollars to receive another currency.
For a dollar denominated put, we have the option to receive dollars by using another currency.
For a Euro denominated call, we have the option to use Euros to receive another currency.
For a Euro denominated put, we have the option to receive Euros by using another currency.
122
See equation 9.9 in Derivatives Markets by McDonald.
2014-MFE, Financial Economics 6 Options on Currency HCM 11/13/13, Page 155
Derivation of the Relationship of Dollar and Foreign Denominated Currency Options:
Let x(t) be the exchange rate at time t; x(t) is the value in dollars of one of the foreign currency
at time t.
123

The payoff at expiration of a K-strike dollar denominated call on a foreign currency is in dollars:
(x(T) - K)
+
.
Now at time T, this payoff would be equal to in the foreign currency:
(x(T) - K)
+
/ x(T) = (1 - K/x(T))
+
= K (1/K - 1/x(T))
+
.
The payoff at expiration of a 1/K-strike foreign denominated put on dollars is in that foreign currency:
(1/K - 1/x(T))
+
.
Therefore, both in the foreign currency at expiration:
K (payoff of 1/K-strike foreign denominated put on dollars) =
(payoff of K-strike dollar denominated call on foreign currency).
Therefore, K(premium of 1/K-strike foreign denominated put on dollars) =
(premium in foreign currency of K-strike dollar denominated call on foreign currency) =
(premium in dollars of K-strike dollar denominated call on foreign currency) / x
0
.
124

In other words, C
$
(x
0
, K, T) / x
0
= K P
f
(1/x
0
, 1/K, T).
% C
$
(x
0
, K, T) = x
0
K P
f
(1/x
0
, 1/K, T).
123
For example, the current exchange rate could be $0.8 per Euro.
124
For example, if one could pay 0.10 Euros to purchase the dollar denominated call on Euros, and the current
exchange rate were $0.8 per Euro, then one could also pay 0.10/0.8 = $0.125 to purchase this call.
2014-MFE, Financial Economics 6 Options on Currency HCM 11/13/13, Page 156
Examples of Using Currency Options for Hedging:
Aidan Math is an American consulting actuary. On January 1, Aidan takes a three month assignment
consulting for the Seguros Popular, a Mexican Insurance Company. Aidan will be paid a lump sum
for his work on March 31 at the completion of his assignment. Aidan will be doing all the work at his
office in the United States.
The exchange rate on January 1 is 10 pesos per U.S. dollar, or $0.10 U.S. per Mexican Peso.
If Seguros Popular agrees to pay Aidan 600,000 Pesos on March 31, then Aidan faces a risk of the
exchange rate changing. On January 1, 600,000 Pesos is equivalent to $60,000. If on March 31 the
exchange rate were 12 pesos per U.S. dollar, then Aidan would receive only the equivalent of:
600,000/12 = $50,000.
Aidan would have the choice of just accepting this risk, or buying an option to hedge this risk.
Aidan could buy 60,000 peso denominated 3 month 10 peso European call options to buy
dollars.
125

126
Then on March 31, if the exchange rate is more than 10 peso per dollar, he can exercise
his options and use the 600,000 Pesos he is paid by Seguros Popular to buy $60,000.
If instead on March 31 the exchange rate were 8 pesos per U.S. dollar, then Aidan would receive
the equivalent of 600,000/8 = $75,000. In this case, Aidan would benefit from the movement in the
exchange rate. In general, if on March 31 the exchange rate were less than 10 pesos per U.S. dollar,
then the 600,000 pesos Aidan receives would be the equivalent of more than $60,000.
Instead of accepting this possibility of being lucky, Aidan could have sold 600,000 peso
denominated 3 month 10 peso European put options to buy dollars.
127
In that case, if the exchange
rate were less than 10 pesos per U.S. dollar, then the person who bought these puts from Aidan
would exercise them, and buy the 600,000 Pesos that Aidan receives from Seguros Popular for
$60,000.
128
Aidan would no longer benefit if the exchange rate were less than 10 pesos per U.S.
dollar, but in any case he would have the money from selling the puts. This money would help to
offset the cost of the calls he bought.
125
Equivalently, Aidan could buy 600,000 dollar denominated 3 month $0.1 European put options to sell pesos.
Then on March 31, if the exchange rate is more than 10 peso per dollar, in other words less than $0.1 per peso, he
can exercise his option and sell the 600,000 Pesos he is paid by Seguros Popular for $60,000.
126
As will be discussed subsequently, such a call could be priced using the Black-Scholes formula. If / = 12%, the
interest rate in pesos is 6%, and the interest rate in dollars is 5%, then the premium of each such call would be about
0.25 pesos. Thus 60,000 such calls would cost Aidan about 15,000 pesos on January 1, the equivalent of $1500.
127
Each such put would cost about 0.224 pesos, and Aidan would receive about $1460. Thus if Aidan both buys
the calls and sells the puts, then for a net cost of about $1500 - $1460 = $40, he would be unaffected by changes in
currency exchange rates. In this case, the calls cost more than the puts, since I have assumed the interest rate in
pesos is greater than the interest rate in dollars.
128
For example, if the exchange rate on March 31 were 8 pesos per U.S. dollar, obtaining 600,000 pesos would
normally cost 600,000/8 = $75,000. Using the options bought from Aidan, this person could instead obtain
600,000 pesos for only $60,000.
2014-MFE, Financial Economics 6 Options on Currency HCM 11/13/13, Page 157
Since Aidan will be doing all the work at his office in the United States, he would prefer to be paid in
dollars. If Seguros Popular agrees to pay Aidan $60,000 on March 31, then Seguros Popular rather
than Aidan would face a risk of the exchange rate changing. In this case, if on March 31 the exchange
rate were 12 pesos per U.S. dollar, then Seguros Popular would have to use: (12)(60000) =
720,000 pesos in order to pay $60,000 to Aidan.
Similar to the previous situation, Seguros Popular would have the choice of just accepting this risk, or
buying an option to hedge this risk. Seguros Popular could buy 60,000 peso denominated 3 month
10 peso European call options to buy dollars.
129
Then on March 31, if the exchange rate is more
than 10 peso per dollar, Seguros Popular can exercise its options and use the 600,000 Pesos to
buy $60,000, with which to pay Aidan. If on March 31 the exchange rate is less than 10 peso per
dollar, then Seguros Popular would benefit from the exchange rate movement; they would need to
use less than 600,000 pesos in order to pay Aidan $60,000.
Rosetta Stone is a Mexican pension actuary. On January 1, Rosetta takes a six month assignment
consulting for the Grate American Cheese Company, which is based in the United States. Rosetta
will be paid a lump sum for her work on June 30 at the completion of her assignment. Rosetta will
be doing all the work at her office in Mexico.
The exchange rate on January 1 is 10 pesos per U.S. dollar, or $0.10 U.S. per Mexican Peso.
Exercise: Grate American Cheese Company has agreed to pay Rosetta $100,000. Briefly
discuss the currency exchange risk faced by Rosetta and how she can hedge it using options.
[Solution: If the exchange rate is less than 10 pesos per dollar, then the $100,000 Rosetta will be
paid is worth less than the 1 million pesos she expected. Rosetta could buy 100,000 peso
denominated 6 month 10 peso-strike European put options to sell dollars. Alternately, Rosetta
could buy 1 million dollar denominated 6 month $0.1-strike European call options to buy pesos.]
Exercise: Grate American Cheese Company has agreed to pay Rosetta 1 million pesos. Briefly
discuss the currency exchange risk faced by the company and how to hedge it using options.
[Solution: If the exchange rate is less than 10 pesos per dollar, then the 1 million pesos Rosetta will
be paid will cost the company more than the $100,000 it expected. The company could buy 1
million dollar denominated 6 month $0.1-strike peso European call options to buy pesos.
Alternately, it could buy 100,000 peso denominated 6 month 10 peso-strike European put options
to sell dollars.
Comment: In this and the previous exercise, Rosetta and the company each face the same risk, and
can hedge that risk the same way. If the company had agreed to pay Rosetta $50,000 and
500,000 pesos, then Rosetta and the company would share the currency exchange risk.]
129
Equivalently, Popular could buy 600,000 dollar denominated 3 month $0.1 European put options to sell pesos.
2014-MFE, Financial Economics 6 Options on Currency HCM 11/13/13, Page 158
Problems:
6.1 (2 points) The spot exchange rate of dollars per euro is 1.1.
Dollar and euro interest rates are 5.0% and 6.0%, respectively.
The price of a $0.93 strike 18-month call option is $0.28.
What is the price of a similar put?
A. Less than $0.05
B. At least $0.05, but less than $0.10
C. At least $0.10, but less than $0.15
D. At least $0.15, but less than $0.20
E. At least $0.20
6.2 (2 points) Currently one can buy one dollar for 120 yen.
Dollar and yen interest rates are 4.5% and 3.0%, respectively.
The price of a 100 yen strike 2 year put option is 13 yen.
What is the price in yen of a similar call?
A. Less than 15
B. At least 15, but less than 20
C. At least 20, but less than 25
D. At least 25, but less than 30
E. At least 30
6.3 (2 points) Currently one can buy one Swiss Franc for 0.40 British Pounds (0.4).
The interest rate for Swiss Francs is 5%.
The interest rate for British Pounds is 3%.
The price of a 0.5 strike 3 year call option is 0.05.
Determine the price in Swiss Francs of a 2 Swiss Franc strike 3 year put option.
A. 0.04 B. 0.05 C. 0.10 D. 0.125 E. 0.25
6.4 (2 points) The price of a $0.02 strike 1 year call option on an Indian Rupee is $0.00565.
The price of a $0.02 strike 1 year put option on an Indian Rupee is $0.00342.
Dollar and rupee interest rates are 4.0% and 7.0%, respectively.
How many dollars does it currently take to purchase one rupee?
A. 0.020 B. 0.021 C. 0.022 D. 0.023 E. 0.24
6.5 (2 points) The current exchange rate is $1.90 per British Pound.
A $2 strike 6 month call on the British Pound currently costs $0.0554.
What is the price in pounds of a 0.5 strike 6 month put on United States dollars?
A. 0.010 B. 0.015 C. 0.020 D. 0.025 E. 0.030
2014-MFE, Financial Economics 6 Options on Currency HCM 11/13/13, Page 159
6.6 (2 points) Currently one can buy 1 Brazilian Real for $0.47.
Dollar and Real interest rates are 4.0% and 7.0%, respectively.
The price of a $0.50 strike 6-month call option to buy 1 Brazilian Real is $0.09.
What is the price of a similar put?
A. Less than $0.05
B. At least $0.05, but less than $0.10
C. At least $0.10, but less than $0.15
D. At least $0.15, but less than $0.20
E. At least $0.20
6.7 (2 points) The current exchange rate is $0.15 per Chinese Yuan Renminbi.
An 8 yuan strike 2 year call on the U.S. Dollar currently costs 0.568 yuan.
What is the price in dollars of a $0.125 strike 2 year put on yuan?
A. Less than $0.006
B. At least $0.006, but less than $0.008
C. At least $0.008, but less than $0.010
D. At least $0.010, but less than $0.012
E. At least $0.012
6.8 (2 points) Currently one can buy 1 U.S. Dollar for 11 Mexican Pesos.
Dollar and Peso interest rates are 4.0% and 6.0%, respectively.
The price of a 12 Peso strike 1-year put option is 1.36 Pesos.
What is the price in Pesos of a similar call?
A. Less than 0.60
B. At least 0.60, but less than 0.70
C. At least 0.70, but less than 0.80
D. At least 0.80, but less than 0.90
E. At least 0.90
6.9 (3 points) The spot exchange rate of dollars per South Korean Won is 0.00097.
Dollar and Won interest rates are 4.0% and 7.0%, respectively.
The price for one million $0.00100 strike 3 year call options on Won is $40.11.
What is the price in Won of a 1000 Won strike 3 year call on dollars?
A. 135 B. 140 C. 145 D. 150 E. 155
2014-MFE, Financial Economics 6 Options on Currency HCM 11/13/13, Page 160
6.10 (3 points) The spot exchange rate of yen per Euro is 125.
You are given the following premiums for five year Euro denominated calls on yen.
Strike Premium
%0.004 %0.003618
%0.006 %0.002132
%0.008 %0.001072
%0.010 %0.000482
%0.012 %0.000203
%0.014 %0.000083
Determine the premium for a five year 100 yen strike put on Euros.
A. 5 B. 6 C. 9 D. 13 E. 17
6.11 (3 points) You are given:
(i) The current exchange rate is 1.16%/.
(ii) A two-year European call option on Euros with a strike price of 0.80 sells for 0.120.
(iii) The continuously compounded risk-free interest rate on British Pounds is 5%.
(iv) The continuously compounded risk-free interest rate on Euros is 3%.
Calculate the price of 1000 two-year European call options on British Pounds with a strike price of
%1.25.
(A) %44 (B) %46 (C) %48 (D) %50 (E) %52
2014-MFE, Financial Economics 6 Options on Currency HCM 11/13/13, Page 161
6.12 (CAS5B, 11/93, Q.36) (3.5 points) A U.S. construction company, XYZ Inc., is placing a bid
to build a commercial office in Germany. If accepted, XYZ will be paid its bid (in marks) one week
from today. Assume the following:
XYZ's revenue from the project must be $200,000 to break even.
The one-week forward rate is $0.50 per mark.
Today's spot rate is $0.50 per mark.
A finance manager at XYZ bids 400,000 marks. She wants to insure that no losses from currency
fluctuations can happen if the bid is accepted.
a. (1 point) Discuss the current risk to the manager's company that can occur by the end of the week.
b. (1 point) Advise the manager on how to avoid the losses from currency fluctuations by using a
single option entered into on the day of the bid, ignoring the actual cost of the option and the
time value of money.
c. (1.5 points) Demonstrate through formulas and, when possible, actual amounts how the goal is
achieved under the possible outcomes facing the company.

6.13 (CAS5B, 5/99, Q.14) (1 point) An American manufacturer has contracted to sell a large order
of widgets for one million Canadian dollars, with payment due on delivery in six months.
How could the American company reduce foreign exchange risk?
1. Borrow Canadian currency against its receivables, convert to U.S. dollars at the spot rate,
and invest proceeds in the U.S.
2. Buy an option to sell Canadian dollars in six months at a specific price.
3. Buy Canadian currency forward.
A. 1 B. 1, 2 C. 1, 3 D. 2, 3 E. 1, 2, 3
6.14 (CAS3, 11/07, Q.15) (2.5 points)
A nine-month dollar-denominated call option on euros with a strike price of $1.30 is valued at $0.06.
A nine-month dollar-denominated put option on euros with the same strike price is valued at 0.18.
The current exchange rate is $1.2/euro and the continuously compounded risk-free rate on dollars is
7%. What is the continuously compounded risk-free rate on euros?
A. Less than 7.5%
B. At least 7.5%, but less than 8.5%
C. At least 8.5%, but less than 9.5%
D. At least 9.5%, but less than 10.5%
E. At least 10.5%
2014-MFE, Financial Economics 6 Options on Currency HCM 11/13/13, Page 162
6.15 (MFE/3F, 5/09, Q.9) (2.5 points) You are given:
(i) The current exchange rate is 0.011$/.
(ii) A four-year dollar-denominated European put option on yen with a strike price of $0.008
sells for $0.0005.
(iii) The continuously compounded risk-free interest rate on dollars is 3%.
(iv) The continuously compounded risk-free interest rate on yen is 1.5%.
Calculate the price of a four-year yen-denominated European put option on dollars with a strike price
of 125.
(A) 35 (B) 37 (C) 39 (D) 41 (E) 43
2014-MFE, Financial Economics 6 Options on Currency HCM 11/13/13, Page 163
Solutions to Problems:
6.1. C. C
$
(x
0
, K, T) = P
$
(x
0
, K, T) + x
0
exp[-Tr
%
] - K e
-Tr
.
0.28 = P + (1.1)e
-(1.5)(.06)
- (.93)e
-(1.5)(.05)
. % P = $0.137.
6.2. D. C
Y
(x
0
, K, T) = P
Y
(x
0
, K, T) + x
0
exp[-Tr
$
] - K exp[-Tr
Y
] =
13 + (120)e
-(2)(.045)
- (100)e
-(2)(.03)
= 28.5 yen.
Comment: We have a yen denominated option, and therefore yen act as money, while dollars act
as the asset. r
Y
3

r = return on money
.
r
$
3 # = return on asset (stock).
This is the reverse of what we would do for a dollar denominated option.
6.3. E. C
a
(x
0
, K, T) = x
0
KP
b
(1/x
0
, 1/K, T).
0.05 = (0.4/ Swiss Franc) 0.5 P. P = (.05)(1/.4)/.5 = 0.25 Swiss Francs.
Alternately, the call and the put both have value if three years from now one Swiss Franc is worth
more than 0.5. For example, assume that in 3 years one Swiss Franc is worth 0.6;
in other words, 1 is worth 1.667 Swiss francs. Then the call is worth 0.10, and the put is worth
1/3 Swiss Franc. At that future exchange rate, 1/3 Swiss Franc = 0.2. Thus the put is worth twice the
call. This will be true for other future exchange rates, and thus the put should cost twice the price of
the call. (2)(0.05) = 0.10. At the current exchange rate 0.10 = .1/.4 = 0.25 Swiss Francs.
6.4. D. C
$
(x
0
, K, T) = P
$
(x
0
, K, T) + x
0
exp[-Tr
R
] - K e
-Tr
.
0.00565 = 0.00342 + x
0
e
-.07
- .02e
-.04
. % x
0
= $0.0230.
6.5. B. C
$
(x
0
, K, T) = x
0
K P
f
(1/x
0
, 1/K, T).
$0.0554 = (1.90)(2) P

. % P

= 0.0146 pounds.
Alternately, Sam buys 10,000 calls.
In six months, Sam will have the option to buy 10,000 pounds for $20,000.
Sam!s position is worth (10,000)($0.0554) = $554.
Nigel buys 20,000 puts.
In six months, Nigel will have the option to sell $20,000 for 10,000 pounds.
Sam and Nigel!s positions are equivalent.
Nigel!s position must be currently worth $554 or 554/1.90 = 291.58 pounds.
Thus, each of Nigel!s puts must be worth: 291.58 pounds / 20,000 = 0.0146 pounds.
2014-MFE, Financial Economics 6 Options on Currency HCM 11/13/13, Page 164
6.6. C. C
$
(x
0
, K, T) = P
$
(x
0
, K, T) + x
0
exp[-Tr
R
] - K e
-Tr
.
0.09 = P + (0.47)e
-(0.5)(.07)
- (0.50)e
-(0.5)(.04)
. % P = $0.126.
Comment: x
0
is the number of dollars one currently has to spend in order to buy one Brazilian Real.
6.7. D. P
$
(x
0
, K, T) = x
0
K C
f
(1/x
0
, 1/K, T) = (.15)(.125)(.568) = $0.01065.
Alternately, Ping buys 1000 calls.
In two years, Ping will have the option to buy 1000 dollars for 8000 yuan.
Ping!s position is worth: (1000)(0.568) = 568 yuan.
Sam buys 8000 puts.
In two years, Sam will have the option to sell 8000 yuan for: (8000)(0.125) = 1000 dollars.
Sam and Ping!s positions are equivalent.
Sam!s position must be currently worth 568 yuan or (568)(0.15) = $85.2.
Thus, each of Sam!s puts must be worth: $85.2 / 8000 = $0.01065.
6.8. B. C
P
(x
0
, K, T) = P
P
(x
0
, K, T) + x
0
exp[-Tr
$
] - K exp[-Tr
P
].
= 1.36 + (11)e
-0.04
- (12)e
-0.06
= 0.63 Pesos.
Comment: Since these are peso denominated options, r
Peso
acts like r, and r
$
acts like f
foreign
.
r
Peso
3

r = return on money

r
$
3 # = return on asset (stock)
This is the reverse of what we would do for a dollar denominated option.
x
0
is the price of the asset dollars. 3 S
0
is the price of the asset stock.
Note that all terms in the equation are in pesos.
2014-MFE, Financial Economics 6 Options on Currency HCM 11/13/13, Page 165
6.9. C. By put call parity, C = P + S e
-#T
- K e
-rT
.
Apply parity to the dollar denominated call in order to get the premium for the similar dollar
denominated put:
40.11/1,000,000 = P + (0.00097) exp[-(.07)(3)] - (0.00100) exp[-(.04)(3)]. % P = $0.0001408.
Now use the premium for the dollar denominated put in order to get the premium for the
corresponding Won denominated call. P
$
(x
0
, K, T) = x
0
K C
f
(1/x
0
, 1/K, T).
Therefore, 0.0001408 = (0.00097)(0.00100) C
f
. % C
f
= 145 Won.
Alternately, use the premium for the dollar denominated call in order to get the premium for the
corresponding Won denominated put. C
$
(x
0
, K, T) = x
0
K P
f
(1/x
0
, 1/K, T).
Therefore, 40.11/1,000,000 = (0.00097)(0.00100) P
f
. % P
f
= 41.35 Won.
Now apply parity to the Won denominated put in order to get the premium for the similar Won
denominated call.
C = 41.35 + (1/0.00097)exp[-(.04)(3)] - (1000) exp[-(.07)(3)] = 145 Won.
Comment: Similar to MFE 5/09, Q.9.
Note that the strike for the Won denominated options is: 1 / 0.001 = 1000 Won.
The spot exchange rate of South Korean Won per dollar is: 1 / 0.00097 = 1030.9.
A diagram of the two different paths to a solution:
$ denominated call $ denominated put
Won denominated put Won denominated call
Sam-Johann
Put-Call Parity
Put-Call Parity
Sam-Johann
2014-MFE, Financial Economics 6 Options on Currency HCM 11/13/13, Page 166
6.10. B. A Euro denominated call on yen is similar to a yen denominated put on Euros.
A 100 yen strike put on Euros, gives one the opportunity to sell 1 Euro for 100 yen.
We can obtain 100 yen for 1 Euro.
This is the same as 100 calls in which we can pay 1/100 = 0.01 Euros per yen.
These one hundred 0.01 Euro strike calls on yen cost: (100)(0.000482 yen) = 0.0482 yen.
At the current exchange rate this is: (125)(0.0482) = 6.025 yen.
Alternately, P
yen
(x
0
, K, T) = x
0
K C
Euro
(1/x
0
, 1/K, T) = (125)(100)C
Euro
(1/125, 1/100, 5)
= 12,500 C
Euro
(0.08, 0.010, 5) = (12,500)(0.000482) = 6.025 yen.
6.11. B. One Euro currently costs 1/1.16 British Pounds.
By put-call parity, (with Pounds acting as money and Euros acting as the asset,) a two-year
pound-denominated European put option on Euros with a strike price of 0.80 sells for:
0.120 - (1/1.16) exp[-(2)(3%)] + 0.80 exp[-(2)(5%)] = 0.0320.
C
%
(x
0
, K, T) = x
0
K P

(1/x
0
, 1/K, T) = (1.16)(1.25)(0.0320) = %0.0464. (1000)(0.0464) = $46.4.
Alternately, P
%
(x
0
, K, T) = x
0
K C

(1/x
0
, 1/K, T) = (1.16)(1.25)(0.120) = %0.174.
By put-call parity, (with Euros acting as money and pounds acting as the asset,) a two-year
Euro-denominated European call option on British Pounds with a strike price of %1.25 sells for:
0.174 + 1.16 exp[-(2)(5%)] - 1.25 exp[-(2)(3%)] = %0.0464. (1000)(0.0464) = $46.4.
Comment: Similar to MFE/3F, 5/09, Q.9.
2014-MFE, Financial Economics 6 Options on Currency HCM 11/13/13, Page 167
6.12. (a) By the end of the week the exchange rate may change. If the bid is accepted, the
400,000 marks XYZ will be paid will not be worth $200,000. XYZ is concerned that the exchange
rate will go down, for example to $0.45 per mark, in which case the 400,000 marks will be worth less
than $200,000.
(b) Buy mark-denominated calls on dollars to allow one to use marks to buy dollars.
Equivalently, buy dollar-denominated puts on marks, to allow one to sell marks for dollars.
The options should be at-the-money and have one week to expiration.
(c) There are four possibilities.
1. If XYZ does not get the bid, and the exchange rate increases, then it lets the options expire.
2. If XYZ does not get the bid, and the exchange rate decreases, then its options turn a profit.
3. If XYZ does gets the bid, and the exchange rate increases, then it lets the options expire.
4. If XYZ does gets the bid, and the exchange rate decreases, then it uses it options to convert the
400,000 marks it receives into $200,000.
Comment: Since the creation of the Euro, there are no longer German marks.
In any case, you should take this simplified example with a grain of salt.
A forward contract to buy dollars for marks at $0.50 per mark would not do as good a job of
hedging. If XYZ does not get the bid, and thus has no need to convert marks to dollars, and the
exchange rate increases, then it would lose money on the futures contract. For example, if the
exchange rate in one week were $0.55, then XYZ would have to buy marks at $0.55 in order to sell
them for $0.50 to the other party to the futures contract.
(The futures contract may just be settled monetarily. In any case, XYZ would lose money.)
Of course the puts or calls would cost money to buy.
6.13. B. Buying a Canadian currency forward would obligate you to buy a specified number of
Canadian dollars for a specified price in six months. The American company would reduce its foreign
exchange risk by selling a Canadian currency forward, which would obligate it to sell one million
Canadian dollars for a specified price in six months.
6.14. D. C = P + X
0
exp[-T r

] - K exp[-T r
$
].
0.06 = 0.18 + 1.2 exp[-0.75 r ] - 1.3 exp[-(0.75)(0.07)].
% exp[-0.75 r ] = 1.1135/1.2. % r = 9.97%.
Comment: Since this is a dollar-denominated option, euros are the asset analogous to stock, and
therefore the risk-free rate on euros is analogous to the dividend rate #.
2014-MFE, Financial Economics 6 Options on Currency HCM 11/13/13, Page 168
6.15. E. By put-call parity, (with dollars acting as money and yen acting as the asset,) a four-year
dollar-denominated European call option on yen with a strike price of $0.008 sells for:
$0.0005 + $0.011 exp[-(4)(1.5%)] - $0.008 exp[-(4)(3%)] = $0.003764.
C
$
(x
0
, K, T) = x
0
K P

(1/x
0
, 1/K, T). % 0.003764 = (0.011)(0.008) P

(1/0.011, 1/0.008, 4).


% P

(1/0.011, 1/0.008, 4) = 0.003764 / {(0.011)(0.008)} = 42.77.


Alternately, P
$
(x
0
, K, T) = x
0
K C

(1/x
0
, 1/K, T).
Thus, 0.0005 = (0.011)(0.008) C

(1/0.011, 1/0.008, 4).


% C

(1/0.011, 1/0.008, 4) = 0.0005 / {(0.011)(0.008)} = 5.68.


By put-call parity, (with yen acting as money and dollars acting as the asset,) a four-year
yen-denominated European put option on dollars with a strike price of 125 sells for:
5.68 + 125 exp[-(4)(1.5%)] - (1/0.011) exp[-(4)(3%)] = 42.77.
Alternately, Sam buys 1000 four-year dollar-denominated European call option on yen with a strike
price of $0.008, as determined previously selling for $0.003764 each.
This gives Sam the option to buy 1000 yen for $8.
Hiro buys 8 four-year yen-denominated European put options on dollars with a strike price of 125.
This gives Hiro the option to sell $8 for 1000 yen.
Since Sam and Hiro!s positions are equivalent, the cost of their portfolios must be the same.
Thus, each of Hiro!s puts must cost: $3.764/8 = $0.4705.
In yen, each of his puts costs: 0.4705/0. 011 = 42.77.
Comment: Note that 1/0.008 = 125.
Similar to Q. 9.7 in Derivatives Markets by McDonald.
2014-MFE, Financial Economics 6 Options on Currency HCM 11/13/13, Page 169
Section 7, Exchange Options
130

One can buy an option to exchange one asset for another.
For example, Steve is given the option to exchange one share of a competitor!s Stock B for one
share of the stock of his employer Stock A. This is a call option with underlying asset Stock A and
strike asset Stock B.
A call exchange option allows one to exchange one share of a Stock B (strike asset) for one share of
the stock of Stock A (underlying asset). So rather than a strike price K in dollars, we have the option
to use an asset, Stock B, in order to obtain Stock A.
Assuming a European call, then at time T, Steve would make the exchange if a share of Stock A is
worth more than a share of Stock B. The future value of the call is (S
T
- Q
T
)
+
, where S is price of the
underlying asset, Stock A, and Q is the price of the strike asset, Stock B.
Whether one will exercise the call depends on the relative value of the underlying asset and the
strike asset, both of which are random variables. If at expiration the underlying asset is worth more
than the strike asset, then it would be worthwhile to exercise the call.

For example, if the call expires in 2 years, among the many possible situations:
Price of Stock A Price of Stock B Exercise
(Underlying Asset) (Strike Asset) Exchange
2 Years from now 2 Years from now Call?
100 90 Yes
110 130 No
120 100 Yes
70 80 No
A similar put exchange option would instead allow us to sell stock A in exchange for Stock B, in other
words, get B in exchange for A.
130
See Section 9.2 of Derivatives Markets by McDonald. Also called an outperformance option.
2014-MFE, Financial Economics 7 Exchange Options HCM 11/13/13, Page 170
Put-Call Parity:
Buy one European call, sell a similar put, sell a prepaid forward contract on Stock A, and buy a
prepaid forward contract on Stock B.
131

At Time T, you will get a share of Stock B due to the forward contract you bought.
If S
T
" Q
T
, you will exercise your call and exchange your Share of Stock B for a Share of Stock A.
Then you will provide this share of Stock A to the person who bought a forward contract from you.
If instead S
T
< Q
T
, then the person who bought your put, will exchange a share of Stock A for your
share of Stock B. You will then provide this share of Stock A to the person who bought a forward
contract from you.
In either case, at time T you end up with nothing after all of the transactions.
Therefore, the correct price for this position is zero.

Let

F
0,T
P
(S) be the prepaid forward price of a share of Stock A.
Let

F
0,T
P
(Q) be the prepaid forward price of a share of Stock B.
We have shown that: 0 = C
Eur
(S
0
, Q
0
, T) - P
Eur
(S
0
, Q
0
, T) -

F
0,T
P
(S) +

F
0,T
P
(Q).
C
Eur
( S
0
, Q
0
, T) = P
Eur
( S
0
, Q
0
, T) +

F
0,T
P
(S) -

F
0,T
P
(Q).
132

133

If the current price of Stock A is $100 and the current price of Stock B is $120, and neither one pays
dividends, then

F
0,T
P
(S) = 100 and

F
0,T
P
(Q) = 120. Therefore,
C
Eur
(S
0
, Q
0
, T) = P
Eur
(S
0
, Q
0
, T) + 100 - 120 = P
Eur
(S
0
, Q
0
, T) - 20.
Exercise: Golden Goblin stock has a current price of $70, and will pay a dividend of $2 in 1 month.
Wolfram & Hart stock has a current price of $80, and will pay a dividend of $3 in 2 months.
A three month call on Golden Goblin with strike asset of Wolfram & Hart costs $0.89.
If r = 12%, determine the cost of the similar put.
[Solution: Prepaid forward price of Golden Goblin is: 70 - 2/exp[1%] = 68.02.
Prepaid forward price of Wolfram & Hart is: 80 - 3/exp[2%] = 77.06.
P = C +

F
0,T
P
(Q) -

F
0,T
P
(S) = 0.89 + 77.06 - 68.02 = $9.93.]
131
Under a prepaid forward contract, you pay now but receive the asset at a fixed point of time in the future.
132
See Equation 9.8 in Derivatives Markets by McDonald.
133
This put-call parity follows from the fact that: (S(T) - Q(T))
+
+ Q(T) = (Q(T) - S(T))
+
+ S(T).
See remark (iii) on MFE Sample Exam Q.55.
2014-MFE, Financial Economics 7 Exchange Options HCM 11/13/13, Page 171
We can display the payoffs in this general situation in a table:
134
At Expiration (TIme T)
Transaction Time 0 If S
T
# Q
T
if S
T
> Q
T
Buy Call (to use Q to get S) - C 0 S
T
- Q
T
Sell Put P S
T
- Q
T
0
Sell Prepaid Forward on S

F
0,T
P
(S) -S
T
-S
T
Buy Prepaid Forward on Q -

F
0,T
P
(Q) Q
T
Q
T
Total P - C +

F
0,T
P
(S) -

F
0,T
P
(Q) 0 0
Since the payoff at time T is always zero, unless P - C +

F
0,T
P
(S) -

F
0,T
P
(Q) = 0, there would be an
opportunity for arbitrage.
If instead the underlying asset pays dividends at a continuous rate #
S
and the strike asset pays
dividends at a continuous rate #
Q
, then

F
0,T
P
(S) = S
0
exp[-T#
S
] and

F
0,T
P
(Q) = Q
0
exp[-T#
Q
].
135

Therefore: C
Eur
( S
0
, Q
0
, T) = P
Eur
( S
0
, Q
0
, T) + S
0
exp[-T#
S
] - Q
0
exp[-T#
Q
].
Exercise: The current price of Stock A is $100 and the current price of Stock B is $120.
Stock A pays dividends at a continuous annual rate of 1%, while Stock B pays dividends at a
continuous annual rate of 2%. What is the difference in price between a 3 year European call option
to exchange Stock B for Stock A, and the similar put?
[Solution: C
Eur
(S
0
, Q
0
, 3) - P
Eur
(S
0
, Q
0
, 3) = (100)exp[-(3)(1%)] - (120)exp[-(3)(2%)] = -15.97.
Comment: In this case, the put is worth more than the similar call. Exchange Call: can use B to get A.
Exchange Put: can use A to get B. Since Stock B is currently worth much more than Stock A, the put
is more likely to be worth something at expiration than is the call.]
C
Eur
(S
0
, Q
0
, T) - P
Eur
(S
0
, Q
0
, T) =

F
0,T
P
(S
0
) -

F
0,T
P
(Q
0
).
Therefore, the premium of the put is equal to the premium of the similar call, if and only if the two
stocks have the same prepaid forward price.
134
See Table 9.2 in Derivatives Markets by McDonald.
135
While you pay for the prepaid forward contract now, you do not get the dividends that are paid on the stock
between now and time T, since you do not own the stock until time T.
2014-MFE, Financial Economics 7 Exchange Options HCM 11/13/13, Page 172
Note that this form of put-call parity includes the situations previously discussed as special cases.
For example, if the strike asset is money, specifically K dollars, then the dividend on money is
interest at rate r, and
C
Eur
(S
0
, K, T) = P
Eur
(S
0
, K, T) + S
0
exp[-T#
S
] - K exp[-Tr].
Parity is the observation that buying a European call and selling a European put with
the same strike price and time to expiration is equivalent to making a leveraged
investment in the underlying asset, less the value of cash payments to the underlying
asset over the life of the option.
136

Points of View:
As discussed previously, a call from one point of view is a put from another point of view.
Neither Stock A nor Stock B pay dividends.
Adam has a 2 year European call with underlying asset Stock A and strike asset Stock B.
Adam has the option 2 years from now to buy a share of Stock A in exchange for a share of Stock
B. He will do so if two years from now Stock A is worth more than Stock B.
Eve has a 2 year European put with underlying asset Stock B and strike asset Stock A.
Eve has the option 2 years from now to sell a share of Stock B in exchange for a share of Stock A .
She will do so if two years from now Stock A is worth more than Stock B.
Two years from now they will each own whichever of the two stocks is worth more.
Adam and Eve have options with the same outcome and therefore the same value.
In the absence of dividends, C(S
0
, Q
0
, T) = P(Q
0
, S
0
, T).
Call to buy Stock A for price Stock B: A is the underlying asset and B is the strike asset.
Put to sell Stock B for price Stock A: B is the underlying asset and A is the strike asset.
Despite the different language, both give you the option to use Stock B to obtain Stock A.
136
See page 305 of Derivatives Markets by McDonald.
2014-MFE, Financial Economics 7 Exchange Options HCM 11/13/13, Page 173
Problems:
7.1 (2 points) Consider the case of a European exchange option in which the underlying stock is
Widget Co. with a current price of $65 per share. The strike asset is Gadget Inc., with a per share
price of $62. Neither stock will pay dividends within the next 3 months. Interest rates are 5% and the
3 month call option is trading for $8. What is the price of the similar put?
A. 3 B. 5 C. 7 D. 9 E. 11
7.2 (2 points) A 2 year European exchange call option has underlying asset one share of MGH
Shipping with a current price of $100 per share. The strike asset is 2 shares of Galactus Transport,
with a per share price of $60. One will have the option to use 2 shares of Galactus in order to obtain
one share of MGH. The price of this call option is $11.
MGH Shipping pays dividends at a continuous rate of 2% per year.
Galactus Transport pays dividends at a continuous rate of 1% per year.
What is the price of the similar put?
A. Less than $20
B. At least $20, but less than $25
C. At least $25, but less than $30
D. At least $30, but less than $35
E. At least $35
7.3 (2 points) A 3 year European exchange option has underlying asset one share of Peach
Computer Company with a current price of $200 per share. The strike asset is the Silicon Valley
Stock Index, with a current price of $210. The price of this call option is $13.
Peach Computer Company pays dividends at a continuous rate of 3% per year.
Silicon Valley Stock Index pays dividends at a continuous rate of 1% per year.
What is the price of the similar put?
A. Less than $20
B. At least $20, but less than $25
C. At least $25, but less than $30
D. At least $30, but less than $35
E. At least $35
7.4 (2 points) Consider four-year European exchange options involving the stocks of Global
Dynamics and Deon International. The price of an option to exchange a share of Global Dynamics
for a share of Deon International is the same as the price of an option to exchange a share of Deon
International for a share of Global Dynamics.
Global Dynamics has a current price of 92 and pays dividends at a continuous rate of 2% per year.
Deon International pays dividends at a continuous rate of 0.9% per year.
What is the current price of a share of Deon International?
A. 85 B. 86 C. 87 D. 88 E. 89
2014-MFE, Financial Economics 7 Exchange Options HCM 11/13/13, Page 174
7.5 (2 points) Daedalus stock has a current price of $65, and will pay a dividends of $1 in 2 months
and 5 months.
Hooper stock has a current price of $60, and will pay dividends of $1 in 1 month and 4 months.
A six month put on Daedalus stock with strike asset of Hooper stock costs $2.79.
If r = 6%, determine the cost of the similar call.
A. 7.00 B. 7.20 C. 7.40 D. 7.60 E. 7.80
2014-MFE, Financial Economics 7 Exchange Options HCM 11/13/13, Page 175
Solutions to Problems:
7.1. B. C
Eur
(S
0
, Q
0
, T) = P
Eur
(S
0
, Q
0
, T) +

F
0,T
P
(S
0
) -

F
0,T
P
(Q
0
).
8 = P + 65 - 62. % P = $5.
Comment: Since there are no dividends, the prepaid forward price is the same as the current stock
price. The interest rate has no impact.
7.2. D. C
Eur
(S
0
, Q
0
, T) = P
Eur
(S
0
, Q
0
, T) +

F
0,T
P
(S
0
) -

F
0,T
P
(Q
0
).
11 = P + 100exp[-(2)(2%)] - (2)(60)exp[-(2)(1%)]. % P = $32.54.
7.3. D. C
Eur
(S
0
, Q
0
, T) = P
Eur
(S
0
, Q
0
, T) +

F
0,T
P
(S
0
) -

F
0,T
P
(Q
0
).
13 = P + 200exp[-(3)(3%)] - 210exp[-(3)(1%)]. % P = $34.01.
7.4. D. From put-call-parity: C
Eur
(S
0
, Q
0
, T) = P
Eur
(S
0
, Q
0
, T) +

F
0,T
P
(S
0
) -

F
0,T
P
(Q
0
).
Therefore, the premium of the put is equal to the premium of the similar call, if and only if the two
stocks have the same prepaid forward price.
Prepaid forward price of Global Dynamics is: 92 exp[-(4)(.02)] = 84.93.
Prepaid forward price of Deon International is: Q
0
exp[-(4)(.009)].
Q
0
exp[-(4)(.009)] = 84.93. % Q
0
= 88.04.
7.5. E. Prepaid forward price of Daedalus is: 65 - 1/exp[1%] - 1/exp[2.5%] = 63.03.
Prepaid forward price of Hooper is: 60 - 1/exp[0.5%] - 1/exp[2%] = 58.02.
C = P +

F
0,T
P
(S) -

F
0,T
P
(Q) = 2.79 + 63.03 - 58.02 = $7.80.
2014-MFE, Financial Economics 7 Exchange Options HCM 11/13/13, Page 176
Section 8, Futures Contracts
137

As discussed previously, a forward contract is an agreement that sets the terms today,
including price and quantity, at which one will buy or sell an asset or commodity at a
specific time in the future. A futures contract is similar, except that the buyer and seller
post margin, the contract is marked to market, and the contract is typically traded on an
exchange.
138

An Example of a Futures Contract:
Frank is a farmer. He expects to harvest 10,000 bushels of wheat in July. On April 15, Frank enters
into a futures contract for $4 per bushel agreeing to deliver 10,000 bushels of wheat on July 15 to
Mom!s Bakery.
139
Frank has sold a futures contract.
140
Frank!s position would be referred to as the
short position; he has agreed to deliver wheat. Mom!s Bakery has bought a futures contract. Mom!s
Bakery has a long position; it has agreed to accept delivery of wheat at a predetermined price.
141

Frank has guarded against the risk of the price of wheat falling between April and July. Note that
Frank has not guarded against for example his crop being destroyed by hail prior to harvest.
Mom!s Bakery has guarded against the risk of the price of wheat rising between April and July.
There is no investment required to enter a futures contract; however, Frank and Mom!s Bakery must
each post margin with the broker who arranged the contract. The total value is $40,000, so for
example, 10% margin would require that they each post a $4000 margin with the broker.
Let us assume that on April 16, the futures price on July 15 delivery of wheat is $3.99.
Then Frank has made money on his contract, since he would in essence get $4.00 per bushel for his
wheat, instead of the $3.99 he would expect to get in the absence of his futures contract.
The contract is marked to market daily.
142
Thus Frank!s margin account will be increased by:
($0.01)(10,000) = $100, while Mom!s margin account will be decreased by $100.
Each of them earns one day of interest. Assuming r = 5%, they earn $4000(e
0.05/365
- 1) = $0.55.
137
See Section 5.4 of Derivatives Markets by McDonald, on the syllabus of a previous exam.
138
One can think of mark to market as follows, as the for ward price moves up or down the owner of the futures
contract either makes or loses money right away.
In contrast, for a forward contract the profit or loss is realized at expiration.
139
Futures contracts can be on other assets like gold, stock indices such as the S&P 500, currency such as yen, etc.
140
A contract would be for 5000 bushels, so Frank has actually sold two contracts.
141
Some futures contracts are settled by delivery, but most are closed out prior to the delivery date.
When you enter into a futures contract you are obligated to make payments if the forward price moves in the wrong
direction for you. Thus the effect of the movement in the forward price is captured monetarily on an ongoing basis.
142
Therefore, Frank will take his profit now, and the new futures price is $3.99.
2014-MFE, Financial Economics 8 Futures Contracts HCM 11/13/13, Page 177
Thus Frank!s margin account is now: $4000 + $100 + $0.55 = $4100.55.
Mom!s margin account is now: $4000 - $100 + $0.55 = $3900.55.
Exercise: On April 17 the futures price on July 15 delivery of wheat is $4.02.
What happens to Frank!s and Mom!s margin accounts?
[Solution: The one day move in price is an increase of $0.03.
Frank!s margin account has ($0.03)(10,000) = $300 subtracted.
He earns one day of interest of: $4100.55(e
0.05/365
- 1) = $0.56.
Frank!s margin account is now: $4100.55 - $300 + $0.56 = $3801.11.
Similarly, Mom!s margin account is now: $3900.55 + $300 + $0.54 = $4201.09.]
Options on Futures Contracts:
One can buy or sell calls and puts on futures contracts.
If a call is exercised, then the owner of the call acquires a long position in a futures contract with futures
price equal to the strike price of the call. The owner of the call if and when he exercises it, will enter
into an agreement to accept delivery of the commodity such as corn.
If a call is exercised, then the writer (seller) of the call acquires a short position in a futures contract with
futures price equal to the strike price of the call. The writer of the call if and when it is exercised, will
enter into an agreement to deliver the commodity such as gold.
If a put is exercised, then the owner of the put acquires a short position in a futures contract with
futures price equal to the strike price of the put. The owner of the put if and when he exercises it, will
enter into an agreement to deliver the commodity such as oil.
If a put is exercised, then the writer (seller) of the put acquires a long position in a futures contract with
futures price equal to the strike price of the put. The writer of the put if and when it is exercised, will
enter into an agreement to accept delivery of the commodity such as cattle.
An Example of Using Options on Futures:
We have seen how Frank can use a futures contract to guard against the risk of the price of wheat
falling between April and July. Frank could instead buy an option on such a futures contract.
143
For example, on March 15 Frank could purchase a 3 month put to sell a futures contract on 10,000
bushels of wheat for July delivery. This put would give Frank the option to sell in June such a futures
contract at a given strike price.
144

143
By buying an option on a futures contract Frank would get price protection without limiting his profit potential.
144
There are two important dates: when the option expires, May, and the delivery date in the futures contract, July.
One could instead of a European option have an American option, to be discussed subsequently. In that case, one
could exercise the option at any date up to expiration. Options on futures contracts are often American, with
expiration date one month before the delivery date of the commodity.
2014-MFE, Financial Economics 8 Futures Contracts HCM 11/13/13, Page 178
Assume for example the strike price were $4.00 per bushel. Then in June, Frank would have the
option to exercise his put and enter into an agreement to deliver wheat in July at $4.00 per bushel.
If the futures price in June were more than $4.00 per bushel, then Frank would not exercise his put. If
instead the futures price in June were less than $4.00 per bushel, then Frank would exercise his put.
For example, if the futures price in June were $3.80 per bushel, then Frank would exercise his put.
145

He would now have locked in a price of $4.00 per bushel for his wheat, which is better than the
$3.80 he could get by entering into a futures contract in June.
146

The payoff to Frank from the put would be: $4.00 - $3.80 = $0.20 per bushel. The payoff on his
put is: (Strike Price - Futures Price)
+
. This is the same payoff formula as for a put on stock, with the
futures price taking the place of the stock price. Thus even though there is no money required to
invest in a futures contract, one can apply the same mathematics to options on futures contracts as to
options on stocks, with appropriate modifications.
147

Such a put would protect Frank against the price of wheat declining between March and June. If such
a price decline occurs, Frank!s wheat is worth less, but his put is worth more.
Similarly, on March 15, Mom!s Bakery could purchase a 3 month $4 per bushel strike call to buy a
futures contract on 10,000 bushels of wheat for July delivery. This call would give Mom!s Bakery the
option to buy in June such a futures contract. In June, Mom!s would have the option to exercise its
call and enter into an agreement to accept delivery of wheat in July at $4.00 per bushel.
If the futures price in June were less than $4.00 per bushel, then Mom!s Bakery would not exercise
this call. If the futures price in June were more than $4.00 per bushel, then Mom!s Bakery would
exercise this call.
If for example the futures price in June were $4.10 per bushel, then Mom!s would exercise its call
and now have entered into a futures contract to receive wheat in July for $4.00 a bushel. Mom!s
would now have locked in a price of $4.00 per bushel for buying wheat, which is better than the
$4.10 it could get via a futures contract. The payoff to Mom!s from the call would be: 4.10 - 4.00 =
$0.10 per bushel. The payoff on this call is: (Futures Price - Strike Price)
+
. This is the same payoff
formula as for a call on stock, with the futures price taking the place of the stock price.
Such a call would protect Mom!s Bakery against the price of wheat increasing between March and
June. If such a price increase occurs, Mom!s Bakery would have to pay more for wheat, but the call
is worth more.
145
Alternately, Frank could make money by selling his put which would have increased in value.
146
In June, Frank could enter into a futures contract for July delivery of wheat with a $3.80 per bushel price. I
Instead using his put option on a futures contract, Frank can enter into a futures contract in June for July delivery of
wheat with a $4.00 per bushel price.
147
A form of put-call parity for options on future contracts will be discussed subsequently. In subsequent sections,
will be discussed how to price options on futures contracts, using either Binomial Trees or the Black-Scholes
formula, in a manner parallel to pricing options on stocks.
2014-MFE, Financial Economics 8 Futures Contracts HCM 11/13/13, Page 179
Advantages of Using Options on Futures rather than Options on the Underlying Asset:
148

Options on futures contracts are preferable to options on the underlying asset when it is cheaper
and/or more convenient to deliver/receive the futures contract on the asset rather than the asset itself.
It is easier to deliver or receive a futures contract on cattle than it is to deliver or receive the cattle.
The futures contract will usually be closed out prior to delivery, and therefore, options on futures
contracts are usually settled in cash.
In many situations options on futures contracts tend to have lower transaction costs than options on
the underlying asset.
Trading on futures and options on futures are usually conducted side by side on the same
exchange, which makes it easier to hedge, etc., and makes the markets more efficient.
Put-Call Parity for Options on Futures Contracts:
PV[F] = F
0,T
e
-rT
.
Therefore, the put-call parity relationship for options on futures contracts is:
C = P + F
0 , T
e
-rT
- Ke
-rT
.
The put-call parity relationship for options on futures contracts can be obtained from
that for options on stocks by: S
0
3 F
0 , T
, and # 3 r.
Exercise: On March 15 a 3 month $4 per bushel strike put on a futures contract on 10,000 bushels
of wheat for July 15 delivery has a premium of $815. (The buyer of this put would on June 15
have the option to sell, to the person who wrote the put, a futures contract on 10,000 bushels of
wheat for July 15 delivery at $4 per bushel.)
Currently, the futures price for July 15 delivery of wheat is $4.20 per bushel.
If r = 5%, what is the premium for the corresponding call?
[Solution: C = P + F
0,T
e
-rT
- Ke
-rT
=
$815 + (10,000)($4.20) e
-(0.05)(1/4)
- (10,000)($4.00) e
-(0.05)(1/4)
= $2790.]
148
See Options, Futures, and Other Derivatives by Hull, not on the syllabus.
2014-MFE, Financial Economics 8 Futures Contracts HCM 11/13/13, Page 180
An Example of Put-Call Parity for Options on Futures Contracts:
Here is an example that illustrates why r takes the place of # in put-call parity for options on futures
contracts.
149

Groucho buys from Chico a K-Strike T-Year put on a futures contract on corn.
Groucho sells to Harpo a K-Strike T-Year call on a futures contract on corn.
Groucho enters into a T-year futures contract; Zeppo agrees to deliver corn to Groucho in
T years for the futures price of F
0,T
(cash on delivery.)
Groucho also borrows Ke
-rT
and lends F
0,T
e
-rT
.
At time T, Groucho owes K for the money he borrowed.
At time T, when the options expire, let the price of corn be X.
At time T Groucho has F
0,T
from the money he lent.
He uses that to pay Zeppo for the corn, to fulfill the futures contract.
Groucho sells the corn for X.
At time T, if X > K, then Harpo exercises his call and buys a futures contract on corn from Groucho,
with delivery price of K. In other words, Harpo will have a long position in corn while Groucho will
have a short position in corn. Since the current price of corn is X > K, due to mark-to-market Groucho
pays Harpo X - K.
150

Taking into account the money he owes and the money he got for selling corn, Groucho now has:
X - K - (X - K) = 0.
If instead at time T we have X < K, then Groucho exercises his put and sells a futures contract on corn
to Chico, with delivery price of K. In other words, Chico will have a long position in corn while
Groucho will have a short position in corn. Since the current price of corn is X < K, due to
mark-to-market Chico pays Groucho K - X.
151
Taking into account the money he owes and the money he got for selling corn, Groucho now has:
X - K + (K - X) = 0.
In either case, Groucho ends up with zero.
Therefore, the value of his initial portfolio must be zero:
0 = P - C + F
0,T
e
-rT
- Ke
-rT
. % C = P + F
0,T
e
-rT
- Ke
-rT
.
149
The same substitution is made for Binomial Trees and Black-Scholes.
150
If Harpo were to get delivery of corn at price K, he could sell it for X > K, and make X - K.
151
If Groucho were to buy corn at the market price X, he could sell it to Chico for K > X, and make K - X.
2014-MFE, Financial Economics 8 Futures Contracts HCM 11/13/13, Page 181
Problems:
8.1 (2 points) A futures contract provides for the delivery of 1000 barrels of Light Sweet Crude Oil
in September.
In January, a 4 month $100 per barrel strike call on this futures contract has a premium of $3400.
In January, the futures price for delivery of oil in September is $95 per barrel.
If r = 6%, determine the premium for the corresponding put.
A. $7900 B. $8000 C. $8100 D. $8200 E. $8300
8.2 (2 points) The premium for a 6 month 1500 strike put on a futures contract is 53.
The current futures price is 1600.
If r = 4%, determine the premium for the corresponding call.
A. 130 B. 140 C. 150 D. 160 E. 170
8.3 (3 points) On January 1, Saul enters into a futures contract on 5000 bushels of soybeans at
$12.40 per bushel for delivery in August. Saul has a long position; he has agreed to accept delivery
of soybeans in August. Saul post a margin of 10%. The interest rate is 6%.
The futures prices for August delivery are on the following dates:
January 1 $12.40
January 2 $12.60
January 3 $12.55
January 4 $12.30
January 5 $12.40
Determine the amount in Saul!s margin account on each of these days.
8.4 (1 point) The premium for an at-the money call on a futures contract is $5. What is the premium
for an at-money put on the same futures contract with the same time until maturity?
2014-MFE, Financial Economics 8 Futures Contracts HCM 11/13/13, Page 182
Solutions to Problems:
8.1. E. P = C - F
0,T
e
-rT
+ Ke
-rT
=
$3400 - (1000)($95) e
-(.06)(1/3)
+ (1000)($100)e
-(.06)(1/3)
= $8301.
Comment: An airline might either enter into a futures contract or buy a call on a futures contract in order
to protect itself against a possible future rise in oil prices, which would lead to a rise in the price of jet
fuel.
8.2. C. C = P + F
0,T
e
-rT
- Ke
-rT
= 53 + (1600) e
-(.04)(1/2)
- (1500) e
-(.04)(1/2)
= 151.
8.3. Saul!s original margin account is: (10%)(5000)($12.40) = $6200.
When on January 2 the futures price increases $.20 per bushel he makes ($.20)(5000) = $1000.
(He would be getting soybeans for only $12.40 per bushel rather than the expected $12.60.)
He also made one days interest.
On January 2 his margin account is: $6200 exp[.06/365] + $1000 = $7201.02.
On January 3 his margin account is: $7201.02 exp[.06/365] + (-$.05)(5000) = $6952.20.
On January 4 his margin account is: $6952.20 exp[.06/365] + (-$.25)(5000) = $5703.34.
On January 5 his margin account is: $5703.34 exp[.06/365] + ($.10)(5000) = $6204.28.
Comment: See Table 5.8 in Derivatives Markets by McDonald, on the syllabus of an earlier exam.
Here Saul has a long position, so he benefits from increases in the future price. If instead he had a
short position, had agreed to deliver soybeans at a predetermined price, then he would benefit
from decreases in the futures price.
8.4. At-the-money means K = F
0,T
.
C = P + F
0,T
e
-rT
- Ke
-rT
. % 5 = P + F
0,T
e
-rT
- F
0,T
e
-rT
. % P = $5.
2014-MFE, Financial Economics 8 Futures Contracts HCM 11/13/13, Page 183
Section 9, Synthetic Positions
152
Put-call parity followed from creating a synthetic position which matched the cashflows of another
investment. We made use of the law of one price which states that two positions that
generate the exact same cashflows should have the same cost. Creating synthetic
positions can be useful to derive results, help ones understanding, and may also have practical
applications in investing.
Synthetic Share of Stock:
We can create a synthetic share of stock by buying a call, selling a put, lending the present value of
the strike price K, and lending the present value of any dividends.
Exercise: Show that this position is equivalent to buying the stock now and holding it until time T.
[Solution: If we buy and hold the stock, then we will receive the dividend payments and own the
stock at time T. In the above position, we will receive the dividend payments as that loan is repaid.
Also we will be paid back K at time T. If S
T
" K, then we use our call to buy a share for K.
If instead S
T
< K, then we buy a share of Stock for K from the person to whom we sold the put. In
either case, we own a share of stock at time T, as well as having received the dividend payments.]
This shows that S
0
= C
Eur
(K, T) - P
Eur
(K, T) + K e
-rT
+ PV[Div], one of the put-call parity
relationships discussed previously.
One way to remember how to create a given synthetic position is to write the put-call
parity relationship with the desired item alone on one side of the equation.
If the stocks pays dividends continuously, then S
0
= e
#T
{C
Eur
(K, T) - P
Eur
(K, T) + K e
-rT
}.
We can create a synthetic share of stock by: buying e
#T
calls, selling e
#T
puts, lending K e
-(r-#)T
.
152
See pages 268-269 of Derivatives Markets by McDonald.
2014-MFE, Financial Economics 9 Synthetic Positions HCM 11/13/13, Page 184
Synthetic Treasury Bill:
K e
-rT
+ PV[Div] = S
0
- C
Eur
(K, T) + P
Eur
(K, T).
Therefore, we can create a synthetic T-Bill by buying the stock, selling a call, and buying the put.
This is called a conversion. One is lending money.
If S
T
> K, then we sell our share for K to the person who bought the call. If instead S
T
# K, then we
sell a share of stock for K using our put. In either case, we have K at time T.
Thus for an investment now, we get K at time T. This is equivalent to a risk free Treasury Bill that
returns K at time T, plus we get the present value of any dividends on the stock.
Alternately, one could also borrow the present value of dividends.
This is equivalent to a risk free Treasury Bill that returns K at time T, since:
Ke
-rT
= S
0
- C
Eur
(K, T) + P
Eur
(K, T) - PV[Div].
Alternately, if dividends are paid continuously, then one could instead buy exp[-#T] shares of stock,
and reinvest the dividends. Then we would end up with one share of stock at time T. This is
equivalent to a risk free Treasury Bill that returns K at time T, since
Ke
-rT
= S
0
e
-#T
- C
Eur
(K, T) + P
Eur
(K, T).
One can create a synthetic short T-Bill position by shorting the stock, buying a call, and selling the
put. This is called a reverse conversion. One is borrowing money.
Synthetic Options:
We can create a synthetic call by: buying the stock, buying a put, borrowing the present value of the
strike price, and borrowing the present value of any dividends.

Exercise: Show that this position is equivalent to buying the call.
[Solution: We repay one loan as we receive the dividend payments.
If S
T
" K, then we sell our share for S
T
, repay the loan by paying K, and are left with S
T
- K.
If instead S
T
< K, then we use our put to sell our share for K, repay the loan by paying K, and are
left with 0. These are the same payoffs as if we had purchased the call.]
This shows that C
Eur
(K, T) = S
0
+ P
Eur
(K, T) - K e
-rT
- PV[Div], one of the put-call parity
relationships discussed previously.
2014-MFE, Financial Economics 9 Synthetic Positions HCM 11/13/13, Page 185
We can create a synthetic put by buying a call, shorting the stock, lending the present value of the
strike price, and lending the present value of any dividends.
Exercise: Show that this position is equivalent to buying the put.
[Solution: We borrow a share of stock, sell it for S
0
, and will give this person a share of stock when
the option expires. We also must pay this person the stock dividends they would have gotten on
the stock, when they would have gotten them. We pay the dividend payments from one of the
loans. If S
T
" K, then we buy a share for K, using our call and the money from the other loan.
After returning the share to the person we borrowed it from, we are left with nothing.
If instead S
T
< K, then we buy a share for S
T
. After returning the share to person we borrowed it
from, we are left with K - S
T
. These are the same payoffs as if we had purchased the put.]
This shows that P
Eur
(K, T) = C
Eur
(K, T) - S
0
+ K e
-rT
+ PV[Div], one of the put-call parity
relationships discussed previously.
In the case of continuous dividends, P
Eur
(K, T) = C
Eur
(K, T) - e
-#T
S
0
+ K e
-rT
.
Thus we can create a synthetic put by buying a call, shorting e
-#T
shares of stock,
and lending the present value of the strike price.
C
Eur
(K, T) = P
Eur
(K, T) + e
-#T
S
0
- K e
-rT
.
Thus we can create a synthetic call by buying a put, buying e
-#T
shares of stock,
and borrowing the present value of the strike price.
In the case of discrete dividends, C
Eur
(K, T) = P
Eur
(K, T) + S
0
- K e
-rT
- PV[Div].
Thus we can create a synthetic call by buying a put, buying a share of stock, borrowing the present
value of the strike price, and borrowing the present value of any dividends.
Synthetic Forward Contract:
153
We can create a synthetic forward contract by buying a call and selling a put.
If S
T
" K, then we buy a share for K, using our call.
If instead S
T
< K, then we buy a share for K, from the person to whom we sold the put.
Thus we have a forward contract to buy the stock at price K. This has present value PV
0,T
[F
0,T
- K].
Therefore, PV
0,T
[F
0,T
- K] = C
Eur
(K, T) - P
Eur
(K, T), as discussed previously.
153
See page 266 of Derivatives Markets by McDonald.
2014-MFE, Financial Economics 9 Synthetic Positions HCM 11/13/13, Page 186
Summary of Synthetic Positions:
Synthetic Stock (discrete dividends): Buy Call, Sell Put, Lend PV of Strike, Lend PV of Dividends.
Synthetic Stock (continuous dividends): Buy e
#T
Calls, Sell e
#T
Puts, Lend K e
-(r-#)T
.
Synthetic T-Bill (conversion): Buy Stock, Sell Call, Buy Put.
154

Synthetic Short T-Bill (reverse conversion): Sell Stock, Buy Call, Sell Put.
155

Synthetic Call (discrete dividends):
Buy Stock, Buy Put, Borrow PV of Strike, Borrow PV of Dividends.
Synthetic Call (continuous dividends): Buy e
-#T
shares of Stock, Buy Put, Borrow PV of Strike.
Synthetic Put (discrete dividends): Sell Stock, Buy Call, Lend PV of Strike, Lend PV of Dividends.
Synthetic Put (continuous dividends): Sell e
-#T
shares of Stock, Buy Call, Lend PV of Strike.
In each case, one way to remember how to create a given synthetic position is to write the put-call
parity relationship with the desired item alone on one side of the equation.
Synthetic Forward Contract on a Stock: Buy Call, Sell Put.
154
One could in addition borrow the present value of any dividends.
155
One could in addition lend the present value of any dividends.
2014-MFE, Financial Economics 9 Synthetic Positions HCM 11/13/13, Page 187
Problems:
9.1 (2 points) The price of a stock is $135. The stock pays dividends at the continuous rate of 2%.
The price of a European call with a strike price of $140 is $17.39 and the price of a European put
with a strike price of $140 is $19.14. Both options expire in eight months.
Calculate the annual continuously compounded risk-free rate on a synthetic T-Bill created using these
options.
A. Less than 3%
B. At least 3%, but less than 3%
C. At least 4%, but less than 4%
D. At least 5%, but less than 6%
E. At least 6%
9.2 (2 points) Consider options on a non-dividend paying stock.
The price of a 3 year European call with a strike price of $100 is $39.70.
The price of a 3 year European put with a strike price of $100 is $8.23.
r = 6%.
Calculate the price of a synthetic share of stock created using these options.
A. 100 B. 105 C. 110 D. 115 E. 120
9.3 (3 points) The stock of Rich Resorts has a current price of 150, and pays no dividends.
The stock of Cereal Growers has a current price of 80, and pays no dividends.
This coming summer will either be dry or wet.
If the summer is dry, then 4 months from now Rich Resorts stock price will be 180,
and Cereal Growers stock price will be 70.
If the summer is wet, then 4 months from now Rich Resorts stock price will be 130,
and Cereal Growers stock price will be 90.
What is the continuously compounded risk free rate?
A. 3.25% B. 3.50% C. 3.75% D. 4.00% E. 4.25%
9.4 (2 points) Consider options on a dividend paying stock.
The price of a 9-month European call with a strike price of $60 is $5.67.
The price of a 9-month European put with a strike price of $60 is $7.52.
r = 7%.
Using these options you create a synthetic 9-month forward contract on a share of stock.
Calculate the price, with payment on delivery, of this forward contract.
A. 55 B. 56 C. 57 D. 58 E. 59
2014-MFE, Financial Economics 9 Synthetic Positions HCM 11/13/13, Page 188
9.5 (CAS3, 5/07, Q.13) (2.5 points) The price of a non-dividend paying stock is $85.
The price of a European call with a strike price of $80 is $6.70 and the price of a European put with a
strike price of $80 is $1.60.
Both options expire in three months.
Calculate the annual continuously compounded risk-free rate on a synthetic T-Bill created using these
options.
A. Less than 1%
B. At least 1%, but less than 2%
C. At least 2%, but less than 3%
D. At least 3%, but less than 4%
E. At least 4%
2014-MFE, Financial Economics 9 Synthetic Positions HCM 11/13/13, Page 189
Solutions to Problems:
9.1. D. Using put call parity, C = P - Ke
-rT
+ Se
-#T
.
17.39 = 19.14 - 140 e
-r2/3
+ 135 e
-(.02)(2/3)
. % e
-r2/3
= 134.96/140. % r = 5.5%.
Comment: Similar to CAS3, 5/07, Q.13.
9.2. D. From put-call parity, P
Eur
(K, T) = C
Eur
(K, T) - S
0
+ K e
-rT
+ PV[Div].
8.23 = 39.70 - S
0
+ 100 e
-.18
+ 0. % S
0
= $115.00.
9.3. E. Assume we buy x shares of Rich Resorts and y shares of Cereal Growers.
Then after a dry summer our position is worth: 180x + 70y.
After a wet summer our position is worth: 130x + 90y.
Setting these two equal: 180x + 70y = 130x + 90y. % y = 2.5x.
Take for example x = 2 and y = 5. Then the portfolio costs: (2)(150) + (5)(80) = 700.
If the summer is dry, then 4 months from now the portfolio is worth: (2)(180) + (5)(70) = 710.
If the summer is wet, then 4 months from now the portfolio is worth: (2)(130) + (5)(90) = 710.
Thus we have a risk free return. 710/700 = e
r/3
. % r = 4.26%.
Comment: We want to create a risk free investment, a synthetic treasury bill.
What we were asked to determine, r, is the return on a such risk free bond.
We equate the value of the portfolio in the two possible future states.
In the case where we buy 2 shares of Rich Resorts and 5 shares of Cereal Growers, our portfolio
will be worth 710 in either state.
An investment that is always worth a fixed amount in the future is just like a risk free bond.
9.4. D. Prepaid Forward Price is: e
-#T
S
0
= C - P + e
-rT
K = 5.67 - 7.52 + 60 e
-(.07)(0.75)
= 55.08
Forward Price = e
rT
(prepaid forward price) = e
(.07)(0.75)
55.08 = 58.05.
9.5. A. Using put-call parity, C = P - Ke
-rT
+ Se
-#T
.
6.70 = 1.60 - 80 e
-r/4
+ 85 e
-0/4
. % e
-r/4
= 79.9/80. % r = 0.50%.
Comment: A synthetic T-Bill can be created by buying the stock, buying the put, and selling the call.
If the stock paid dividends, then one would need to buy a forward contract on the stock in order to
create a synthetic T-Bill.
2014-MFE, Financial Economics 9 Synthetic Positions HCM 11/13/13, Page 190
Section 10, American Options
An American style option may be exercised at any time up to the expiration date.
For example, Jim buys a 3 year American put option on ABC Stock with a strike price of $100.
This gives Jim the right to sell a share of ABC Stock for $100 at any time during the next 3 years.
156

American Versus European Options:
157
Since Jim may choose to exercise his American option prior to its expiration, an American option
may be worth more than a European option. Since Jim may choose to never exercise early and
instead only exercise his American option on its expiration date, an American option is worth at
least as much as the similar European option.
The put-call parity relationships discussed previously for the European options do not hold for an
American option.
158

Maximum and Minimum Prices:
As discussed previously, S
0
" C
Eur
" (PV
0,T
[F
0,T
] - PV
0,T
[K])
+
" 0.
In fact, since a European call can only be exercised at expiration: PV
0,T
[F
0,T
] " C
Eur
.
Using the same reasoning as used previously with respect to a European option, an American

call
option can not be worth more than the current stock price. Therefore, since C
Amer
" C
Eur
we have:
S
0
" C
Amer
( S
0
, K, T) " C
Eur
( S
0
, K, T) " (PV
0 , T
[F
0 , T
] - PV
0 , T
[K])
+
.
159

In fact, since an American call can be exercised right away: C
Amer
" (S
0
- K)
+
.
160

156
Recall that for a European style option, Jim would get the option to sell a share of stock on only a single day.
157
See Section 9.3 of Derivatives Markets by McDonald.
158
Appendix 9.A of McDonald, not on the syllabus, discusses parity bounds for American options.
C
Amer
(S
0
, K, T) + K - PV
0,T
[F
0,T
] " P
Amer
(S
0
, K, T) " C
Amer
(S
0
, K, T) + PV
0,T
[K] - S.
P
Amer
(S
0
, K, T) + S - PV
0,T
[K] " C
Amer
(S
0
, K, T) " P
Amer
(S
0
, K, T) + PV
0,T
[F
0,T
] - K.
Note that for European Options, C
Eur
(S
0
, K, T) + K - PV
0,T
[F
0,T
] " C
Eur
(S
0
, K, T) + PV
0,T
[K] - PV
0,T
[F
0,T
]
= P
Eur
(S
0
, K, T) " C
Eur
(S
0
, K, T) + PV
0,T
[K] - S.
159
See Equation 9.9 in McDonald.
160
Not discussed in Derivatives Markets by McDonald. If S
0
> K, in other words if the call is in the money, then one
could exercise the American call immediately, getting a payoff of S
0
- K.
If the dividend rate is small, then PV
0,T
[F
0,T
] - PV
0,T
[K] > S
0
- K.
If the dividend rate were large, such as when dealing with options on currency, the reverse could be true.
2014-MFE, Financial Economics 10 American Options HCM 11/13/13, Page 191
As discussed previously, K " P
Eur
" (PV
0,T
[K] - PV
0,T
[F
0,T
])
+
" 0.
In fact, since a European put can only be exercised at expiration: PV
0,T
[K] " P
Eur
.
161

Using the same reasoning as used previously with respect to a European option, an American

put
option can not be worth more than the strike price. Therefore, since P
Amer
" P
Eur
we have:
K " P
Amer
( S
0
, K, T) " P
Eur
( S
0
, K, T) " (PV
0 , T
[K] - PV
0 , T
[F
0 , T
])
+
.
162

In fact, since an American put can be exercised right away: P
Amer
" (K - S
0
)
+
.
163
Time Until Expiration:
Owen buys an American option with one year to expiration.
Tom buys an otherwise similar American option with two years to expiration.
Tom decides to exercise his option whenever Owen exercises his option within the first year, and
Tom throws his option away if he still holds it at the end of one year and it is not in the money.
164
If Tom mirrors Owen actions, then Tom!s option is worth the same as Owen!s option.
In general, an American option is worth at least as much as a similar option with less
time to expiration.
If instead Tom employs a better strategy, then his option would be worth more than Owen!s.
Tom!s option provides more opportunities for action than Owen!s option with a shorter time until
expiration. This is not true for a European option.
Olivia buys a one year European option and Tracy buys an otherwise similar two year European
option. Olivia can only exercise her option at time = 1, while Tracy can only exercise her option at
time = 2. While Tracy!s option is usually worth more, this is not always the case.
161
See MFE Sample Exam Q. 26.
162
See Equation 9.10 in McDonald.
163
Not discussed in Derivatives Markets by McDonald. See MFE Sample Exam Q. 26.
If S
0
< K, in other words if the put is in the money, then one could exercise the American put immediately, getting a
payoff of K - S
0
.
164
Mirroring Owen is not an optimal strategy for Tom.
2014-MFE, Financial Economics 10 American Options HCM 11/13/13, Page 192
Exercise: Olivia and Tracy have purchased puts with strike prices of $100.
At time = 1 the stock is worth $0.01. Whose option is worth more?
[Solution: Olivia exercises her option at time = 1 and makes $99.99.
The present value is $99.99e
-r
.
The most Tracy can make by exercising her option at time = 2 is $100.
The present value is $100e
-2r
, less than $99.99e
-r
for any reasonable value of r.
Comment: We have shown that it is possible that a European put could be worth less than a similar
option with less time until expiration.]
Exercise: Olivia and Tracy have purchased calls with strike prices of $100.
At time = 1 the stock is worth $125. At time equal 1.1 the stock will pay a dividend of $120.
Whose option is worth more?
[Solution: Olivia exercises her option at time = 1 and makes $25.
After paying the very large dividend, the price of the stock will decline to something in the range of
$5. It is extremely unlikely that the price of the stock by time = 2 will exceed 100. At time = 2, the
only time Tracy can exercise her option, it is very unlikely that her call option is in the money.
Tracy!s option will almost always turn out to be worthless.
Comment: We have shown that it is possible when dividends are paid that a European call could
be worth less than a similar option with less time until expiration.]
For European calls on stocks that do not pay dividends, the premium is the same as that of the
corresponding American call. Therefore, for a European call on a given non-dividend paying
stock, the call is worth at least as much as a similar call with less time to expiration.
However, the same relationship does not necessarily hold for either European calls on
stocks that pay dividends, or for European puts.
165
Early Exercise:
It turns out that for an American option, depending on the current price of the stock, the strike price,
and the time until expiration, etc., sometimes it is worthwhile to exercise the option prior to the
expiration date, and sometimes it is not. Here we are talking about the present value of each
strategy, rather than a 20-20 hindsight view of what actually turned out to happen to the stock price
after one had made a decision.
As will be shown subsequently, if the stock pays no dividends, then it is never worthwhile
to exercise an American call option early.
166



165
See page 280 of Derivatives Markets by McDonald.
166
If the stock does pay dividends, then we are only concerned about any dividends paid between the time the
option is bought and the time it expires.
2014-MFE, Financial Economics 10 American Options HCM 11/13/13, Page 193
Thus for a stock that pays no dividends, an American call option is
worth the same as a European call option.
As discussed previously, an American option is worth at least as much as a similar American option
with less time to expiration.
Therefore, for a stock that pays no dividends, a European option is worth at least as much as a
similar option with less time to expiration.
Proof that American Call Equals European Call When There Are No Dividends:
Let us assume that at time t we own a K-strike American Call on a non-dividend paying stock.
The call expires at time T > t.
As has been discussed previously: C
Amer
(t) " C
Eur
(t) " (PV
t,T
[F
t,T
] - PV
t,T
[K])
+
.
However, since there are no dividends: PV
t,T
[F
t,T
] = S
t
.
Thus, (PV
t,T
[F
t,T
] - PV
t,T
[K])
+
= (S
t
- K exp[-r(T-t)] )
+
> (S
t
- K)
+
.
However, (S
t
- K)
+
is the exercise value of the call at time t.
Thus we have shown that: C
Amer
(t) > the exercise value of the call at time t.
In other words, in the absence of dividends, it is never optional to exercise an American Call early.
2014-MFE, Financial Economics 10 American Options HCM 11/13/13, Page 194
Difficulty of Deciding Whether to Exercise Early:
There are various simple strategies one could use to decide when to exercise an American Option.
For example, one could choose to always exercise the option right away. Jim buys a 3 year
American put option on ABC Stock with a strike price of $100. If the current market price of ABC
Stock is $120, this is a stupid strategy, since Jim would be better off selling the stock for the $120
market price rather than the $100 strike price.
Assume that in one year, it turns out that ABC Stock has a market price of $90. If Jim buys a share
of ABC Stock for $90 and then exercises his option to sell at $100, he makes $10. However, this
may or may not be the best decision. If over the next two years, the price of ABC Stock never falls
below $90, then Jim should have exercised his option early. If over the next two years, the price of
ABC Stock falls to $80, then Jim could have made $20 if he had waited to exercise his option.
Out of the many possible paths, let us look at four, all of which have a price of $90 at time 1:
167

Time Price Path 1 Price Path 2 Price Path 3 Price Path 4
1 $90 $90 $90 $90
2 $95 $90 $85 $85
3 $100 $90 $80 $100
In path #1, Jim should exercise his option at time 1. If he instead waits until time 2 to exercise his
option, then he makes only $5 rather than $10, and also loses the time value of money. If he does
not exercise his option until expiration, he is out of luck and makes nothing!
In path #2, Jim should exercise his option at time 1. If he waits to exercise his option, he still makes
$10, but loses the time value of money.
In path #3, Jim should wait to exercise his option. If he waits until time 2 he makes $15 instead of
$10. If he waits until time 3 he makes $20!
In path #4, Jim should also wait to exercise his option. If he waits until time 2 he makes $15 instead
of $10. However, if he then waits until time 3 he makes nothing!
Assuming Jim can not see the future, it can be a tough decision whether or not to exercise his
American option early. Ideally, Jim should exercise his option whenever the value of doing so is
greater than the present value of not doing so.
Thus a key tool would be some way for Jim to estimate the present value of his put option based
on the current stock price, time to expiration, strike price, etc. In subsequent sections, we will discuss
ways to tackle this problem.
167
Jim can exercise his option on any day through expiration. I have only looked at three points in time for simplicity.
2014-MFE, Financial Economics 10 American Options HCM 11/13/13, Page 195
Early Exercise of a Call:
168

The continuation value of an option is the value of the option at a given point in time and
at a given stock price, if we do not exercise the option immediately.
In other words, the continuation value is the actuarial present vale of waiting.
One should exercise early if the continuation value of the option is less than the value of exercising
the option. In other words, one should exercise a call at time t < T if C
Amer
(S
t
, K, T- t) < S
t
- K.
If one exercises an American call before expiration, one will own the stock, and benefit from
dividends. If there is a particularly large stock dividend, this would make it worthwhile to exercise the
call just before the payment of the dividend.
169

By exercising an American call before expiration, one loses interest one can earn on the strike price.
Also by exercising early, one loses the implicit protection against the stock price moving below the
strike price. If one exercises the call early and buys the stock, we take on the risk that the stock price
might go down a lot. We can understand this by examining the value of a similar European option.
Put-call parity states that for a stock that pays dividends continuously, at time t:
C
Amer
(S
t
, K, T - t) " C
Eur
(S
t
, K, T - t) = P
Eur
(S
t
, K, T - t) + S
t
e
-#(T-t)
- K e
-r(T-t)
= (S
t
- K) + P
Eur
(S
t
, K, T - t) + K(1 - e
-r(T-t)
) - S
t
{1 -e
-#(T-t)
}
= (Exercise Value of the Call) + (Insurance against S
T
< K) + (Time value of money on K)
- (Present Value of future Dividends).
More generally, assume that you own a stock and wish to lock in your capital gains. If you wish to
guarantee that you can sell the stock for K at time T in the future, then you could buy a K-strike
European put that expires at time T. This put acts as insurance against a decrease in the stock price.
For a stock that does not pay dividends, put-call parity states that at time t:
C
Amer
(S
t
, K, T - t) = C
Eur
(S
t
, K, T - t) = (S
t
- K) + P
Eur
(S
t
, K, T - t) + K{1 - e
-r(T-t)
}
= (Exercise Value of the call) + (Insurance against S
T
< K) + (Time value of money on K).
170
P
Eur
(S
t
, K, T - t) " 0 and K(1 - e
-r(T-t)
) " 0.
Therefore, for a stock that does not pay dividends: C
Amer
(S
t
, K, T - t) " S
t
- K.
Therefore, there is never a reason to exercise an American call early in the absence of dividends.
168
See pages 294 to 296 and Section 11.1 of Derivatives Markets by McDonald.
169
A dividend is paid to the person who owns the stock on a specific day. If for example that day is March 30, and if
one buys the stock on March 31, then the price is ex-dividend. If Dave buys the stock from Judy on March 31, Judy
will receive the dividend, even if the dividend is actually paid in early April.
170
See equation 9.11 in Derivatives Markets by McDonald.
2014-MFE, Financial Economics 10 American Options HCM 11/13/13, Page 196
For American calls there are two reasons to wait and one reason to exercise early:
1. The time value of the strike price is lost if one exercises early.
2. The implicit insurance protection against the stock price moving below the strike price
is lost if one exercises early.
3. The value of dividends is gained if one exercises early.
If we exercise early, then we spend the strike price. If instead we were to wait to exercise, then we
do not spend the strike price and we can earn interest on that money.
If we exercise early, then we own the stock, and risk the stock price going down. If instead we were
to wait to exercise, then we retain this implicit insurance protection against the stock price moving
below the strike price.
171

172
Put another way, if we wait to exercise the call, we avoid the possibility
of capital losses on the stock.
If we exercise the call early, then we own the stock and get the dividends.
C
Amer
(S
t
, K, T - t) " C
Eur
(S
t
, K, T - t) = P
Eur
(S
t
, K, T - t) + PV
t,T
[F
t,T
] - PV
t,T
[K]
" PV
t,T
[F
t,T
] - PV
t,T
[K]. Therefore, C
Amer
" PV[F
t,T
] - PV[K], as discussed previously.
The payoff for exercising early an American call at time t is:
S
t
- K = PV[F
t,T
] + PV[Div] - PV[K] - (K - PV[K])
= PV[F
t,T
] - PV[K] + PV[Div] - (K - PV[K]) # C
Amer
(S
t
, K, T - t) + PV[Div] - (K - PV[K]).
Therefore, if PV
t,T
[Div] - (K - PV
t,T
[K]) < 0, then S
t
- K # C
Amer
,
and it would not be worthwhile to exercise early.
If K - PV
t , T
[K] > PV
t , T
[Div], then one should not exercise an American call early
at time t.
173

174
In other words, if K{1 - exp[-r(T- t)]} is greater than the present value of the future
dividends that will be paid until the call expires, then it is better to wait rather than exercise early.
For example, if K = $100, r = 5%, and there are 6 months to expiration of an American call, then if
PV[Div] < 100(1 - e
-0.025
) = $2.47, it would not be worthwhile to exercise at this time. For example,
if there is a single $2 dividend tomorrow it would not be worthwhile to exercise at this time.
In this case, the benefit of getting interest on the strike price exceeds the benefit from exercising
early and thus receiving the stock dividends; therefore, it is optimal to wait.
171
Personally, I do not find McDonald!s use of the term insurance implicit in the call to be helpful.
172
This implicit protection can be thought of as the implicit put from put-call parity. The larger /, the larger the value of
this implicit put, and thus the less likely it will be optimal to exercise early, all else being equal.
173
See Equation 9.14 in Derivatives Markets by McDonald. K - PV
t,T
[K] = K(1 - exp[-r(T- t)])
Specifically, if there are no dividends, then one should not exercise an American call early.
174
If K - PV[K] < PV[Div], then it may or may not make sense to exercise an American call early.
Another necessary condition to exercise early an American call is that S
t
> K.
2014-MFE, Financial Economics 10 American Options HCM 11/13/13, Page 197
If instead, PV[Div] > $2.47, then it might be worthwhile to exercise at this time. For example, if there
is a single $3 dividend two months from now, then PV[Div] = 3e
-0.05/6
= 2.98 > 2.47, and it might
be worthwhile to exercise at this time.
In this case, the benefit from exercising early and thus receiving the stock dividends exceeds the
benefit of getting interest on the strike price. However, if we exercise early, besides losing the
interest on the strike, we would also lose the implicit insurance protection. Therefore, it may or may
not be optimal to exercise early. In general, it may be optimal to exercise early an American Call if
the dividend rate is large compared to the risk free rate.
In fact, C
Amer
(S
t
, K, T - t) " P
Eur
(S
t
, K, T - t) + PV
t,T
[F
t,T
] - PV
t,T
[K].
Therefore, the payoff for exercising early an American call at time t is:
S
t
- K = PV[F
t,T
] + PV[Div] - PV[K] - (K - PV[K]) = PV[F
t,T
] - PV[K] + PV[Div] - (K - PV[K])
# C
Amer
(S
t
, K, T - t) - P
Eur
(S
t
, K, T - t) + PV[Div] - (K - PV[K]).
Therefore, if PV
t,T
[Div] - (K - PV
t,T
[K]) - P
Eur
(S
t
, K, T - t) < 0, then S
t
- K # C
Amer
,
and it would not be worthwhile to exercise early.
If P
Eur
(S
t
, K, T - t) + K - PV
t,T
[K] > PV
t,T
[Div], then one should not exercise an American call
early at time t.
175
If were to exercise an American Call early, then we would own the stock and
collect its dividends, but we would lose the interest on the strike price and also lose the implicit
insurance protection represented by the put premium.
Let us assume there is a single $3 dividend two months from now.
Jubal and Anderson each have identical 6 month $100 strike American calls.
Jubal exercises his call now, pays $100 and owns the stock.
Anderson instead invests $100 e
-0.05/6
= $99.17 at the risk free rate for two months.
Two months from now, just before the dividend is paid, Anderson has $100 and uses it exercises
his call and own the stock. At that point in time both Jubal and Anderson own the stock and get the
dividend payment. However, Jubal invested $100 today, while Anderson only invested $99.17.
Thus Anderson!s strategy was better.
176

In general, if it is optimal to exercise an American call early, and dividends are paid at
discrete times, then it is best to exercise right before the payment of a (large) dividend,
in other words at the last moment before the ex-dividend date.
177

175
Not discussed in Derivatives Markets by McDonald.
176
While Anderson does better than Jubal, it may still not be optimal to exercise the call early.
177
Note that this statement is conditional on it being optimal to exercise the call early.
2014-MFE, Financial Economics 10 American Options HCM 11/13/13, Page 198
Early Exercise of a Put:
178
The situation for early exercise of puts is somewhat reversed from calls.
For American puts there are two reasons to wait and one reason to exercise early:
1. The time value of the strike price is gained if one exercises early.
2. The implicit insurance protection against the stock price moving above the strike price
is lost if one exercises early.
3. The value of dividends is lost if one exercises early.
If we exercise early, then we receive the strike price and we can earn interest on that money.
If we exercise the put early and use it to sell the stock, then we do not own the stock, and fail to gain
if the stock price goes up. If instead we were to wait to exercise, then we retain this implicit insurance
protection against the stock price moving above the strike price.
179
Saying it another way, if we wait
to exercise the put, we retain the possibility of additional capital gains on the stock.
If we exercise the put early, then we do not own the stock and do not get the dividends.
It can make sense to exercise an American put early, whether or not the stock pays dividends.
By Put-Call Parity: P
Eur
= Ke
-rT
- S
T
+

PV[Div] + C
Eur
.
P
Amer
" P
Eur
. % P
Amer
" K - S
T
+ PV[Div] + C
Eur
- K (1 - e
-rT
) =
(Exercise Value of the Put) + (Present Value of Future Dividends)
+ (Insurance against S
T
> K) - (Time Value of Money on Strike).
Moneyness:
180

If at a point in time it is worthwhile to exercise an American call with a strike price of K, it also makes
sense to exercise an otherwise similar call with a strike price lower than K. For example, if one should
exercise an option to buy at 100, then one should also exercise a similar option to buy at 80.
If at a point in time it is worthwhile to exercise an American put with a strike price of K, it also makes
sense to exercise an otherwise similar put with a strike price higher than K. For example, if one
should exercise an option to sell at 60, then one should also exercise a similar option to sell at 70.
If it makes sense to exercise an option that is in the money, then it also makes sense to exercise an
option that is more in the money.
178
See pages 278 to 279 and Section 11.1 of Derivatives Markets by McDonald.
179
This implicit protection can be thought of as the implicit call from put-call parity. The larger /, the larger the value of
this implicit call, and thus the less likely it will be optimal to exercise early, all else being equal.
180
See page 286 of Derivatives Markets by McDonald.
2014-MFE, Financial Economics 10 American Options HCM 11/13/13, Page 199
Early Exercise Boundaries:
For an American option, using for example Binomial Trees to be discussed subsequently, one can
determine the stock price at which one should exercise early.
In the case of a call, one would exercise early if the stock price were greater than a given value. For
example, at time 1.5 one might exercise a call early if the stock price were more than 200. In the
case of a put, one would exercise early if the stock price were less than a given value. For example,
at time 3 one might exercise a put early if the stock price were less than 55.
For a given option, as a function time, the set of stock prices at which the continuation
value is equal to the value of immediate exercise is called the early exercise boundary.
The following graph shows early exercise boundaries for a 5-year 100 strike American call, for the
volatility of the future price of the stock, /, either 30% or 50%:
181 182
1 2 3 4 5
Time
150
200
250
300
350
Stock Price
sigma=0.5
sigma=0.3
We note two features. First, as / increases, the exercise boundary is higher. For larger /, the implicit
insurance protection is larger. Therefore, the larger /, the larger the continuation value. Therefore, a
higher stock price, and thus a higher value of exercising the call immediately, is required in order to
make it worthwhile to exercise the option early.
Second, as the option gets closer to expiration, the exercise boundary approaches the strike price.
As the option approaches expiration, we lose less insurance protection by exercising early.
181
Based on Figure 11.1 of Derivatives Markets by McDonald, which was computed using Binomial Trees, to be
discussed subsequently. The volatility will be discussed in subsequent sections.
182
While one needs to know how to work with Binomial Trees, the huge amount of work required to determine the
exercise boundary is well beyond what you could be required to do on the exam, even if one were supplied with a
computer!
2014-MFE, Financial Economics 10 American Options HCM 11/13/13, Page 200
The following graph shows early exercise boundaries for a 5-year 100 strike American put, for /
either 30% or 50%:
183
1 2 3 4 5
Time
30
40
50
60
70
80
90
100
Stock Price
sigma=0.5
sigma=0.3
We note two features. First, as / increases, the exercise boundary is lower. For larger /, the implicit
insurance protection is larger. Therefore, the larger /, the larger the continuation value. Therefore, a
lower stock price, and thus a higher value of exercising the put immediately, is required in order to
make it worthwhile to exercise the option early.
Second, as the option gets closer to expiration, the exercise boundary approaches the strike price.
As the option approaches expiration, we lose less insurance protection by exercising early.
For both puts and calls, the early-exercise criterion becomes less stringent as the option
has less time until expiration.
For both puts and calls, the early-exercise criterion becomes more stringent as the
volatility of the stock increases.
Put-Call Parity:
184
The put-call parity relationships discussed previously for the European options do not hold for an
American option. However, it is the case that for American Options:
S
0
- PV[Div] - K # C
Amer
- P
Amer
# S
0
- Ke
-rT
.
183
Based on Figure 11.2 of Derivatives Markets by McDonald, which was computed using Binomial Trees, to be
discussed subsequently.
184
See Appendix 9.A of McDonald, not on the syllabus, discusses parity bounds for American options.
2014-MFE, Financial Economics 10 American Options HCM 11/13/13, Page 201
Properties of Premiums of American Options:
185
The same three properties hold for American Options as for European Options.
Strike Prices: K
1
< K
2
< K
3
. C is the premium for a Call, while P is the premium for a put.
Condition Arbitrage if the Condition is Violated
C(K
1
) " C(K
2
). Buy the K
1
Call and Sell the K
2
Call (Call Bull Spread)
C(K
1
) - C(K
2
) # K
2
- K
1
. Sell the K
1
Call and Buy the K
2
Call (Call Bear Spread)

C(K
1
) - C(K
2
)
K
2
- K
1
Buy + = (K
3
- K
2
)/(K
3
- K
1
) of K
1
,

Sell 1 of K
2
,

Buy 1 - + of K
3
.
186
"

C(K
2
) - C(K
3
)
K
3
- K
2
.
(Asymmetric butterfly spread)
P(K
2
) " P(K
1
). Sell the K
1
Put and Buy the K
2
Put (Put Bear Spread)
P(K
2
) - P(K
1
) # K
2
- K
1
. Buy the K
1
Put and Sell the K
2
Put (Put Bull Spread)

P(K
2
) - P(K
1
)
K
2
- K
1
Buy + = (K
3
- K
2
)/(K
3
- K
1
) of K
1
,

Sell 1 of K
2
,

Buy 1 - + of K
3
.
187

#

P(K
3
) - P(K
2
)
K
3
- K
2
.
(Asymmetric butterfly spread)
185
See page s281-285 of Derivatives Markets by McDonald.
186
There are other sets of amounts of each call that would also demonstrate arbitrage.
187
There are other sets of amounts of each put that would also demonstrate arbitrage.
2014-MFE, Financial Economics 10 American Options HCM 11/13/13, Page 202
Proving the Three Properties of Call Premiums
:
188

Let us assume we have three otherwise similar calls with different strikes.
Strike Prices: K
1
< K
2
< K
3
. C is the premium for a call.
If C(K
2
) > C(K
1
), then we can demonstrate arbitrage by buying a call bull spread:
buy the K
1
call and sell the K
2
call.
189

We lend: C(K
2
) - C(K
1
) > 0.
If we are dealing with European options, then if at expiration S
T
# K
1
, both options expire worthless.
So we have: {C(K
2
) - C(K
1
)} e
rT
> 0.
If at expiration K
2
" S
T
> K
1
, then the option we sold expires worthless.
So we have: (S
T
- K
1
) + {C(K
2
) - C(K
1
)} e
rT
> 0.
If at expiration S
T
> K
2
, then both options expire in the money.
So we have: (S
T
- K
1
) - (S
T
- K
2
) + {C(K
2
) - C(K
1
)} e
rT
= (K
2
- K
1
) + {C(K
2
) - C(K
1
)} e
rT
> 0.
Thus we have demonstrated arbitrage for European calls.
If instead we have American calls, we can deal with the same three cases.
If the call we sold is not exercised early, then the same logic applies as for European calls.
190

If instead the option we sold is exercised early at time t, then it must be that S
t
> K
2
.
We can then also exercise at time t the call that we bought.
191

We then have: (S
t
- K
1
) - (S
t
- K
2
) + {C(K
2
) - C(K
1
)} e
rt
= (K
2
- K
1
) + {C(K
2
) - C(K
1
)} e
rt
> 0.
Thus we have demonstrated arbitrage for American calls.
188
See Appendix 9.B of McDonald, not on the syllabus.
189
The low-strike call is underpriced so we buy it; the high-strike call is overpriced so we sell it.
190
We can wait to expiration to exercise the call we bought and be guaranteed ending with a positive amount.
191
If at time t we can sell the call we bought for more than its exercise value, so much the better for us.
2014-MFE, Financial Economics 10 American Options HCM 11/13/13, Page 203
If C(K
1
) - C(K
2
) > K
2
- K
1
, then we can demonstrate arbitrage by buying a call bear spread:
sell the K
1
call and buy the K
2
call.
192

We lend: C(K
1
) - C(K
2
) > K
2
- K
1
> 0.
If we are dealing with European options, then if at expiration S
T
# K
1
, both options expire worthless.
So we have: {C(K
1
) - C(K
2
)} e
rT
> 0.
If at expiration K
2
> S
T
" K
1
, then the option we bought expires worthless.
So we have: {C(K
1
) - C(K
2
)} e
rT
- (S
T
- K
1
) > {K
2
- K
1
} e
rT
- (S
T
- K
1
) >
{K
2
- K
1
} e
rT
- (K
2
- K
1
) > 0.
If at expiration K
2
< S
T
, then both options expire in the money.
So we have: {C(K
1
) - C(K
2
)} e
rT
+ (S
T
- K
2
) - (S
T
- K
1
) = {C(K
1
) - C(K
2
)} e
rT
- (K
2
- K
1
) >
{K
2
- K
1
} e
rT
- (K
2
- K
1
) > 0.
Thus we have demonstrated arbitrage for European calls.
If instead we have American calls, we can deal with the same three cases.
If the call we sold is not exercised early, then the same logic applies as for European calls.
193

If instead the option we sold is exercised early at time t, then it must be that S
t
> K
1
.
If K
2
> S
t
" K
1
, then the option we bought expires worthless.
So we have: {C(K
1
) - C(K
2
)} e
rt
- (S
t
- K
1
) > {K
2
- K
1
} e
rt
- (S
t
- K
1
) >
{K
2
- K
1
} e
rt
- (K
2
- K
1
) > 0.
If S
t
> K
2
, then we can also exercise at time t the call that we bought.
194

We then have: {C(K
1
) - C(K
2
)} e
rt
+ (S
t
- K
2
) - (S
t
- K
1
) = {C(K
1
) - C(K
2
)} e
rt
- (K
2
- K
1
) >
{K
2
- K
1
} e
rt
- (K
2
- K
1
) > 0.
Thus we have demonstrated arbitrage for American calls.
192
This the reverse of what we did to demonstrate arbitrage if the previous property was violated.
193
We can wait to expiration to exercise the call we bought and be guaranteed ending with a positive amount.
194
If at time t we can sell the call we bought for more than its exercise value, so much the better for us.
2014-MFE, Financial Economics 10 American Options HCM 11/13/13, Page 204
If

C(K
1
) - C(K
2
)
K
2
- K
1
<

C(K
2
) - C(K
3
)
K
3
- K
2
3 (K
2
- K
1
) {C(K
2
) - C(K
3
)} - (K
3
- K
2
) {C(K
1
) - C(K
2
)} > 0
3 (K
3
- K
1
) C(K
2
) - (K
3
- K
2
) C(K
1
) - (K
1
- K
2
) C(K
3
) > 0,
then we can demonstrate arbitrage by buying an asymmetric butterfly call spread:
Buy + = (K
3
- K
2
)/(K
3
- K
1
) of low strike,

sell 1 of medium strike, and,

buy 1 - + of the high strike call.
We lend:
C(K
2
) - + C(K
1
) - (1 - +) C(K
3
) =

(K
3
- K
1
)C(K
2
) - (K
3
- K
2
)C(K
1
) - (K
1
- K
2
)C(K
3
)
K
3
- K
1
> 0.
If we are dealing with European options, then if at expiration S
T
# K
1
, all options expire worthless.
So we have: {C(K
2
) - + C(K
1
) - (1 - +) C(K
3
)} e
rT
> 0.
If at expiration K
2
" S
T
> K
1
, then the low strike option we bought expires in the money.
So we have: {C(K
2
) - + C(K
1
) - (1 - +) C(K
3
)} e
rT
+ +(S
T
- K
1
) > 0.
If at expiration K
3
" S
T
> K
2
, then the low and medium strike options expire in the money.
So we have: {C(K
2
) - + C(K
1
) - (1 - +) C(K
3
)} e
rT
+ +(S
T
- K
1
) - (S
T
- K
2
) >
C(K
2
) - + C(K
1
) - (1 - +) C(K
3
) - (1 - +)S
T
- +K
1
+ K
2
>
C(K
2
) - + C(K
1
) - (1 - +) C(K
3
) - (1 - +)K
3
- +K
1
+ K
2
=
C(K
2
) - + C(K
1
) - (1 - +) C(K
3
) - K
3


K
2
- K
1
K
3
- K
1
- K
1


K
3
- K
2
K
3
- K
1
+ K
2
=
C(K
2
) - + C(K
1
) - (1 - +) C(K
3
) > 0.
195

If at expiration K
3
< S
T
, then all three options expire in the money.
So we have: {C(K
2
) - + C(K
1
) - (1 - +) C(K
3
)} e
rT
+ +(S
T
- K
1
) - (S
T
- K
2
) + (1 - +)(S
T
- K
3
) >
C(K
2
) - + C(K
1
) - (1 - +) C(K
3
) - (1 - +)K
3
- +K
1
+ K
2
=
C(K
2
) - + C(K
1
) - (1 - +) C(K
3
) - K
3


K
2
- K
1
K
3
- K
1
- K
1


K
3
- K
2
K
3
- K
1
+ K
2
=
C(K
2
) - + C(K
1
) - (1 - +) C(K
3
) > 0.
Thus we have demonstrated arbitrage for European calls.
195
Note that from the definition of + = (K
3
- K
2
)/(K
3
- K
1
), it follows that: K
2
- +K
1
- (1 - +)K
3
= 0.
2014-MFE, Financial Economics 10 American Options HCM 11/13/13, Page 205
If instead we have American calls, we can deal with the same four cases.
If the call we sold is not exercised early, then the same logic applies as for European calls.
196

If instead the option we sold is exercised early at time t, then it must be that S
t
> K
2
.
If K
3
> S
t
" K
2
, then the high strike option we bought has an exercise value of zero and we can
exercise the low strike call we bought.
197
So we have: {C(K
2
) - + C(K
1
) - (1 - +) C(K
3
)} e
rt
+ +(S
t
- K
1
) - (S
t
- K
2
) >
C(K
2
) - + C(K
1
) - (1 - +) C(K
3
) - (1 - +)S
t
- +K
1
+ K
2
>
C(K
2
) - + C(K
1
) - (1 - +) C(K
3
) - (1 - +)K
3
- +K
1
+ K
2
=
C(K
2
) - + C(K
1
) - (1 - +) C(K
3
) - K
3


K
2
- K
1
K
3
- K
1
- K
1


K
3
- K
2
K
3
- K
1
+ K
2
=
C(K
2
) - + C(K
1
) - (1 - +) C(K
3
) > 0.
If S
t
> K
3
, then we can also exercise at time t both calls that we bought.
198

So we have: {C(K
2
) - + C(K
1
) - (1 - +) C(K
3
)} e
rt
+ +(S
t
- K
1
) - (S
t
- K
2
) + (1 - +)(S
t
- K
3
) >
C(K
2
) - + C(K
1
) - (1 - +) C(K
3
) - (1 - +)K
3
- +K
1
+ K
2
=
C(K
2
) - + C(K
1
) - (1 - +) C(K
3
) - K
3


K
2
- K
1
K
3
- K
1
- K
1


K
3
- K
2
K
3
- K
1
+ K
2
=
C(K
2
) - + C(K
1
) - (1 - +) C(K
3
) > 0.
Thus we have demonstrated arbitrage for American calls.
196
We can wait to expiration to exercise the calls we bought and be guaranteed ending with a positive amount.
197
If at time t we can sell the calls we bought for more than their exercise value, so much the better for us.
198
If at time t we can sell the calls we bought for more than their exercise value, so much the better for us.
2014-MFE, Financial Economics 10 American Options HCM 11/13/13, Page 206
Proving the Three Properties of Put Premiums:
199

Let us assume we have three otherwise similar puts with different strikes.
Strike Prices: K
1
< K
2
< K
3
. P is the premium for a put.
If P(K
2
) < P(K
1
), then we can demonstrate arbitrage by buying a put bear spread:
sell the K
1
put and buy the K
2
put.
200

We lend: P(K
1
) - P(K
2
) > 0.
If we are dealing with European options, then if at expiration S
T
" K
2
, both options expire worthless.
So we have: {P(K
1
) - P(K
2
)} e
rT
> 0.
If at expiration K
2
> S
T
" K
1
, then the option we sold expires worthless.
So we have: (K
2
- S
T
) + {P(K
1
) - P(K
2
)} e
rT
> 0.
If at expiration K
1
> S
T
, then both options expire in the money.
So we have: (K
2
- S
T
) - (K
1
- S
T
) + {P(K
1
) - P(K
2
)} e
rT
= (K
2
- K
1
) + {P(K
1
) - P(K
2
)} e
rT
> 0.
Thus we have demonstrated arbitrage for European puts.
If instead we have American puts, we can deal with the same three cases.
If the put we sold is not exercised early, then the same logic applies as for European puts.
201

If instead the option we sold is exercised early at time t, then it must be that S
t
< K
1
.
We can then also exercise at time t the put that we bought.
202

We then have: (K
2
- S
t
) - (K
1
- S
t
) + {P(K
1
) - P(K
2
)} e
rt
= (K
2
- K
1
) + {P(K
1
) - P(K
2
)} e
rt
> 0.
Thus we have demonstrated arbitrage for American puts.
199
See Appendix 9.B of McDonald, not on the syllabus.
200
The high-strike put is underpriced so we buy it; the low-strike put is overpriced so we sell it.
201
We can wait to expiration to exercise the put we bought and be guaranteed ending with a positive amount.
202
If at time t we can sell the put we bought for more than its exercise value, so much the better for us.
2014-MFE, Financial Economics 10 American Options HCM 11/13/13, Page 207
If P(K
2
) - P(K
1
) > K
2
- K
1
, then we can demonstrate arbitrage by buying a put bull spread:
buy the K
1
put and sell the K
2
put.
203

We lend: P(K
2
) - P(K
1
) > K
2
- K
1
> 0.
If we are dealing with European options, then if at expiration S
T
" K
2
, both options expire worthless.
So we have: {P(K
2
) - P(K
1
)} e
rT
> 0.
If at expiration K
2
> S
T
" K
1
, then the option we bought expires worthless.
So we have: {P(K
2
) - P(K
1
)} e
rT
- (K
2
- S
T
) > {K
2
- K
1
} e
rT
- (K
2
- S
T
) >
{K
2
- K
1
} e
rT
- (K
2
- K
1
) > 0.
If at expiration K
1
> S
T
, then both options expire in the money.
So we have: {P(K
2
) - P(K
1
)} e
rT
+ (K
1
- S
T
) - (K
2
- S
T
) = {P(K
2
) - P(K
1
)} e
rT
- (K
2
- K
1
) >
{K
2
- K
1
} e
rT
- (K
2
- K
1
) > 0.
Thus we have demonstrated arbitrage for European puts.
If instead we have American puts, we can deal with the same three cases.
If the put we sold is not exercised early, then the same logic applies as for European puts.
204

If instead the option we sold is exercised early at time t, then it must be that S
t
< K
2
.
If K
2
> S
t
" K
1
, then the option we bought expires worthless.
So we have: {P(K
2
) - P(K
1
)} e
rt
- (K
2
- S
t
) > {K
2
- K
1
} e
rt
- (K
2
- S
t
) >
{K
2
- K
1
} e
rt
- (K
2
- K
1
) > 0.
If S
t
< K
1
, then we can also exercise at time t the put that we bought.
205

We then have: {P(K
2
) - P(K
1
)} e
rt
- (K
2
- S
t
) + (K
1
- S
t
) = {P(K
2
) - P(K
1
)} e
rt
- (K
2
- K
1
) >
{K
2
- K
1
} e
rt
- (K
2
- K
1
) > 0.
Thus we have demonstrated arbitrage for American puts.
203
This the reverse of what we did to demonstrate arbitrage if the previous property was violated.
204
We can wait to expiration to exercise the put we bought and be guaranteed ending with a positive amount.
205
If at time t we can sell the put we bought for more than its exercise value, so much the better for us.
2014-MFE, Financial Economics 10 American Options HCM 11/13/13, Page 208
If

P(K
2
) - P(K
1
)
K
2
- K
1
>

P(K
3
) - P(K
2
)
K
3
- K
2
3 (K
2
- K
1
) {P(K
2
) - P(K
3
)} - (K
3
- K
2
) {P(K
1
) - P(K
2
)} > 0
3 (K
3
- K
1
) P(K
2
) - (K
3
- K
2
) P(K
1
) - (K
1
- K
2
) P(K
3
) > 0,
then we can demonstrate arbitrage by buying an asymmetric butterfly put spread:
Buy + = (K
3
- K
2
)/(K
3
- K
1
) of low strike,

sell 1 of medium strike, and,

buy 1 - + of the high strike put.
We lend:
P(K
2)
- + P(K
1
) - (1 - +) P(K
3
) =


(K
3
- K
1
)P(K
2
) - (K
3
- K
2
)P(K
1
) - (K
1
- K
2
)P(K
3
)
K
3
- K
1
> 0.
If we are dealing with European options, then if at expiration S
T
" K
3
, all options expire worthless.
So we have: {P(K
2
) - + P(K
1
) - (1 - +) P(K
3
)} e
rT
> 0.
If at expiration K
3
> S
T
" K
2
, then the high strike option we bought expires in the money.
So we have: {P(K
2
) - + P(K
1
) - (1 - +) P(K
3
)} e
rT
+ (1 - +)(K
3
- S
T
) > 0.
If at expiration K
2
> S
T
" K
1
, then the high and medium strike options expire in the money.
So we have: {P(K
2
) - + P(K
1
) - (1 - +) P(K
3
)} e
rT
+ (1 - +)(K
3
- S
T
) - (K
2
- S
T
) >
P(K
2
) - + P(K
1
) - (1 - +) P(K
3
) + +S
T
+ (1 - +)K
3
- K
2
>
P(K
2
) - + P(K
1
) - (1 - +) P(K
3
) + +K
1
+ (1 - +)K
3
- K
2
=
P(K
2
) - + P(K
1
) - (1 - +) P(K
3
) + K
1


K
3
- K
2
K
3
- K
1
+ K
3


K
2
- K
1
K
3
- K
1
- K
2
=
P(K
2
) - + P(K
1
) - (1 - +) P(K
3
) > 0.
206

If at expiration K
1
> S
T
, then all three options expire in the money.
So we have: {P(K
2
) - + P(K
1
) - (1 - +) P(K
3
)} e
rT
+ +(K
1
- S
T
) + (1 - +)(K
3
- S
T
) - (K
2
- S
T
) >
P(K
2
) - + P(K
1
) - (1 - +) P(K
3
) + (1 - +)K
3
+ +K
1
- K
2
=
P(K
2
) - + P(K
1
) - (1 - +) P(K
3
) + K
3


K
2
- K
1
K
3
- K
1
+ K
1


K
3
- K
2
K
3
- K
1
- K
2
=
P(K
2
) - + P(K
1
) - (1 - +) P(K
3
) > 0.
Thus we have demonstrated arbitrage for European puts.
206
Note that from the definition of + = (K
3
- K
2
)/(K
3
- K
1
), it follows that: K
2
- +K
1
- (1 - +)K
3
= 0.
2014-MFE, Financial Economics 10 American Options HCM 11/13/13, Page 209
If instead we have American puts, we can deal with the same four cases.
If the put we sold is not exercised early, then the same logic applies as for European puts.
207

If instead the option we sold is exercised early at time t, then it must be that S
t
< K
2
.
If K
2
> S
t
" K
1
, then the low strike option we bought has an exercise value of zero and we can
exercise the high strike put we bought.
208
So we have: {P(K
2
) - + P(K
1
) - (1 - +) P(K
3
)} e
rt
+ (1 - +)(S
t
- K
3
) - (S
t
- K
2
) >
P(K
2
) - + P(K
1
) - (1 - +) P(K
3
) + +S
t
+ (1 - +)K
3
- K
2
>
P(K
2
) - + P(K
1
) - (1 - +) P(K
3
) + +K
1
+ (1 - +)K
3
- K
2
=
P(K
2
) - + P(K
1
) - (1 - +) P(K
3
) + K
1


K
3
- K
2
K
3
- K
1
+ K
3


K
2
- K
1
K
3
- K
1
- K
2
=
P(K
2
) - + P(K
1
) - (1 - +) P(K
3
) > 0.
If S
t
< K
1
, then we can also exercise at time t both puts that we bought.
209

So we have: {P(K
2
) - + P(K
1
) - (1 - +) P(K
3
)} e
rt
+ +(K
1
- S
t
) + (1 - +)(K
3
- S
t
) - (K
2
- S
t
) >
P(K
2
) - + P(K
1
) - (1 - +) P(K
3
) + (1 - +)K
3
+ +K
1
- K
2
=
P(K
2
) - + P(K
1
) - (1 - +) P(K
3
) + K
3


K
2
- K
1
K
3
- K
1
+ K
1


K
3
- K
2
K
3
- K
1
- K
2
=
P(K
2
) - + P(K
1
) - (1 - +) P(K
3
) > 0.
Thus we have demonstrated arbitrage for American puts.
Bermudan Options:
210

In the case of a Bermudan option, the buyer has the right to exercise only at a set of discretely
spaced times. This is intermediate between a European option, which allows exercise at a single
time, and an American option, which allows exercise at any time up to expiration. For example, a
Bermudan interest rate swap might allow exercise at six month intervals.
In a subsequent section, we will discuss pricing American options via Binomial Trees. If we used a
tree with nodes spaced 3 months apart, then we would really be pricing a Bermudan option where
one can only exercise at 3 month intervals. As the time between the nodes approaches zero, the
premium of the American option is a limit of those of the Bermudan options.
207
We can wait to expiration to exercise the puts we bought and be guaranteed ending with a positive amount.
208
If at time t we can sell the puts we bought for more than their exercise value, so much the better for us.
209
If at time t we can sell the puts we bought for more than their exercise value, so much the better for us.
210
Not on the syllabus.
2014-MFE, Financial Economics 10 American Options HCM 11/13/13, Page 210
Problems:
10.1 (2 points) r = 0%. Stock ABC pays dividends. You own an American call on ABC.
Is it ever optimal to exercise this option before expiration? Briefly explain why.
10.2 (1 point) You have an American put option to exchange one share of Stock A for one share of
Stock B. Neither stock pays dividends. In what circumstances might you exercise this option early?
10.3 (1 point) Which of the following statements about American style options is not true?
A. The option may be exercised at any time during its life.
B. If the underlying stock does not pay a dividend, one should not exercise a call option early.
C. The option is worth at least as much as a similar European style option.
D. The option is worth at least as much as a similar option with more time to expiration.
E. All of A, B, C, and D are true.
10.4 (2 points) On March 1, ABC company stock is worth 20.
However, ABC has announced that on April 1 it will pay a liquidating dividend; ABC will pay its
entire value to its stockholders.
On March 1, compare the values of a European 10-strike call on ABC stock that expires
on March 15 and a similar option that expires on April 15.
10.5 (2 points) r = 0%. Stock ABC pays dividends. You own an American put on ABC.
Is it ever optimal to exercise this option before expiration? Briefly explain why.
10.6 (1 point) An American 85 strike 18 month put is an option on a stock with current price 81 and
forward price of 84. r = 3%.
Which of the following intervals represents the range of possible premiums for this option?
A. [0, 81] B. [0, 84] C. [0, 85] D. [0.96, 84] E. [0.96, 85]
10.7 (1 point) Which of the following statements is true?
1. A European put is worth at least as much as a similar put with more time until expiration.
2. A European call is worth at least as much as a similar call with more time until expiration.
3. A European option is worth at most as much as a similar American style option.
A. 1 B. 2 C. 3 D. 1, 2, 3 E. None of A, B, C, or D
10.8 (1 point) You have an American call option to exchange one share of Stock A for one share of
Stock B. Neither stock pays dividends. In what circumstances might you exercise this option early?
10.9 (3 points) On January 1 you purchase a 1 year $90 strike American call. r = 6%.
The stock will pay dividends of $1 each on: March 1, June 1, September 1, and December 1.
At what points in time might it be optimal to exercise your call?
2014-MFE, Financial Economics 10 American Options HCM 11/13/13, Page 211
10.10 (2 points) XYZ company goes bankrupt on January 1 and its stock becomes worthless.
On January 1, compare the values of a European 100-strike put on XYZ stock that expires April 1
and a similar option that expires July 1.
10.11 (1 point) You have an American call option with strike price of $100. r = 5% and # = 2%.
If the stock price had zero volatility, under what circumstances should you exercise this option early?
10.12 (2 points) r = 6%.
Consider otherwise similar American and European options on a stock that does not pay dividends.
Which of the following could be a graph of the option premiums as a function of the initial stock price,
holding everything else constant?
In each case, the American option premium is shown as dashed.
90 100 110 120
S0
5
10
15
20
25
C
A.
90 100 110 120
S0
5
10
15
20
C
B.
90 100 110 120
S0
5
10
15
P
C.
90 100 110 120
S0
5
10
15
20
P
D.
E. None of A, B, C, or D
10.13 (2 points) Briefly discuss early exercise of American options if the stock has a volatility of
zero, / = 0, in other words if the future price of the stock were deterministic rather than random.
2014-MFE, Financial Economics 10 American Options HCM 11/13/13, Page 212
10.14 (1 point) An American 95 strike 3 year call is an option on a stock with current price 90 and
forward price of 100. r = 5%. Determine the width of the range of possible prices of this option.
A. 84 B. 86 C. 88 D. 90 E. 92
10.15 (4 points) Use the following information:
A 2-year American put option on Euros has K = $1.30.
r
$
= 4%.
r
%
= 6%.
Draw and label a diagram to show bounds (ignoring transaction costs) on the possible values of the
American put as a function of the exchange rate when the option is bought.
The bounds illustrated should be independent of any model of the movement of exchange rates.
You should explain the rationale behind the bounds shown.
10.16 (2 points) You own a six-month 50-strike American call option on a stock.
The stock pays dividends at the continuously compounded annual rate of 2%.
The stock currently sells at 54, and is expected to sell at either 50 or 62 six months from now.
What annual continuously compounded risk-free interest rate would equate the value of exercising
the call immediately with the present value of waiting until the end of the term?
A. 1% B. 2% C. 3% D. 4% E. 5%
10.17 (2 points) Which of the following statements is true?
i. For an American option, the early exercise criterion becomes less stringent closer to expiration.
ii. For American calls, the early exercise boundary is higher for larger volatility.
iii. For American puts, the early exercise boundary is higher for larger volatility.
(A) Only (i) is true
(B) Only (ii) is true
(C) Only (i) and (ii) are true
(D) Only (i) and (iii) are true
(E) (i), (ii) and (iii) are true
10.18 (1 point) An American 70-strike six-month put is an option on a stock with current price of 74.
# = 1%. r = 4%.
Determine the range of possible put premiums.
2014-MFE, Financial Economics 10 American Options HCM 11/13/13, Page 213
10.19 (2 points) An 80-strike American call has a premium of 21.
A similar 110-strike American call has a premium of 9.
Based on each of the three properties of American option premiums, what is the range of possible
prices for a similar 100-strike American call?
10.20 (2 points) Consider an American 70-strike six-month call on a stock.
The stock will pay a dividend of $1 in one month and another dividend of $1 in four months.
If r = 8%, when might it be optimal to exercise this call? Briefly explain why.
10.21 (4 points) Use the following information:
A 5-year American call on a stock has K = 100.
r = 4%.
# = 3%.
Draw and label a diagram to show bounds (ignoring transaction costs) on the possible values of the
American call as a function of the stock price when the option is bought.
The bounds illustrated should be independent of any model of the movement of stock prices.
You should explain the rationale behind the bounds shown.
10.22 (2 points) A 50-strike American put has a premium of 7.
A similar 80-strike American put has a premium of 28.
Based on each of the three properties of American option premiums, what is the range of possible
prices for a similar 60-strike American put?
10.23 (3 points) Consider an American 100-strike one-year call on a stock.
The stock will pay a dividend in two months, five months, eight months, and eleven months.
Each of these dividends will be of the same size D.
r = 7%.
What is the smallest value of D such that is possible that this American Call will be worth more than
the similar European call?
A. Less than 0.50
B. At least 0.50, but less than 0.75
C. At least 0.75, but less than 1.00
D. At least 1.00, but less than 1.25
E. At least 1.25
2014-MFE, Financial Economics 10 American Options HCM 11/13/13, Page 214
10.24 (8 points) Consider European and American options on a stock. You are given:
(i) All options have the same strike price of 50.
(ii) All options expire in 2 years.
(iii) The continuously compounded risk-free interest rate is 7%.
(iv) Dividends are paid at the continuously compounded rate of 3%.
For each option, graph the area of possible premiums as a function of the current stock price.
Put the possible stock prices on the horizontal axis, and put the possible option premiums on the
vertical axis.
(a) American Call
(b) European Call
(c) American Put
(d) European Put
10.25 (1 point) On January 1, you purchased an American put option on Stock ABC, with a strike
price of $100 and an expiration date of December 31. Stock ABC pays no dividends.
During the first half of the year, the stock price for ABC stock fell rapidly, and it stood at $60 on
July 1. You expect a "market correction," with the stock price increasing to about $80 by year end.
If you exercise the option on July 1, you will have a net payoff of $40.
If you wait until December 31, you expect to gain only $20. What ought you to do?
10.26 (CAS5B, 11/91, Q.62) (1 point)
The difference between an European and an American option is:
A. A European option is an specific form of a call option while an American option is a specific form
of a put option.
B. A European option can be exercised only at the maturity date of the option where an American
option can be exercised at any time up to and including the maturity date of the option.
C. A European options contains a specific strike price but an American option does not.
D. A European option can be exercised on or before the maturity date and an American option can
be exercised only on one particular day.
E. None of the above.
10.27 (CAS5B, 11/91, Q.65) (1 point)
Which of the following statements about puts and calls are true?
1. When a stock does not pay dividends, an American call is always worth more alive than dead.
2. An American put is always more valuable than a European put.
3. The Black-Scholes formula can be used to value a European put.
A. 1 B. 1, 2 C. 1, 3 D. 2, 3 E. 1, 2, 3
2014-MFE, Financial Economics 10 American Options HCM 11/13/13, Page 215
10.28 (CAS5B, 11/93, Q.13) (1 point) Suppose that just after-you buy an American put and an
European put on a stock, the stock value drops to zero. The effective annual risk-free rate is 10%.
After the drop, the difference in value between the American put and the European put on the stock
is $50. Assume that the term for both options is one year and that they have the same strike price.
In what range does the strike price fall?
A. Less than $450
B. At least $450, but less than $550
C. At least $550, but less than $650
D. $650 or more
E. Cannot be answered from the information given.
10.29 (CAS5B, 11/94, Q.32) (3 points) A non-dividend paying stock, currently valued at $100,
may increase by 60% or decrease by 40% at year end. Assume an investor owns an American put
with a one-year term and a $110 strike price.
What is the continuously compounded annual risk-free rate that makes the investor indifferent to
exercising now or holding the put? Show all work.
10.30 (CAS5B, 11/98, Q.35) (2 points) You have purchased a two-year American call option on
a stock that pays an annual dividend. The dividend will always equal 10% of the share price and the
next dividend is payable one year from today. The strike price is equal to the current share price.
Each year the stock may increase in value by 20% or decrease in value by 20%. The value of the
stock will drop by the dividend amount immediately after it is paid. The continuously compounded
risk-free rate is 5%.
Assuming that investors are indifferent to risk, under what circumstances would you exercise the
option prior to expiration? Explain.
10.31 (CAS5B, 5/99, Q.34) (1 point) You own a one-year American put option on a non-
dividend paying stock with an strike price of $42. The stock currently sells at $40, and is expected to
sell at either $38 or $48 one year from now. What annual continuously compounded risk-free interest
rate would equate the value of exercising the put immediately with the present value of waiting until
the end of the term? Show all work.
10.32 (IOA 109 Specimen Exam 5/99, Q.5) (3.75 points) Draw a diagram to show bounds
(ignoring transaction costs) on the possible values of an American call option as a function of the
price of the underlying security. The bounds illustrated should be independent of any model for the
price process followed by the underlying security. You should explain the rationale behind the
bounds shown.
2014-MFE, Financial Economics 10 American Options HCM 11/13/13, Page 216
10.33 (CAS5B, 11/99, Q.15) (1 point)
For which of the following options might it be rational to exercise before maturity?
1. American put on a dividend-paying stock
2. American put on a non-dividend-paying stock
3. American call on a dividend-paying stock
A. 1 B. 3 C. 1, 2 D. 2, 3 E. 1, 2, 3
2014-MFE, Financial Economics 10 American Options HCM 11/13/13, Page 217
10.34 (MFE Sample Exam, Q.26)
Consider European and American options on a nondividend-paying stock. You are given:
(i) All options have the same strike price of 100.
(ii) All options expire in six months.
(iii) The continuously compounded risk-free interest rate is 10%.
You are interested in the graph for the price of an option as a function of the current stock
price. In each of the following four charts I - IV, the horizontal axis, S, represents the
current stock price, and the vertical axis, 5, represents the price of an option.


100
S
100
5
5 = S
I .
5 = S - 100

95.12
S
95.12
5
5 = S
5 = S - 95.12
II.

100
S
100
5 = 100 - S
5
III.

95.12
S
95.12
5 = 95.12 - S
5
I V.
Match the option with the shaded region in which its graph lies.
If there are two or more possibilities, choose the chart with the smallest shaded region.
European Call American Call European Put American Put
(A) I I III III
(B) II I IV III
(C) II I III III
(D) II II IV III
(E) II II IV IV
2014-MFE, Financial Economics 10 American Options HCM 11/13/13, Page 218
10.35 (CAS3, 5/07, Q.12) (2.5 points)
Which of the following effects are correct on the price of a stock option?
I. The premiums would not decrease if the options were American rather than European.
II. For European put, the premiums increase when the stock price increases.
III. For American call, the premiums increase when the strike price increases.
A. I only B. I and II only C. I and III only D. II and III only E. I, II and III
10.36 (CAS3, 11/07, Q.13) (2.5 points) Given the following chart about call options on a particular
dividend paying stock, which option has the highest value?
Option Option Style Time Until Expiration Strike Price Stock Price
A European 1 year 50 42
B American 1 year 50 42
C European 2 years 50 42
D American 2 years 50 42
E American 2 years 55 42
A. Option A B. Option B C. Option C D. Option D E. Option E
2014-MFE, Financial Economics 10 American Options HCM 11/13/13, Page 219
Solutions to Problems:
10.1. For the American call option, dividends on the stock are the reason why we want to receive
the stock earlier. With the interest rate equal to zero, we do not have the benefit of earning interest
on the strike, which is normally a reason for waiting to exercise the call. However, there is another
benefit to waiting to exercise the call, the insurance protection against the stock price moving below
the strike price. We will not exercise the option if it is out-of-the-money. Therefore, there may be
circumstances in which we will early exercise, but we will not always early exercise.
Comment: Similar to question 9.13 in Derivatives Markets by McDonald.
10.2. The underlying asset is Stock A, and the strike consists of Stock B. As Stock B does
not pay a dividend, the interest rate on Stock B is zero. We will therefore never early exercise the
put option, because we cannot receive earlier any benefits associated with holding Stock B there
are none. As Stock B does not pay dividends, we are indifferent between holding the option and
the stock.
Comment: Similar to question 9.14 in Derivatives Markets by McDonald.
10.3. D. Let us compare an option with three years to expiration to a similar option with only two
years to expiration. If we decide to always exercise the first option within two years, then its value is
that of the second option. So an American style option is worth at least as much as a similar option
with less time to expiration.
10.4. The call that expires March 15 will have a positive payoff provided the stock price is more
than 10. This seems likely. In any case, this call premium is positive
Once the liquidating dividend is paid on April 1, ABC stock will become worthless.
Even though one would like to early exercise the call that expires April 15, we cannot since it is
European, rather than American. By the time we can exercise this call, the option to buy ABC stock
at 10 will not be used; this call expires worthless.
In this situation, the call that expires March 15 is worth more than the call that expires April 15.
Comment: An example of how even though the premium of a European call on a dividend paying
stock usually increases with time until expiration, this is not always true.
10.5. For the American put option, when the interest rate is zero, no interest on the strike price is
lost by waiting until maturity to exercise. We still have a volatility benefit from waiting. Therefore, we
should never use the early exercise feature of the American put option.
Comment: If the interest rate is zero, then an American put is worth the same as a European put.
Similar to question 9.13 in Derivatives Markets by McDonald.
2014-MFE, Financial Economics 10 American Options HCM 11/13/13, Page 220
10.6. E. K " P(S
0
, K, T) " (PV
0,T
[K] - PV
0,T
[F
0,T
])
+
.
85 " P " (85 e
-.045
- 84 e
-.045
)
+
= 0.96.
Comment: While not discussed in Derivatives Markets by McDonald, if S
0
< K, then one could
exercise the American put immediately, getting a payoff of K - S
0
.
Therefore, P
Amer
(S
0
, K, T) " (K - S
0
)
+
= 85 - 81 = 4.
For a small dividend rate, PV
0,T
[K] - PV
0,T
[F
0,T
] = Ke
-rT
- S
0
e
-#T
< K - S
0
.
10.7. C. A European put is usually worth at least as much as a similar put with more time until
expiration, but not always.
A European call is usually worth at least as much as a similar call with more time until expiration, but
not always if dividends are paid.
A European option is worth at most as much as a similar American style option; usually the American
option is worth more.
10.8. The underlying asset is Stock A, which does not pay a dividend. Therefore, we
have an American call option on a non-dividend-paying stock. It is never optimal to early exercise
such an option.
Comment: Similar to question 9.14 in Derivatives Markets by McDonald.
2014-MFE, Financial Economics 10 American Options HCM 11/13/13, Page 221
10.9. It may be optimal to exercise at expiration, December 31 or January 1 of next year.
If it is optimal to exercise early, it will be just prior to the payment of one of the dividends.
It is not optimal to exercise early if: K - PV
t,T
[K] > PV
t,T
[Div].
On February 28 (or just before the ex-dividend day): K - PV
t,T
[K] = (90){1 - e
-0.06(10/12)
} = 4.39.
PV
t,T
[Div] = (1)(1 + e
-0.015
+ e
-0.03
+ e
-0.045
) = 3.91 < 4.39. Not optimal to exercise the call.
On May 30 (or just before the ex-dividend day):
K - PV
t,T
[K] = (90){1 - e
-0.06(7/12)
} = 3.10.
PV
t,T
[Div] = (1)(1 + e
-0.015
+ e
-0.03
) = 2.96 < 3.10. Not optimal to exercise the call.
On August 31 (or just before the ex-dividend day):
K - PV
t,T
[K] = (90){1 - e
-0.06(4/12)
} = 1.78.
PV
t,T
[Div] = (1)(1 + e
-0.015
) = 1.99 > 1.78. Might be optimal to exercise the call.
On November 30 (or just before the ex-dividend day):
K - PV
t,T
[K] = (90){1 - e
-0.06(1/12)
} = 0.45.
PV
t,T
[Div] = 1 > 0.45. Might be optimal to exercise the call.
Comment: If we do not exercise early, then at expiration we exercise if the call is in the money.
We can only determine times where it may be optimal to exercise the call early.
We never exercise early, if the option is not in the money!
On August 31 or November 30, it is optimal to exercise early if the call is sufficiently in the money. In
other words, on August 31 or November 30 we should exercise early if the stock price is sufficiently
high, although with the given information we can not determine how high.
According to Hull in Options, Futures, & Other Derivatives, not on the syllabus, when there are
a series of dividends D
i
at times t
i
, then it is not optimal to exercise at time t
i
if
D
i
# K(1 - exp[-r(t
i+1
- t
i
])]).
In this case, looking at August 31, K(1 - exp[-r(t
i+1
- t
i
]) = (90){1 - exp[-.06(1/4)]} = 1.34 > 1.
Therefore, it can not be optimal to exercise early at August 31.
10.10. Both puts will pay off 100 at expiration.
The put that expires April 1 has a present value of: 100e
-r/4
.
The put that expires July 1 has a present value of: 100e
-r/2
< 100e
-r/4
.
In this situation, the put that expires April 1 is worth more than the put that expires July 1.
Comment: An example of how even though the premium of a European put usually increases with
time until expiration, this is not always true.
2014-MFE, Financial Economics 10 American Options HCM 11/13/13, Page 222
10.11. Since / = 0, the implicit insurance protection against the stock price moving below the strike
price is zero. Therefore, it is optimal to exercise early whenever the dividends gained by exercising
early exceed the interest on the strike price. # S > r K. % S > (r/#)K = $250.
Exercise early when S > $250.
Comment: See Section 11.1 of Derivatives Markets by McDonald.
10.12. D. Since there are no dividends, the American call is worth the same as the European call,
eliminating A and B.
With r > 0, the American put is worth more than the European put.
As the stock price gets smaller, it is more likely to be advantageous to exercise early, and the
difference between the American and European put premiums increases, as in graph D.
Comment: For a very high initial stock price, both the European and American puts would be
virtually worthless; as the initial stock price approaches infinity, the difference in these put premiums
approaches zero. For a very low initial stock price, one would probably exercise the American put
immediately for an immediate payoff of K - S
0
, while the payoff on the European put would take
place at time T. As the initial stock price approaches zero, the American put premium approaches
K, the European put premium approaches Ke
-rT
, and the difference in these put premiums
approaches: K - Ke
-rT
.
10.13. If / = 0, then the implicit insurance protection is zero.
Thus for an American call, we would exercise early if the value of the dividends gained is greater than
the value of the interest on the strike price that would be lost. We would exercise an American call
early if: #S > rK 3 S > rK/#.
For an American put, we would exercise early if the value of the dividends lost is less than the value
of the interest on the strike price that would be gained. We would exercise an American put early if:
#S < rK 3 S < rK/#.
Comment: See Section 11.1 of Derivatives Markets by McDonald.
2014-MFE, Financial Economics 10 American Options HCM 11/13/13, Page 223
10.14. B. S
0
" C
Am
(S
0
, K, T) " (PV
0,T
[F
0,T
] - PV
0,T
[K])
+
" 0.
90 " C
Am
" (100 e
-.15
- 95 e
-.15
)
+
= 4.30. The width of the interval is: 90 - 4.30 = 85.70.
Comment: The parties agree now that the forward price, F
0,3
, will be paid 3 years from now in order
to obtain a share of stock three years from now.
PV[F
0,T
] = S
0
- PV[Div]. % 100 e
-.15
= 90 - PV[Div]. % PV[Div] = 90 - 100 e
-.15
= 3.93.
The prepaid forward price,

F
0, 3
P
, would instead be paid now in order to own a share of stock three
years from now.
If there were continuous dividends at a continuously compounded rate of 1.5%, then the prepaid
forward price of the stock would be: 90e
-(3)(.015)
= 86.04 6 86.07 = 100 e
-.15
, the given prepaid
forward price.
While not discussed in Derivatives Markets by McDonald, if S
0
> K, then one could exercise the
American call immediately, getting a payoff of S
0
- K.
Therefore, C
Amer
(S
0
, K, T) " (S
0
- K)
+
= (90 - 95)
+
= 0, which in this case has no effect.
If the dividend rate were large so that (S
0
- K)
+
> (PV
0,T
[F
0,T
] - PV
0,T
[K])
+
, such as when dealing
with options on currency, then this would have an effect.
2014-MFE, Financial Economics 10 American Options HCM 11/13/13, Page 224
10.15. Dollars act as money and Euros act as the asset.
r 3 r
$
= 4%. # 3 r
%
= 6%. Current exchange rate = X 3 S(0).
PV
0,T
[K] - PV
0,T
[F
0,T
] = 1.3 e
-(2)(4%)
- X e
-(2)(6%)
= 1.200 - X e
-0.12
.
This is zero for: X = 1.3 e
0.04
= 1.353. Thus for X > 1.353, (1.200 - X e
-0.12
)
+
= 0.
K " P " (PV
0,T
[K] - PV
0,T
[F
0,T
])
+
. 3 1.30 " P " (1.200 - X e
-0.12
)
+
Also, if the American put is in the money, then one could exercise it immediately, getting a payoff of
K - X. Therefore, P
Amer
(S
0
, K, T) " (K - X)
+
= (1.30 - X)
+

Thus P " Max[0, 1.30 - X, 1.200 - X e
-0.12
].
Setting 1.30 - X = 1.200 - X e
-0.12
. % X = 0.8839.
Thus this lower bound on P is:

1.3 - X for X , 0.8839
1.200 - X e
- 0.12
for 1.353 & X & 0.8839
0 for X & 1.353
'
(
-
)
-
.
Recalling that in addition, P # 1.30, the diagram is as follows:
0.8839 1.353
X
0.4161
1.3
P
Area of
Possible
Premiums 1.3 - X
1.2 - X Exp[-0.12]
0
0
2014-MFE, Financial Economics 10 American Options HCM 11/13/13, Page 225
10.16. B. If we exercise right away, we get $4.
At the up node the payoff would be 12, while at the down node the payoff would be zero.
The continuation value is: 12p* / e
r/2
.
p* = (e
(r-0.02)/2
- 50/54) / (62/54 - 50/54) = (54e
-0.01
e
r/2
- 50)/(62 - 50) = 4.4552e
r/2
- 25/6.
We want 4 = 12p* / e
r/2
. % e
r/2
/ 3 = p* = 4.4552e
r/2
- 25/6 % e
r/2
= 1.011. % r = 2.2%.
Comment: Similar to 5B, 5/99, Q.34.
10.17. C. As we get closer to expiration, the implicit insurance protection gets smaller.
Therefore, the continuation value gets smaller, and we are more likely to exercise early.
(For calls the early exercise boundary gets lower, while for puts the early exercise boundary gets
higher, as we approach expiration.) Statement 1 is true.
As sigma gets larger, the implicit insurance protection gets larger.
Therefore, the continuation value of the option is bigger.
Therefore, the exercise value of the option has to be bigger in order for it to be worthwhile to
exercise the option. For calls this means the stock price needs to be higher, but for puts this means
the stock price has to be lower. Statement 2 is true, while statement 3 is false.
Comment: See Figures 11.1 and 11.2 in Derivatives Markets by McDonald.
10.18. PV
0,T
[K] - PV
0,T
[F
0,T
] = K e
-rT
- S(0) e
-#T
= 70 e
-.04/2
- 74 e
-.01/2
= -5.01.
K " P(S
0
, K, T) " (PV
0,T
[K] - PV
0,T
[F
0,T
])
+
.
70 " P " 0. The put premium is at least 0 and at most 70.
Comment: In this case the American put is out of the money. If instead the American put were in the
money, then one could exercise it immediately, getting a payoff of K - S
0
.
Therefore, P
Amer
(S
0
, K, T) " (K - S
0
)
+
.
10.19. The call premiums must decline as the strike price increases, therefore: 21 " C(100) " 9.
The absolute difference in the call premiums can be at most the difference in the strikes.
Thus, C(80) - C(100) # 100 - 80. % 21 - C(100) # 20. % C(100) " 1.
Also, C(100) - C(110) # 110 - 100. % C(100) - 9 # 10. % C(100) # 19.
In addition by convexity, C(100) # (1/3)(21) + (2/3)(9) = 13.
Combining all of the restrictions, 13 " C(100) " 9.
2014-MFE, Financial Economics 10 American Options HCM 11/13/13, Page 226
10.20. If it is optimal to exercise an American call early, and dividends are paid at discrete times,
then it is best to exercise right before the payment of a (large) dividend.
So, it might be optimal to exercise this call at expiration, or in one month (just before the payment of
that dividend), or in four months (just before the payment of that dividend).
However, if K{1 - exp[-r(T- t)]} is greater than the present value of the future dividends that will be
paid until the call expires, then it is better to wait rather than exercise early.
At one month: K{1 - exp[-r(T- t)]} = (70){1 - exp[-(8%)(5/12)} = 2.295.
The present value of future dividends is: 1 + 1 exp[-(8%)(3/12)] = 1.980.
Since 2.295 > 1.980, it is better to wait.
(The benefit from the interest on the strike price exceeds the benefit of exercising early and getting
the stock dividends.)
At four months: K{1 - exp[-r(T- t)]} = (70){1 - exp[-(8%)(2/12)} = 0.927.
The present value of future dividends is 1 > 0.927.
So it may be better to exercise early.
It may be optimal to exercise in four months (just before the ex-dividend date) and in six
months (at expiration.)
Comment: Whether it is optimal to exercise early at four months would depend on the stock price
then; for a sufficiently high stock price, it would make sense to exercise early the call.
If we do not exercise in four months, then it is optimal to wait to expiration; we would then exercise at
expiration if the call were in-the-money.
2014-MFE, Financial Economics 10 American Options HCM 11/13/13, Page 227
10.21. PV
0,T
[F
0,T
] - PV
0,T
[K] = S e
-(5)(3%)
- 100 e
-(5)(4%)
= S e
-0.15
- 81.87.
This is zero for: S = 100 e
-0.05
= 95.12. Thus for S < 95.12, (S e
-0.15
- 81.87)
+
= 0.
S
0
" C " (PV
0,T
[F
0,T
] - PV
0,T
[K])
+
. 3 100 " C " (S e
-0.15
- 81.87)
+
.
Also, if the American call is in the money, then one could exercise it immediately, getting a payoff of
S - 100. Therefore, C
Amer
(S
0
, K, T) " (S - K)
+
= (S - 100)
+

Thus C " Max[0, S - 100, S e
-0.15
- 81.87].
Setting S - 100 = S e
-0.15
- 81.87. % S = 130.16.
Thus this lower bound on C is:

0 for S , 95.12
S e
- 0.15
- 81.87 for 130.16 & S & 95.12
S - 100 for S & 130.16
'
(
-
)
-
.
Recalling that in addition, C # S, the diagram is as follows:
95.12 130.16 200
S
30.16
100
200
C
C = S
Area of
Possible
Premiums
C = S exp[-0.15] - 81.87
C = S - 100
0
0
2014-MFE, Financial Economics 10 American Options HCM 11/13/13, Page 228
10.22. The put premiums must increase as the strike price increases, therefore: 28 " P(60) " 7.
The absolute difference in the premiums can be at most the difference in the strikes.
Thus P(60) - P(50) # 60 - 50. % P(60) - 7 # 10. % P(60) # 17.
Also, P(80) - P(60) # 80 - 60. % 28 - P(60) # 20. % P(60) " 8.
In addition by convexity, P(60) # (2/3)(7) + (1/3)(28) = 14.
Combining all of the restrictions, 14 " P(60) " 8.
10.23. B. In order for the American Call to be worth more than the European Call it must be
optimal to early exercise at some point in time.
If it is optimal to exercise an American call early, and dividends are paid at discrete times, then it is
best to exercise right before the payment of a (large) dividend.
However, if K{1 - exp[-r(T- t)]} is greater than the present value of the future dividends that will be
paid until the call expires, then it is better to wait rather than exercise early.
At two months: K{1 - exp[-r(T- t)]} = (100){1 - exp[-(7%)(10/12)]} = 5.666.
The present value of future dividends is:
D{1 + exp[-(7%)(3/12)] + exp[-(7%)(6/12)] + exp[-(7%)(9/12)]} = 3.8971D.
So it would be better to wait to exercise unless: 3.8971D > 5.666. % D > 1.45.
At five months: K{1 - exp[-r(T- t)]} = (100){1 - exp[-(7%)(7/12)]} = 4.001.
The present value of future dividends is:
D{1 + exp[-(7%)(3/12)] + exp[-(7%)(6/12)]} = 2.9483D.
So it would be better to wait to exercise unless: 2.9483D > 4.001. % D > 1.36.
At eight months: K{1 - exp[-r(T- t)]} = (100){1 - exp[-(7%)(4/12)]} = 2.306.
The present value of future dividends is: D{1 + exp[-(7%)(3/12)]} = 1.9827D.
So it would be better to wait to exercise unless: 1.9827D > 2.306. % D > 1.16.
At eleven months: K{1 - exp[-r(T- t)]} = (100){1 - exp[-(7%)(1/12)]} = 0.582.
The present value of future dividends is: D.
So it would be better to wait to exercise unless: D > 0.582.
Thus it is possible that it will be optimal to exercise early at some point in time, provided D > 0.58.
Comment: In this type of setup with dividends of equal size, the key comparison is when the last
dividend is paid.
2014-MFE, Financial Economics 10 American Options HCM 11/13/13, Page 229
10.24. PV
0,T
[K] = 50 e
-0.14
= 43.47. PV
0,T
[F
0,T
] = e
-0.06
S
0
= 0.942S
0
.
(a) S
0
" C
Amer
" C
Eur
" (PV
0,T
[F
0,T
] - PV
0,T
[K])
+
.
S
0
" C
Amer
" (0.942S
0
- 43.47)
+
.
Now, 0.942S
0
- 43.47 = 0. % S
0
= 46.15. Thus, for S
0
< 46.15, (0.942S
0
- 43.47)
+
= 0.
Also, since an American call can be exercised right away: C
Amer
" (S
0
- K)
+
= (S
0
- 50)
+
.
0.942S
0
- 43.47 = S
0
- 50. % S
0
= 112.59.
Thus, for 112.59 > S
0
> 46.15, the lower bound is: 0.942S
0
- 43.47.
For S
0
> 112.59, the lower bound is: S
0
- 50.
For S
0
< 46.15, the lower bound is 0.
Thus, we get the following graph of possible premiums of the American Call:


C = S
C = S - 50
C = 0.942 S - 43.47
46.15 112.59
S
62.59
C
Area of
Possible
Premiums
2014-MFE, Financial Economics 10 American Options HCM 11/13/13, Page 230
(b) Since a European call can only be exercised at expiration: PV
0,T
[F
0,T
] " C
Eur
.
Thus PV
0,T
[F
0,T
] " C
Eur
" (PV
0,T
[F
0,T
] - PV
0,T
[K])
+
.
0.942S
0
" C
Eur
" (0.942S
0
- 43.47)
+
.
The lower bound is zero for S
0
< 46.15.
For S
0
> 46.15, the lower bound is: 0.942S
0
- 43.47.
Thus, we get the following graph of possible premiums of the European Call:

46.15
S
50
100
150
C
Area of
Possible
Premiums
C = 0.942S - 43.47
C = 0.942 S
2014-MFE, Financial Economics 10 American Options HCM 11/13/13, Page 231
(c) K " P
Amer
" P
Eur
" (PV
0,T
[K] - PV
0,T
[F
0,T
])
+
.
Also, since an American put can be exercised right away: P
Amer
" (K - S
0
)
+
.
Thus, 50 " P
Amer
" Max[(50 - S
0
)
+
, (43.47 - 0.942S
0
)
+
.]
For S
0
< 112.59, (50 - S
0
)
+
> (43.47 - 0.942S
0
)
+
.
Thus only the bigger lower bound, (50 - S
0
)
+
, is relevant.
For S
0
> 50, (50 - S
0
)
+
= 0.
Thus for S
0
> 50, the lower bound is 0.
For S
0
< 50, the lower bound is: 50 - S
0
.
Thus, we get the following graph of possible premiums of the American Put:
Area of
Possible
Premiums
50 - S
50
S
50
P
2014-MFE, Financial Economics 10 American Options HCM 11/13/13, Page 232
(d) K " P
Eur
" (PV
0,T
[K] - PV
0,T
[F
0,T
])
+
" 0.
Also, since a European put can only be exercised at expiration: PV
0,T
[K] " P
Eur
.
Thus, 43.47 " P
Eur
" (43.47 - 0.942S
0
)
+
. For S
0
> 46.15, the lower bound is zero.
Thus, we get the following graph of possible premiums of the European Put:
Area of
Possible
Premiums
43.47 - 0.942 S
46.15
S
43.47
P
Comment: Similar to MFE Sample Exam, Q.26.
10.25. If the market consensus agreed with you about the future behavior of ABC stock, then the
price of the stock on July 1 would not be $60.
What you should do is see what the put is selling for on July 1. If the put is selling for more than $40
you should sell it; otherwise you should exercise the put on July 1 for a $40 payoff.
10.26. B.
10.27. E. All are true. Statement #1 is another way of saying that i the absence of dividends it is
never optimal to exercise an American option prior to expiration.
2014-MFE, Financial Economics 10 American Options HCM 11/13/13, Page 233
10.28. C. A market price of $0 implies that the stock will not rise in value in the future. (A market
price of $0 may result when the firm declares bankruptcy and closes its doors, with no intention of
reorganizing.) The holder of the American put option can exercise the option any time between the
purchase date and the expiration date. Since the market value of the stock will remain $0 during the
entire period, the gain from the exercising the option is the same no matter when it is exercised: the
gain is the strike price. The earlier the put option is exercised, the more time the owner has to invest
the proceeds. Thus the holder of the American put will exercise it immediately, with payoff K.
In contrast, the European put will be exercised in one year; the present value of the payoff is K/1.1.
Thus: K - K/1.1 = 50. % K = 550.
10.29. p* = (100 e
r
- 60) / (160 - 60) = e
r
- 0.6.
The exercise value is 10.
The continuation value is: (110 - 60) (1 - p*) e
-r
= (50)(1.6e
-r
- 1) = 80e
-r
- 50.
Setting the continuation value equal to the exercise value: 10 = 80e
-r
- 50. % r = 28.8%.
10.30. Assume an initial share price of 100, solely for illustrative purposes.
Then in one year, cum-dividend, the prices are 80 or 120.
Ex-dividend the prices are: (80)(0.9) = 72, and (120)(0.9) = 108.
129.6
120 7 108
100 86.4
80 7 72
57.6
If we get to the 120 price node, then if we early exercise, we get a payoff of 120 - 100 = 20.
We want to earn 5% in the risk neutral environment.
Prior to getting the dividend: 120p* + 80(1 - p*) = 100e
.05
.
p* = (e
.05
- 0.8)/(1.2 - 0.8) = 62.8%.
Alternately, after getting the dividend: (110 + 10)p* + (72 + 8)(1 - p*) = 100e
0.05
. % p* = 62.8%.
At the 120 node, the continuation value is: (129.6 - 1000)(.628)/e
0.05
= 17.68 < 20.
Therefore, it is optimal to exercise early at the up node, prior to getting the dividend.
At the down node, the call is out of the money, so we would not exercise early.
Unfortunately, waiting in this case also leads to no payoff.
Comment: American call premium is: (20)(.628) / e
0.05
= 11.95.
2014-MFE, Financial Economics 10 American Options HCM 11/13/13, Page 234
10.31. If we exercise right away, we get $2.
The continuation value is: 4(1 - p*)/e
r
.
p* = (e
r
- 38/40)/(48/40 - 38/40) = (40e
r
- 38)/(48 - 38) = 4e
r
- 3.8.
We want 2 = 4(1 - p*)/e
r
. % .5e
r
= 1 - p* = 4.8 - 4e
r
. % e
r
= 4.8/4.5. % r = 6.45%.
10.32. The underlying share price is an upper bound because if the call were priced higher than the
share it would be possible to sell the call and buy the share to make a risk free immediate profit.
The value of the call must also be greater than zero and greater than the share price less the present
value of the strike price (exercise price) discounted at the risk free rate. If the call were priced less
than this it would be possible to make a risk free profit by buying the call, selling the share and
investing the discounted strike price (exercise price) at the risk free rate.
Where X is the strike price (exercise price):
Comment: The bounds would be the same if it were a European rather than American call.
If there are dividends paid on the stock, then C " S - X exp[-r(t - t)] - PV[Div.].
10.33. E. One could exercise early in any of the given cases. One should not exercise before
maturity an American call on a non-dividend-paying stock.
2014-MFE, Financial Economics 10 American Options HCM 11/13/13, Page 235
10.34. D. S
0
" C
Amer
" C
Eur
" (PV
0,T
[F
0,T
] - PV
0,T
[K])
+
.
Because the stock pays no dividends, PV
0,T
[F
0,T
] = S
0
.
PV
0,T
[K] = K e
-rT
= 100 e
-0.1/2
= 95.12.
Therefore, S
0
" C
Amer
" C
Eur
" (S
0
- 95.12)
+
.
Thus, the shaded region in II contains C
Amer
and C
Eur
.
(The shaded region in I also does, but it is a larger region than II.)
K " P
Eur
" (PV
0,T
[K] - PV
0,T
[F
0,T
])
+
" 0.
In fact, since a European put can only be exercised at expiration: PV
0,T
[K] " P
Eur
.
Thus, 95.12 " P
Eur
" (95.12 - S
0
)
+
.
Thus, the shaded region in IV contains P
Eur
.
K " P
Amer
" P
Eur
" (PV
0,T
[K] - PV
0,T
[F
0,T
])
+
.
In fact, since an American put can be exercised right away: P
Amer
" (K - S
0
)
+
.
Thus, 100 " P
Amer
" (100 - S
0
)
+
.
Thus, the shaded region in III contains P
Amer
.
Comment: In fact, since a European call can only be exercised at expiration: PV
0,T
[F
0,T
] " C
Eur
.
In this case, since there are no dividends, PV
0,T
[F
0,T
] = S
0
, and this makes no difference.
In fact, since an American call can be exercised right away: C
Amer
" (S
0
- K)
+
.
In this case, since there are no dividends, PV
0,T
[F
0,T
] = S
0
, and this makes no difference.
10.35. A. 1. True. An American option is worth at least as much as a similar European Option.
2. False. The premium of any put decreases as the price of the stock increases.
(K - S
T
)
+
decreases as S
T
increases.
3. False. The premium of any call decreases as the strike price increases.
(S
T
- K)
+
decreases as K increases.
10.36. D. Since an American option is worth at least as much as a similar European option, B " A,
and D " C.
Since an American option with more time to expiration is worth at least as much as a similar American
option with less time until expiration. D " B.
Since a call with a lower strike price is worth at least as much as as a similar call with a higher strike
price, D " E. We conclude that option D has the highest value.
2014-MFE, Financial Economics 10 American Options HCM 11/13/13, Page 236

You might also like